0% found this document useful (0 votes)
140 views225 pages

500 Micro Mcqs

Uploaded by

wyanru86
Copyright
© © All Rights Reserved
We take content rights seriously. If you suspect this is your content, claim it here.
Available Formats
Download as PDF, TXT or read online on Scribd
0% found this document useful (0 votes)
140 views225 pages

500 Micro Mcqs

Uploaded by

wyanru86
Copyright
© © All Rights Reserved
We take content rights seriously. If you suspect this is your content, claim it here.
Available Formats
Download as PDF, TXT or read online on Scribd
You are on page 1/ 225

5

5 STEPS TO A

500
AP Microeconomics
Questions
to know by test day

AP Microeconomics_p001-224.indd 1 22/12/20 5:00 PM


Also in the 5 Steps series:
5 Steps to a 5: AP Macroeconomics 2021
5 Steps to a 5: AP Macroeconomics 2021 Elite Student Edition
5 Steps to a 5: AP Microeconomics 2021
5 Steps to a 5: AP Microeconomics 2021 Elite Student Edition
5 Steps to a 5: AP Microeconomics/Macroeconomics Flashcards

Also in the 500 AP Questions to Know by Test Day series:


5 Steps to a 5: 500 AP Biology Questions to Know by Test Day, Third Edition
5 Steps to a 5: 500 AP Calculus AB/BC Questions to Know by Test Day, Third Edition
5 Steps to a 5: 500 AP Chemistry Questions to Know by Test Day, Third Edition
5 Steps to a 5: 500 AP English Language Questions to Know by Test Day, Third Edition
5 Steps to a 5: 500 AP English Literature Questions to Know by Test Day, Third Edition
5 Steps to a 5: 500 AP Environmental Science Questions to Know by Test Day, Third Edition
5 Steps to a 5: 500 AP European History Questions to Know by Test Day, Third Edition
5 Steps to a 5: 500 AP Human Geography to Know by Test Day, Third Edition
5 Steps to a 5: 500 AP Macroeconomics Questions to Know by Test Day, Third Edition
5 Steps to a 5: 500 AP Physics 1 Questions to Know by Test Day, Third Edition
5 Steps to a 5: 500 AP Physics C Questions to Know by Test Day
5 Steps to a 5: 500 AP Psychology Questions to Know by Test Day, Third Edition
5 Steps to a 5: 500 AP Statistics Questions to Know by Test Day, Third Edition
5 Steps to a 5: 500 AP U.S. Government & Politics Questions to Know by Test Day, Second Edition
5 Steps to a 5: 500 AP U.S. History Questions to Know by Test Day, Third Edition
5 Steps to a 5: 500 AP World History Questions to Know by Test Day, Third Edition

AP Microeconomics_p001-224.indd 2 22/12/20 5:00 PM


5

5 STEPS TO A

500
AP Microeconomics
Questions
to know by test day
THIRD EDITION

Anaxos, Inc.
Brian Reddington

AP Microeconomics_p001-224.indd 3 22/12/20 5:00 PM


Anaxos, Inc. (Austin, TX) has been creating education and reference materials for over fifteen years. Based in
Austin, Texas, the company uses writers from across the globe who offer expertise on an array of subjects just
as expansive.

Brian Reddington earned his bachelor’s degree in history from Stony Brook University in Stony Brook, New
York, before pursuing a master’s degree in special education at Queens College in New York City. A dedicated
educator, he has been teaching economics since 2008. He lives in Bellmore, New York, with his wife and two
young daughters, Anna June and Sarah Gretchen.

Copyright © 2021, 2017, 2012 by McGraw Hill. All rights reserved. Except as permitted under the United States
Copyright Act of 1976, no part of this publication may be reproduced or distributed in any form or by any means,
or stored in a database or retrieval system, without the prior written permission of the publisher.

ISBN: 978-1-26-047470-1
MHID: 1-26-047470-4

The material in this eBook also appears in the print version of this title: ISBN: 978-1-26-047469-5,
MHID: 1-26-047469-0.

eBook conversion by codeMantra


Version 1.0

All trademarks are trademarks of their respective owners. Rather than put a trademark symbol after every occur-
rence of a trademarked name, we use names in an editorial fashion only, and to the benefit of the trademark owner,
with no intention of infringement of the trademark. Where such designations appear in this book, they have been
printed with initial caps.

McGraw-Hill Education eBooks are available at special quantity discounts to use as premiums and sales promo-
tions or for use in corporate training programs. To contact a representative, please visit the Contact Us page at
www.mhprofessional.com.

McGraw Hill, the McGraw Hill logo, 5 Steps to a 5, and related trade dress are trademarks or registered trade-
marks of McGraw Hill and/or its affiliates in the United States and other countries and may not be used without
written permission. All other trademarks are the property of their respective owners. McGraw Hill is not associ-
ated with any product or vendor mentioned in this book.

AP, Advanced Placement Program, and College Board are registered trademarks of the College Board, which was
not involved in the production of, and does not endorse, this product.

TERMS OF USE

This is a copyrighted work and McGraw-Hill Education and its licensors reserve all rights in and to the work. Use
of this work is subject to these terms. Except as permitted under the Copyright Act of 1976 and the right to store
and retrieve one copy of the work, you may not decompile, disassemble, reverse engineer, reproduce, modify,
create derivative works based upon, transmit, distribute, disseminate, sell, publish or sublicense the work or any
part of it without McGraw-Hill Education’s prior consent. You may use the work for your own noncommercial
and personal use; any other use of the work is strictly prohibited. Your right to use the work may be terminated if
you fail to comply with these terms.

THE WORK IS PROVIDED “AS IS.” McGRAW-HILL EDUCATION AND ITS LICENSORS MAKE NO
GUARANTEES OR WARRANTIES AS TO THE ACCURACY, ADEQUACY OR COMPLETENESS OF
OR RESULTS TO BE OBTAINED FROM USING THE WORK, INCLUDING ANY INFORMATION THAT
CAN BE ACCESSED THROUGH THE WORK VIA HYPERLINK OR OTHERWISE, AND EXPRESSLY
DISCLAIM ANY WARRANTY, EXPRESS OR IMPLIED, INCLUDING BUT NOT LIMITED TO IMPLIED
WARRANTIES OF MERCHANTABILITY OR FITNESS FOR A PARTICULAR PURPOSE. McGraw-Hill
Education and its licensors do not warrant or guarantee that the functions contained in the work will meet your
requirements or that its operation will be uninterrupted or error free. Neither McGraw-Hill Education nor its
licensors shall be liable to you or anyone else for any inaccuracy, error or omission, regardless of cause, in the
work or for any damages resulting therefrom. McGraw-Hill Education has no responsibility for the content of
any information accessed through the work. Under no circumstances shall McGraw-Hill Education and/or its
licensors be liable for any indirect, incidental, special, punitive, consequential or similar damages that result from
the use of or inability to use the work, even if any of them has been advised of the possibility of such damages.
This limitation of liability shall apply to any claim or cause whatsoever whether such claim or cause arises in
contract, tort or otherwise.
CONTENTS

Introduction vii
Diagnostic Quiz 1
Questions 1–20

Chapter 1 Basic Economic Concepts 19


Questions 21–84

Chapter 2 Supply and Demand 41


Questions 85–187

Chapter 3 Production, Cost, and the Perfect


Competition Model 75
Question 188–276

Chapter 4 Imperfect Competition 105


Question 277–354

Chapter 5 Factor Markets 127


Questions 355–439

Chapter 6 Market Failure and the Role of Government 149


Questions 440–500

Answers 167

❮ v

AP Microeconomics_p001-224.indd 5 22/12/20 5:00 PM


This page intentionally left blank

AP Microeconomics_p001-224.indd 6 22/12/20 5:00 PM


INTRODUCTION

Congratulations! You’ve taken a big step toward AP success by purchasing


5 Steps to a 5: 500 AP Microeconomics Questions to Know by Test Day. We are
here to help you take the next step and score high on your AP exam so you
can earn college credits and get into the college or university of your choice.
This book gives you 500 AP-​style multiple-​choice questions that cover all
the most essential course material. Each question has a detailed answer
explanation. These questions will give you valuable independent practice
to supplement your regular textbook and the groundwork you are already
doing in your AP classroom. This and the other books in this series were
written by expert AP teachers who know your exam inside out and can
identify the crucial exam information as well as questions that are most
likely to appear on the exam.
You might be the kind of student who takes several AP courses and needs
to study extra questions a few weeks before the exam for a final review.
Or you might be the kind of student who puts off preparing until the last
weeks before the exam. No matter what your preparation style is, you will
surely benefit from reviewing these 500 questions, which closely parallel
the content, format, and degree of difficulty of the questions on the actual
AP exam. These questions and their answer explanations are the ideal last-​
minute study tool for those final few weeks before the test.
Remember the old saying “Practice makes perfect.” If you practice with
all the questions and answers in this book, we are certain you will build the
skills and confidence you need to do great on the exam. Good luck!

—Editors of McGraw-​Hill Education

❮ vii

AP Microeconomics_p001-224.indd 7 22/12/20 5:00 PM


This page intentionally left blank

AP Microeconomics_p001-224.indd 8 22/12/20 5:00 PM


Diagnostic Quiz

AP Microeconomics_p001-224.indd 1 22/12/20 5:00 PM


This page intentionally left blank

AP Microeconomics_p001-224.indd 2 22/12/20 5:00 PM


GETTING STARTED:
THE DIAGNOSTIC QUIZ

The following questions refer to different units in this book. These questions
will help you test your understanding of the concepts tested on the AP exam
by giving you an idea of where you need to focus your attention as you
prepare. For each question, simply circle the letter of your choice. Once you
are done with the exam, check your work against the given answers, which
also indicate where you can find the corresponding material in the book.

Good luck!

❮ 3

AP Microeconomics_p001-224.indd 3 22/12/20 5:00 PM


This page intentionally left blank

AP Microeconomics_p001-224.indd 4 22/12/20 5:00 PM


DIAGNOSTIC QUIZ QUESTIONS
1. The opportunity cost in hours that it takes China and Switzerland
to produce mobile phones and clocks is shown in the following table.
According to the theory of comparative advantage, which of the
following is accurate?

China Switzerland
Cars 24 36
Clocks 20 15

(A) China should impose a tariff on Switzerland in order to make


its production of clocks less effective.
(B) China should produce cars and clocks because it is more
effective in producing both, while Switzerland should produce
something else, and the two countries should trade.
(C) China should produce cars, while Switzerland should produce
clocks; these countries should then establish trade relations.
(D) China should produce clocks, while Switzerland should
produce cars because these are the areas where they have the
higher opportunity costs; they should then establish trade
relations.
(E) The opportunity cost for clocks is 12 hours, while the
opportunity cost for producing clocks is 5 hours because this is
the difference between the two opportunity costs.

❮ 5

AP Microeconomics_p001-224.indd 5 22/12/20 5:00 PM


6 ❯ 500 AP Microeconomics Questions to Know by Test Day

2. James is a student who works several part-time jobs. He helps


out at a local restaurant, for which he is paid $14 per hour. He
also sometimes works at a library where he helps sort books, for
which he is paid $18 per hour. James decided that he needs to stop
working and study for an additional hour for a difficult history
exam. What is the opportunity cost of James studying history?
(A) The opportunity cost is $32 per hour because that is the total
value of the two jobs James could have done.
(B) The opportunity cost is $4 per hour because James is doing an
activity that is worth less than the most profitable opportunity.
(C) The opportunity cost is $14 per hour because this is the
minimal amount of income that James could have earned.
(D) There is no opportunity cost because focusing on education is
an intangible good that will benefit James.
(E) The opportunity cost is $18 because this is the most profitable
business activity that James could have selected.

3. The United Kingdom is trying to give its domestic producers of


wheat an opportunity to increase production. The main problem on
the market is the excess amount of Chinese wheat which prevents
domestic producers from making money. Which of the following
would be an example of implementing a quota on Chinese wheat?
(A) if the United Kingdom prohibited any Chinese wheat from
entering the U.K. market
(B) if the United Kingdom placed a tax on Chinese wheat which
made it 15 percent more expensive
(C) if the United Kingdom decided that all Chinese wheat needed
to pass an additional health inspection
(D) if the United Kingdom decided that only 10,000 tons of wheat
could be imported every year
(E) if the United Kingdom ordered that every Chinese producer
had to pay an administrative fee of 10 pounds

AP Microeconomics_p001-224.indd 6 22/12/20 5:00 PM


Getting Started: The Diagnostic Quiz  ❮ 7

4. Which of the following is shown on the diagram if supply shifts


from point A to point B?

Price
S
A
P2

P1 B

Q1 Q2
Quantity

(A) a contraction of supply


(B) an expansion of supply
(C) a shift in the supply curve to the left
(D) a shift in the supply curve to the right
(E) the deadweight cost of supply

5. What type of elasticity is shown in the following diagram?

Demand
Price

Quantity

(A) unit demand elasticity


(B) perfectly elastic demand
(C) relatively elastic demand
(D) relatively inelastic demand
(E) perfectly inelastic demand

AP Microeconomics_p001-224.indd 7 22/12/20 5:00 PM


8 ❯ 500 AP Microeconomics Questions to Know by Test Day

6. The following table shows the number of cars that a factory in


Michigan can produce with up to five workers. Which worker is
responsible for the largest marginal return?

Number of Workers Number of Cars Produced


0 0
1 5
2 12
3 14
4 16
5 19

(A) the first worker


(B) the second worker
(C) the third worker
(D) the fourth worker
(E) the fifth worker

7. Which of the following is NOT considered by accounting profit?


(A) transportation costs
(B) cost of wages
(C) marketing costs
(D) cost of overhead
(E) cost of financial capital

AP Microeconomics_p001-224.indd 8 22/12/20 5:00 PM


Getting Started: The Diagnostic Quiz  ❮ 9

8. The following table shows how much a plane manufacturer can


produce from opening additional plants. If opening a plant costs
$1,000,000 and the company earns $100,000 for every plane sold,
how much will the company earn if it wants to avoid the law of
diminishing returns?

Number of Plants Number of Planes Produced


0 0
1 11
2 24
3 25
4 27
5 26

(A) $100,000
(B) $200,000
(C) $350,000
(D) $400,000
(E) $500,000

9. Company A operates in a market that has many small companies,


none of which can impact the market price. The market is
characterized by consumers who learn most of their information
from advertisements and are not fully informed about the products.
The market can be considered effective, and no companies earn
economic profit in the long term. Why is this NOT an example of a
perfectly competitive market?
(A) because perfectly competitive markets do not have a higher
number of small companies
(B) because consumers should be rational and have all of the
relevant information in a perfectly competitive market
(C) because all companies have the ability to impact the price by
changing their output, meaning that companies are not price
takers in a perfectly competitive market
(D) because all companies make more than an economic profit in
a perfectly competitive environment; otherwise, they would go
out of business
(E) because perfectly competitive markets are not considered
effective, as economies of scale are not possible

AP Microeconomics_p001-224.indd 9 22/12/20 5:00 PM


10 ❯ 500 AP Microeconomics Questions to Know by Test Day

10. A new study found that Florida loses roughly $50,000,000 a year
because of alcohol addiction. The three domestic companies who are
the largest producers of alcohol earn roughly $200,000,000 every
year and produce 10,000,000 bottles of alcoholic drinks combined.
Which of the following would be the best option to make the social
benefit of alcohol equal to the economic cost of alcohol?
(A) if lawmakers decided that the companies should pay
$200,000,000 every year
(B) if lawmakers decided that the companies should pay a tax of $5
for every bottle produced
(C) if lawmakers decided that the companies should pay a tax of
$20 for every bottle produced
(D) if lawmakers decided that the companies should pay a tax of
$50 for every bottle produced
(E) if lawmakers decided that the companies should pay a tax of
$40 for every bottle produced

11. In one case of game theory, an individual selects strategy A, while


another individual selects strategy B. After hearing the alternative
option, both individuals have no incentive to change their selected
strategy. What best describes this concept?
(A) negative externality
(B) positive externality
(C) elasticity of demand
(D) marginal return
(E) Nash equilibrium

12. How are a monopoly and a competitive market similar?


(A) Companies can only make a profit in the inelastic end of the
demand curve in both of these market structures.
(B) The quantity provided by companies is at a level where price
equals marginal costs in both market structures.
(C) Both of these market structures have highly limited barriers to
entry.
(D) Both of these market structures are defined by perfect price
discrimination.
(E) These are the only market structures where the law of diminishing
returns can be applied.

AP Microeconomics_p001-224.indd 10 22/12/20 5:00 PM


Getting Started: The Diagnostic Quiz  ❮ 11

13. The following table lists the quantity of cars presented and the
price of cars in five different market types. Which of the following
would MOST likely be the price and quantity if the market were a
monopoly?

Quantity Produced Price per Car


2,000 $190,000 Market A
700 $130,000 Market B
550 $175,000 Market C
100 $200,000 Market D
1,000 $100,000 Market E

(A) It would be market A because monopolists tend to produce a


large quality and sell at high prices.
(B) It would be market B because this is a price where the market is
in equilibrium.
(C) It would be market C because here the monopolist would have
lower marginal revenue than costs.
(D) It would be market D because monopolists tend to be
associated with lower quantities of products and high prices.
(E) It would be market E because it is the lowest price; monopolists
can afford to sell at such a price, as they do not have marketing
costs.

14. In country A, there is only one company that produces electricity


because no other companies are interested and the company has
a contract with the government. In country B, a company filed a
patent for a lifesaving medicine. How can these market structures be
compared?
(A) The situation described in A is an example of monopolistic
structure, while the structure in country B is an example of an
oligopoly.
(B) Both are examples of a natural monopoly.
(C) Both are examples of the use of antitrust policies.
(D) The situation described in A is an example of a perfect
competition, while the structure in country B is an example of
a monopoly.
(E) The situation described in A is an example of a monopsony,
while the structure in country B is an example of a monopoly.

AP Microeconomics_p001-224.indd 11 22/12/20 5:00 PM


12 ❯ 500 AP Microeconomics Questions to Know by Test Day

15. Which of the following is accurate concerning market equilibrium


in a perfectly competitive market?
(A) Companies are price setters.
(B) The level of output produced for price maximization is when
marginal costs are larger than marginal revenue.
(C) The level of output for price maximization is where marginal
revenue equals marginal costs.
(D) The price is equal to marginal revenue in an effective market.
(E) Companies have significant barriers to entry.

16. Why are subsidies NOT always economically effective?


(A) because it raises the price of the product, which decreases
economic output
(B) because they decrease the quantity of the product, which shifts
the equilibrium of the product
(C) because this is something taxpayers have to pay, so despite the
lower cost of the product, there can be a net loss in societal
welfare
(D) because it will decrease the supply of the product, and it will
also shift the demand curve to the left
(E) because it will lead to an increase in the price when the demand
curve shifts to the right

17. A local road is in a horrible condition despite the fact that it is


frequently used. The road was financed through local contributions,
but many individuals registered their address in another town in
order to avoid paying for the management and maintenance of the
road. Which of the following concepts does this best reflect?
(A) the law of diminishing returns
(B) collective bargaining
(C) free rider
(D) the law of supply and demand
(E) scarcity

AP Microeconomics_p001-224.indd 12 22/12/20 5:00 PM


Getting Started: The Diagnostic Quiz  ❮ 13

18. The Gini coefficient for Belgium is 0.259. Which of the following is
the best representation of that value?
(A) It means that government finances are stable because the budget
deficit is only 2.59 percent.
(B) It means that the unemployment rate is very high because the
value of the Gini coefficient is close to 0.
(C) It means that the inflation rate is very low because the value of
the Gini coefficient is smaller than 1.
(D) It means that income inequality is not such an extreme problem
in Belgium, because the value is much closer to 0 than to 1.
(E) It means that income equality is an extreme problem in
Belgium because the value is very close to 0.

19. Tina is a teacher with a PhD at a private university. When her


students learn from her and then pass that knowledge along to the
rest of their social circle, what is this an example of ?
(A) free-rider problem
(B) collective bargaining
(C) negative externality
(D) positive externality
(E) marginal return

20. The amount of taxes Jane has had to pay in the past five years
is shown in the following table, along with her annual revenue.
Assume that Jane had the same costs every year. This demonstrates a

Year Revenue Taxes Paid


2015 $25,000 $5,000
2016 $70,000 $7,000
2017 $90,000 $8,000
2018 $140,000 $7,000

(A) progressive tax system


(B) regressive tax system
(C) flat tax system
(D) demand tax system
(E) supply tax system

AP Microeconomics_p001-224.indd 13 22/12/20 5:00 PM


This page intentionally left blank

AP Microeconomics_p001-224.indd 14 22/12/20 5:00 PM


DIAGNOSTIC QUIZ ANSWERS
1. (Unit: Basic Economic Concepts. See Chapter 1 for similar questions.)
ANSWER: (C) The concept of comparative advantages suggests that the country that
has the lower opportunity cost should produce the good that it can produce with a
lower opportunity cost, and the countries should then trade to maximize efficiency.
2. (Unit: Basic Economic Concepts. See Chapter 1 for similar questions.)
ANSWER: (E) The opportunity cost is the cost of a lost opportunity. As James can
earn $18 per hour working at the library, this is the largest value he has missed out on.
3. (Unit: Basic Economic Concepts. See Chapter 1 for similar questions.)
ANSWER: (D) A quota is a quantitative restriction of the product, while the remaining
answer choices include outlawing the product, tariffs, and administrative restrictions.
4. (Unit: Supply and Demand. See Chapter 2 for similar questions.)
ANSWER: (A) Downward movement from point A to point B indicates a lower supply
with a lower quantity and price. This means that supply has contracted.
5. (Unit: Supply and Demand. See Chapter 2 for similar questions.)
ANSWER: (B) Perfectly elastic demand, which is only a theoretical concept, would be
represented by a horizontal straight line. A flatter slope of the demand curve is associated
with a more elastic demand.
6. (Unit: Production, Cost, and the Perfect Competition Model. See Chapter 3 for
similar questions.)
ANSWER: (B) Including a second worker caused a marginal increase of seven in cars
produced. This is the highest marginal return.
7. (Unit: Production, Cost, and the Perfect Competition Model. See Chapter 3 for
similar questions.)
ANSWER: (E) Accounting profit doesn’t include all of the implicit costs of doing
business, which includes the cost of financial capital.
8. (Unit: Production, Cost, and the Perfect Competition Model. See Chapter 3 for
similar questions.)
ANSWER: (D) The most profitable option for the company—when taking into
account the profit it can earn—is if it opens two plants. After that, the marginal cost of
opening a new plant is higher than the marginal return. When the company has two
plants, it can produce 24 planes for which it can earn $100,000 for each, while it will
have to pay $2,000,000 for opening two plants. This means that the company will earn
$2,400,000 - $2,000,000 = $400,000.

❮ 15

AP Microeconomics_p001-224.indd 15 22/12/20 5:00 PM


16 ❯ Diagnostic Quiz Answers

9. (Unit: Production, Cost, and the Perfect Competition Model. See Chapter 3 for
similar questions.)
ANSWER: (B) One of the assumptions of a perfectly competitive market is that all
of the consumers have all the relevant information they need to make a purchase and
that they are rational.
10. (Unit: Production, Cost, and the Perfect Competition Model. See Chapter 3 for
similar questions.)
ANSWER: (B) If Florida loses roughly $50,000,000 every year, placing a tax on alco-
holic drinks would be a way for society to offset economic costs.
11. (Unit: Imperfect Competition. See Chapter 4 for similar questions.)
ANSWER: (E) This is a concept within game theory known as the Nash equilibrium.
12. (Unit: Imperfect Competition. See Chapter 4 for similar questions.)
ANSWER: (B) While monopolists get to extract economic surplus, the general quan-
tity made by companies is determined by where marginal costs equal the price of the
product.
13. (Unit: Imperfect Competition. See Chapter 4 for similar questions.)
ANSWER: (D) Monopolies are usually considered a highly ineffective market struc-
ture as they are characterized by high prices and lower quantities compared with other
market structures.
14. (Unit: Imperfect Competition. See Chapter 4 for similar questions.)
ANSWER: (B) Both of these are examples of natural monopolies due to the fact that
they occurred through regular market conditions, but there is only one company oper-
ating in each market.
15. (Unit: Factor Markets. See Chapter 5 for similar questions.)
ANSWER: (C) Companies are price takers in such a market, and the level of output
produced where companies can earn a profit is where marginal revenue equals marginal
costs.
16. (Unit: Factor Markets. See Chapter 5 for similar questions.)
ANSWER: (C) Despite the fact that subsidies will lower the price of the product, these
subsidies paid by the government are still something financed by taxpayers. For this
reason, it is still theoretically possible that despite the decreasing prices, there is a net
loss in societal welfare.
17. (Unit: Market Failure and the Role of Government. See Chapter 6 for similar
questions.)
ANSWER: (C) The problem of people not paying for a public good is called the free
rider problem or free-rider dilemma.

AP Microeconomics_p001-224.indd 16 22/12/20 5:00 PM


Diagnostic Quiz Answers  ❮ 17

18. (Unit: Market Failure and the Role of Government. See Chapter 6 for similar
questions.)
ANSWER: (D) The Gini coefficient is a measurement of income inequality. The closer
the value is to 1, the higher the level of income inequality.
19. (Unit: Market Failure and the Role of Government. See Chapter 6 for similar
questions.)
ANSWER: (D) This is a positive externality. Tina’s students pay for their education,
and the end result is a positive externality of knowledge.
20. (Unit: Market Failure and the Role of Government. See Chapter 6 for similar
questions.)
ANSWER: (B) This is an example of a regressive tax system. As Jane earned a higher
revenue, she had to pay a smaller level of her revenue in taxes. In 2015, Jane paid
$5,000 in taxes, which when divided by her revenue of $25,000 is equal to 20 percent
of her revenue. For 2016, she paid 10 percent, 8.9 percent in 2018, and just 5 percent
in 2019. As the percentage income of her taxes paid decreased, this is an example of a
regressive tax systems.

AP Microeconomics_p001-224.indd 17 22/12/20 5:00 PM


This page intentionally left blank

AP Microeconomics_p001-224.indd 18 22/12/20 5:00 PM


CHAPTER
1
Basic Economic Concepts
21. Deirdre has one free hour to practice the piano for an upcoming
school concert or work at the library for $7 per hour or babysit her
neighbor’s 12-​year-​old son for $10 per hour. She chooses to practice
the piano. What is the opportunity cost of practicing the piano?
(A) The opportunity cost would be $17 because she chose not to
participate in these activities.
(B) Without knowing the marginal value of practicing the piano,
there is no way of knowing the true opportunity cost.
(C) The opportunity cost would be $20 because she needs to
babysit for two hours to earn a profit.
(D) The opportunity cost would be $10 because it is the wage for
the most profitable alternative.
(E) The opportunity cost would be $3 because it is the monetary
difference between two alternative choices.

22. After dedicating two hours to studying for the AP economics exam,
Seth chooses to spend an additional hour studying. Which of the
following is most likely TRUE?
(A) The marginal benefit of the additional hour is at least as great as
the marginal cost of the additional hour.
(B) The marginal cost of the additional hour is less than the
marginal benefit of the additional hour.
(C) Both the marginal benefit and marginal cost are always equal in
this scenario.
(D) The marginal benefit of the second hour is less than the
marginal benefit of the additional hour.
(E) The marginal cost of the second hour is greater than the
marginal cost of the additional hour.

❮ 19

AP Microeconomics_p001-224.indd 19 22/12/20 5:00 PM


20 ❯ 500 AP Microeconomics Questions to Know by Test Day

23. Which of these choices is NOT a factor of production?


(A) capital, the machinery, factories, and industrial equipment used
to produce goods
(B) labor, the human work hours used during production
(C) entrepreneurship, input by the decision maker who allocates
production factors
(D) land, including natural resources such as oil and timber reserves
as well as land itself
(E) money, the financial capital used to purchase goods and services
used in production

24. The concave shape of the production possibilities curve implies the
notion of
(A) increasing opportunity costs
(B) comparative advantage
(C) marginal analysis
(D) allocation of limited resources with unlimited material wants
(E) MB = MC

25. It is beneficial for two countries to trade only when there is


(A) a mutually beneficial trade agreement
(B) increasing returns to scale
(C) decreasing returns to scale
(D) an absolute advantage in production between the two countries
(E) a comparative advantage in production between the two
countries

26. If a society overallocates its resources, then


(A) consumer spending would increase because of an increase
in demand
(B) marginal benefit would be greater than marginal cost
(C) the opportunity cost of producing one more unit would
increase exponentially
(D) marginal benefit would be less than marginal cost
(E) marginal benefit would equal marginal cost

AP Microeconomics_p001-224.indd 20 22/12/20 5:00 PM


Basic Economic Concepts ❮ 21

27. Richard and Michele have to mow their lawn and walk their dog on
Sunday morning. It takes Richard one hour to either mow the lawn
or walk the dog. Michele can also walk the dog in one hour, but
she needs two hours to mow the lawn. What should Richard and
Michele do?
(A) Michele should walk the dog because she has absolute
advantage in mowing the lawn.
(B) Richard should walk the dog because he has comparative
advantage in mowing the lawn.
(C) Richard should mow the lawn because he has comparative
advantage in mowing the lawn.
(D) Michele should mow the lawn because she has comparative
advantage in mowing the lawn.
(E) Richard should walk the dog because he has comparative
advantage in walking the dog.

28. Joe works as a manager at a fast-​food restaurant. He has enough


supplies to make 500 hot dogs and 400 hamburgers to sell to
customers. If he uses all of his supplies to make hot dogs, he
could make 1,000 hot dogs and no hamburgers. If he uses all
of his supplies to make hamburgers instead, he could make 800
hamburgers and no hot dogs. What is Joe’s opportunity cost if he
uses all of his supplies to make hot dogs?
(A) The opportunity cost is 800 hamburgers because this is the
maximum quantity of hamburgers that Joe could make.
(B) The opportunity cost is 400 hamburgers because this is what
Joe is giving up to make hot dogs.
(C) The opportunity cost is 500 hot dogs because this is the
increase in hot dog production.
(D) The opportunity cost is 1,000 hot dogs because this is the
amount of hot dogs produced.
(E) The prices of hot dogs and hamburgers are necessary to
calculate opportunity cost in this example.

AP Microeconomics_p001-224.indd 21 22/12/20 5:00 PM


22 ❯ 500 AP Microeconomics Questions to Know by Test Day

29. Mineral deposits, human capital, entrepreneurship, and use of


technology and machinery are all examples of
(A) factors of production
(B) superior and inferior goods
(C) elements sometimes needed to move an existing company
overseas
(D) public goods
(E) material wants and needs

30. The law of increasing costs is best defined as


(A) when the price of a good rises, the quantity does not fall
(B) more of a product is produced, the greater its opportunity cost
(C) one obtains more of a good, the marginal utility (the value from
one more unit) will decrease
(D) the economy grows, government spending will increase as well
(E) all costs are opportunity costs

31. A country is said to have a comparative advantage over another


country when
(A) it can produce a good at a lower opportunity cost than another
country
(B) it can produce a good using more resources per unit of output
than another country
(C) there is a higher degree of specialization and division of labor
compared to another country
(D) when comparing each country’s production possibilities
frontiers, one country is operating at maximum efficiency
and output
(E) one country’s production possibilities frontier is shifted further
to the right compared with another country’s production
possibilities frontier

AP Microeconomics_p001-224.indd 22 22/12/20 5:00 PM


Basic Economic Concepts ❮ 23

32. Suppose both Spain and Germany produce soccer balls and
golf balls. Spain can produce soccer balls at a lower opportunity
cost than Germany. Germany can produce golf balls at a lower
opportunity cost than Spain. According to the law of comparative
advantage,
(A) Germany should stop producing soccer balls; Spain should stop
producing golf balls; Spain and Germany should establish a
trade for the product they stopped producing.
(B) Germany should stop producing golf balls; Spain should stop
producing soccer balls.
(C) Germany should increase production of golf balls and decrease
the production of soccer balls.
(D) Spain should increase the production of soccer balls and
decrease the production of golf balls.
(E) none of the above

33. Which of the following would occur if the United States has a
significant surge in immigration?
(A) The production possibilities curve would shift to the left.
(B) The production possibilities curve would shift to the right.
(C) Demand for a significant number of products would fall.
(D) Nash equilibrium would be established.
(E) The production possibilities curve would be closer to 0.

34. Which of the following is an example of a non-rivalrous good?


(A) planes, because there is a limited number of them
(B) gold, because it is highly expensive
(C) cars, because they don’t have a realistic substitute
(D) air, because people consuming it don’t significantly impact
supply
(E) platinum, because there is a highly limited market for it

AP Microeconomics_p001-224.indd 23 22/12/20 5:00 PM


24 ❯ 500 AP Microeconomics Questions to Know by Test Day

35. Your school decides to build a new performing arts center. What is
the opportunity cost of the performing arts center?
(A) the money used in construction of the performing arts center
(B) the cost of building the performing arts center now rather than
waiting until next year
(C) any other good or service that cannot be provided right now
because of resources used for the new performing arts center
(D) cannot be determined without knowing what the next best
option was for using the resources that went to the performing
arts center
(E) none of the above

36. Which of the following is accurate about a resource that has a non-
zero cost to consume it?
(A) It is a scarce good.
(B) It is not a scarce good.
(C) It is a perfectly elastic good.
(D) It is a perfectly inelastic good.
(E) It is in a state of Nash equilibrium.

37. Marginal analysis is best defined as


(A) the additional benefit received from the consumption of the
next unit of a good or service
(B) the additional cost from the consumption of the next unit of a
good or service
(C) analyzing the combination of goods and services that provide
the best benefit to society
(D) making decisions based upon the marginal benefits and
marginal costs of that decision
(E) when businesses use their resources to produce goods and
services for which they have a comparative advantage

AP Microeconomics_p001-224.indd 24 22/12/20 5:00 PM


Basic Economic Concepts ❮ 25

38. An economic system is characterized as emphasizing private property


and competition, and prices inform buyers and sellers how to
allocate their resources. No public sector exists in the country. This
economic system would be known as a
(A) mixed system
(B) market system
(C) socialist system
(D) command system
(E) barter system

39. The number of items that Italy and France can produce in a day is
indicated in the following table. Using the theory of comparative
advantage, what would be the most likely outcome?

Italy France
Olive oil 13 15
Cheese 14 26

(A) Italy should produce both goods, as it has a comparative


advantage.
(B) France should produce both goods, as it has an absolute
advantage in both.
(C) France should produce cheese, while Italy should produce
olive oil.
(D) Italy should produce cheese, while France should produce olive
oil.
(E) Both Italy and France should produce only cheese.

40. Which of the following best reflects the concept of absolute


advantage?
(A) if a country has a smaller public debt compared with another
country
(B) if a country can produce a good or service better compared
with another country
(C) if a country could produce a good or service better than any
other country with the same quantity of resources
(D) if a country’s production possibilities curve is constantly
shifting to the left as the output of the economy grows
(E) if a country has a higher GDP compared with another country

AP Microeconomics_p001-224.indd 25 22/12/20 5:00 PM


26 ❯ 500 AP Microeconomics Questions to Know by Test Day

41. When production takes place at a point on the production


possibilities curve, and MB = MC, production is
(A) productively efficient but not allocatively efficient
(B) allocatively efficient but not productively efficient
(C) productively efficient and allocatively efficient
(D) neither productively efficient nor allocatively efficient
(E) none of the above

42. The basic economic questions being answered by the decisions of


buyers and sellers in the marketplace occur in a
(A) mixed economy
(B) command economy
(C) traditional economy
(D) market economy
(E) barter economy

Refer to the following graph for questions 43 and 44.

E
B

Laptops A D

Radios

43. A production possibilities curve passing through which point on the


graph would indicate a recession?
(A) point A
(B) point B
(C) point C
(D) point D
(E) point E

AP Microeconomics_p001-224.indd 26 22/12/20 5:00 PM


Basic Economic Concepts ❮ 27

44. What would a shift in production from point E on the graph to


point D indicate?
(A) Laptop production declined, indicating a possible recession.
(B) Radio production increased, indicating an expanding economy.
(C) The firm became more efficient at radio production and less
efficient at laptop production.
(D) The firm decided to produce more radios and fewer laptops and
its current production capacity hasn’t changed.
(E) Laptop supply costs are rising.

45. What would cause the production possibilities curve to shift to


the left?
(A) an increase in the availability of raw materials
(B) factory capacity expansion
(C) a decision to produce more of one item and less of another
(D) shorter work hours
(E) less vacation time

46. Based on the information in the following table, which of the


following countries is in a recession?

GDP growth in
percentages Country A Country B Country C Country D
First quarter 2019 0.5% 1% -0.5% 4%
Second quarter 0.5% 2% 0.25% 3%
of 2019
Third quarter 0.5% 3% 0.3% -2%
of 2019
Fourth quarter 0.5% -1% -0.1% -1%
of 2019

(A) country A because its economy has stagnated throughout the year
(B) countries B, C, and D because they had at least one quarter of
decline
(C) country B because it had a sharp decline in the final quarter
(D) countries C and D because they had two different quarters of
economic decline
(E) country D because it had two consecutive quarters of
declining GDP

AP Microeconomics_p001-224.indd 27 22/12/20 5:00 PM


28 ❯ 500 AP Microeconomics Questions to Know by Test Day

47. A country allows buyers and sellers to make their own choices.
Ownership of private property is allowed, but the government owns
companies in key industries and makes production decisions for
these industries. What type of economy does this country have?
(A) a command economy
(B) a traditional economy
(C) a mixed economy
(D) a market economy
(E) a barter economy

48. What is an example of the paradox of value?


(A) gasoline costing more per gallon than crude oil
(B) solar power costing more per watt than coal power
(C) acid rain caused by power plant emissions
(D) rubies costing more than bottled water
(E) a steak costing more than a hamburger

49. What item is considered a durable good?


(A) a box of pasta
(B) a truck
(C) a T-​shirt
(D) a pair of shoes
(E) a cartridge of printer ink

50. Buyers and sellers exchange goods and services directly without
trading them for money in which type of economy?
(A) a mixed economy
(B) a market economy
(C) a barter economy
(D) a command economy
(E) a traditional economy

51. Which situation best illustrates the concept of marginal utility?


(A) A nation trades goods for which it has a comparative advantage.
(B) A state raises taxes on jewelry.
(C) A consumer purchases three cans of soda but does not want to
buy a fourth can.
(D) Gold increases in value compared with the euro.
(E) Taxes increase to fund health care and retirement benefits.

AP Microeconomics_p001-224.indd 28 22/12/20 5:00 PM


Basic Economic Concepts ❮ 29

52. France and Switzerland produce cheese and chocolate. France has a
competitive advantage in cheese production while Switzerland has
a competitive advantage in chocolate production. If France places a
trade embargo on Switzerland, what is likely to happen with cheese
and chocolate production in each country?
(A) France will produce more cheese and Switzerland will produce
more chocolate.
(B) France will produce more chocolate and Switzerland will
produce more cheese.
(C) France will produce less chocolate and Switzerland will produce
less cheese.
(D) France will produce less chocolate and Switzerland will produce
more cheese.
(E) France will produce more cheese and Switzerland will produce
less chocolate.

The following graph shows a nation’s production possibilities curve.


Use this graph for questions 53 and 54.

Barley

Peas

53. What does this curve indicate about the nation producing these
two crops?
(A) The curve is not valid, because a production possibilities curve
must be concave.
(B) The nation’s resources are equally suitable for barley or pea
production.
(C) With each additional unit of barley not produced, the nation
can produce a larger amount of peas.
(D) With each additional unit of peas not produced, the nation can
produce a larger amount of barley.
(E) The nation has increasing opportunity costs.

AP Microeconomics_p001-224.indd 29 22/12/20 5:00 PM


30 ❯ 500 AP Microeconomics Questions to Know by Test Day

54. What would happen if the nation developed specialized resources for
barley and pea production?
(A) The curve would become concave but would not shift left
or right.
(B) The curve would shift to the left and remain flat.
(C) The curve would shift to the right and remain flat.
(D) The curve would become convex.
(E) The curve would shift to the right and become concave.

The following graph shows the production possibilities curve for a


bakery that produces cookies and loaves of bread. Refer to this graph for
questions 55 and 56.

600

500 C

D A
300

Cookies

B
Bread 80 140 160

55. The bakery is currently producing 140 loaves of bread. If it decides


to produce 160 loaves of bread instead, what is the opportunity cost
for the bakery?
(A) 200 cookies
(B) 600 cookies
(C) 500 cookies
(D) 300 cookies
(E) 100 cookies

AP Microeconomics_p001-224.indd 30 22/12/20 5:00 PM


Basic Economic Concepts ❮ 31

56. If the bakery is producing cookies and bread at point A on the


production possibilities curve, and then one of its ovens breaks
down and its production moves to point D, what is the opportunity
cost of the broken oven?
(A) 80 units of bread
(B) 140 units of bread
(C) 60 units of bread
(D) 200 cookies
(E) 300 cookies

This production possibilities curve shows bread and car production


possibilities for a nation. Refer to it for questions 57 and 58.

Cars
Quantity
A B

Bread
Quantity

57. If the nation’s production shifts from point A to point B, this


represents
(A) an economic expansion
(B) a recession
(C) the introduction of a biased technology for car manufacturing
(D) the introduction of a biased technology for bread
manufacturing
(E) a decline in bread production efficiency

58. A shift in production from point A to point B results in


(A) a higher bread quantity
(B) a lower car quantity
(C) a higher car quantity
(D) the same car quantity
(E) both A and D

AP Microeconomics_p001-224.indd 31 22/12/20 5:00 PM


32 ❯ 500 AP Microeconomics Questions to Know by Test Day

59. A nation produces spinach and lettuce. A biased technology is


introduced that improves spinach production. If the quantity of
spinach produced does not change, the
(A) lettuce quantity will increase
(B) lettuce quantity will stay the same
(C) lettuce quantity will fall
(D) production possibilities curve will shift to the left
(E) maximum production quantity for lettuce will increase

60. The law of diminishing marginal utility refers to


(A) the marginal utility from consuming one additional item
will fall
(B) the marginal utility from consuming one additional item will
increase
(C) marginal benefit should equal marginal cost in a market
(D) MC > MB when the market is in disequilibrium
(E) price floors help establish MB > MC

61. Marginal utility refers to


(A) the change in total utility as a result of the consumption of an
additional unit of a good
(B) the change in a firm’s total cost from hiring an additional unit
of labor
(C) the change in saving caused by change in disposable income
(D) the additional cost of production for one more unit of output
(E) the additional cost incurred for the consumption of the next
unit of a good

62. Peter loves French fries, and his friends dared him to eat nothing
but French fries all day long. He greatly enjoyed eating 50 French
fries, but when he consumed another 50 fries, he enjoyed them less
and less because his stomach began to hurt. What does this scenario
represent?
(A) marginal utility
(B) law of diminishing marginal utility
(C) MC = MB
(D) MB > MC
(E) A, B, and D

AP Microeconomics_p001-224.indd 32 22/12/20 5:00 PM


Basic Economic Concepts ❮ 33

63. In the market system, resources are allocated in which ways?


I. Customs and traditions influence which goods and services are
produced.
II. Voluntary exchange influences which goods and services are
produced.
III. Government determines which goods and services are produced.
(A) I only
(B) II only
(C) I and III
(D) I and II
(E) I, II, and III

64. Daigoro Inc. is a Japanese company that operates and is fully owned
by the Japanese government. It produces a special type of soy sauce.
The United States has implemented a 20 percent tariff on its soy
sauce that usually costs $100 per case. Which of the following
would then occur?
(A) Japanese consumers would have to pay $80 for the product,
as this will help finance Diagoro’s operations.
(B) Japanese consumers would have to pay $120 for the product,
as this would make the product more expensive.
(C) Both Japanese and U.S. consumers would have to pay $120 as a
result of the tariff.
(D) The company would have to pay $20 for each of the soy sauce
cases that enters the U.S. market.
(E) The price of the product in the United States would likely be
$120 for U.S. domestic consumers, making the product less
competitive.

65. The substitution effect states that as the price of a good falls,
(A) it becomes more expensive in comparison with other goods
(B) it becomes less expensive in comparison with other goods
(C) consumers gain more buying power
(D) consumers lose buying power
(E) consumer income rises

AP Microeconomics_p001-224.indd 33 22/12/20 5:00 PM


34 ❯ 500 AP Microeconomics Questions to Know by Test Day

66. The income effect states that as the price of a good falls,
(A) it becomes more expensive in comparison with other goods
(B) it becomes less expensive in comparison with other goods
(C) consumers gain more buying power
(D) consumers lose buying power
(E) consumer income rises

67. Quantity demanded for cars would fall but demand for cars would
remain the same, if
(A) the price of gasoline rose
(B) trucks became more popular compared with cars
(C) a rise in auto accidents made consumers worried about
car safety
(D) car manufacturers left the market
(E) bus fare decreased so fewer people drove cars to work

68. Quantity demanded for sugar beets would increase but demand for
sugar beets would remain the same, if
(A) researchers discovered that an artificial sweetener had
health risks
(B) consumers switched to regular soda instead of diet soda
(C) more farmers began growing sugar beets
(D) a trade embargo reduced the supply of cane sugar
(E) sugar beets became a biofuel feedstock

69. The number of buyers in the market and the number of producers
in the market are
(A) determinants of elasticity
(B) determinants of demand
(C) determinants of supply
(D) determined by the government in a market economy
(E) determined by the factors of production

70. Quantity supplied for mushrooms would increase but supply for
mushrooms would remain the same, if
(A) more farms began growing mushrooms
(B) a research study highlighted the health benefits of mushrooms
(C) the price of fertilizer, a production input, declined
(D) the government granted a tax credit to mushroom farmers
(E) a new type of tractor made mushroom farmers more efficient

AP Microeconomics_p001-224.indd 34 22/12/20 5:00 PM


Basic Economic Concepts ❮ 35

71. Quantity supplied for strawberries would decrease but supply for
strawberries would remain the same, if
(A) wages for farm workers rose
(B) strawberry farms experienced a drought
(C) consumer income falls
(D) strawberry farmers expect the strawberry price to rise
(E) researchers develop more efficient strawberry harvesting tools

72. If the price of a television set increases by 10 percent and the


quantity demanded for television sets decreases by 50 percent,
(A) elasticity is 5 and television set demand is price elastic
(B) elasticity is 0.2 and television set demand is price inelastic
(C) elasticity is 50 and television set demand is elastic
(D) elasticity is 10 and television set demand is elastic
(E) elasticity is 0.02 and television set demand is price inelastic

73. A movie star puts on a particular sunscreen at the beach, and that
brand of sunscreen becomes much more popular. This will NOT
increase
(A) the market price of that sunscreen
(B) quantity supplied for that sunscreen
(C) supply for that sunscreen
(D) demand for that sunscreen
(E) quantity demanded for that sunscreen

74. A new diet makes bananas more popular with consumers. A new
type of fertilizer makes banana trees more productive. As a result,
(A) the market price of bananas will increase
(B) the market price of bananas will decrease
(C) equilibrium quantity for bananas will increase
(D) equilibrium quantity for bananas will decrease
(E) equilibrium quantity for bananas will not change

75. If the price of a used car increases by 40 percent and the quantity
demanded for used cars decreases by 10 percent,
(A) elasticity is 0.25 and used car demand is price inelastic
(B) elasticity is 4 and used car demand is price elastic
(C) elasticity is 0.4 and used car demand is price inelastic
(D) elasticity is 2.5 and used car demand is price elastic
(E) used car demand is neither price inelastic nor elastic

AP Microeconomics_p001-224.indd 35 22/12/20 5:00 PM


36 ❯ 500 AP Microeconomics Questions to Know by Test Day

76. Which regulation could result in a deadweight loss?


(A) price supports for agricultural goods
(B) a minimum wage law
(C) a rent control law
(D) a maximum interest rate on payday loans
(E) all of the above

77. When the market equilibrium price and the market equilibrium
quantity are located at the same point,
(A) oversupply exists
(B) undersupply exists
(C) a deadweight loss is not present
(D) an effective price floor is present
(E) an effective price ceiling is present

78. A consumer buys three apples. Which purchase prices would reflect
the law of diminishing marginal utility?
(A) $1 for the first apple, $2 for the second apple, and $3 for the
third apple
(B) $2 for each apple
(C) $3 for the first apple, $2 for the second apple, and $3 for the
third apple
(D) $3 for the first apple, $2 for the second apple, and $1 for the
third apple
(E) $6 for all three apples

79. Why do tariffs cause deadweight costs?


(A) because they are a tax imposed on a foreign company which the
company has to pay for
(B) because they are a tax imposed on a foreign company which
carries over to domestic consumers who have to pay for it
(C) because they are an administrative barrier to entry which
foreign companies have to pay in order to enter a market
(D) because they limit the number of products that can be imported
from a foreign country, which enables domestic producers to set
a higher price
(E) because they limit on how much a company can charge for its
products, limiting its ability to make an economic profit

AP Microeconomics_p001-224.indd 36 22/12/20 5:00 PM


Basic Economic Concepts ❮ 37

80. The United States has decided to enact a tariff of 10 percent on


Chinese oil exports. If the price of oil is $80 per barrel, how much
will a barrel of oil cost domestic producers after including the tariff ?
(A) $72
(B) $80
(C) $86
(D) $88
(E) $90

81. Germany has decided that Chinese solar panels are flooding its
market. The German government decided that this is preventing
domestic producers from developing their own products and that
this may make them overly dependent on Chinese solar panels. They
have decided to limit the import of Chinese solar panels to 10,000
every year. What is this an example of ?
(A) a tariff
(B) a nonregulatory barrier to entry
(C) a quota
(D) a tax on imports
(E) a trade violation

AP Microeconomics_p001-224.indd 37 22/12/20 5:00 PM


38 ❯ 500 AP Microeconomics Questions to Know by Test Day

82. Assume that a consumer was forced to choose between the five
products in the following list and also assume that he is a perfectly
rational consumer. How would he make his decision?

Increase in Marginal
Consumer’s Benefit to
Item Price Quantity Utility Society
Computer $200 1 $150 $100
Apples $50 6 $2 $15
Luxurious $1,000 1 $100 $30
wooden table
Car $2,000 1 $15 -$100
Pizza $45 2 $5 $5

(A) He would choose a computer because it has the highest increase


to his utility.
(B) He would choose apples because they would present the largest
increase in utility.
(C) He would choose the luxurious table because it has the best
ratio of price compared with the benefit it brings to society.
(D) He would choose a car because it is the most expensive option.
(E) He would choose a car because rational consumers make
decisions by buying the least expensive item.

83. Which of the following BEST reflects an example of a sunk cost?


(A) when a company has to hire an additional worker to produce
an extra product
(B) when a company is trying to acquire another and the costs of
the acquisition are not public knowledge
(C) when an accounting company has to pay to settle a lawsuit
it lost
(D) when a public relations company needs to rent an office for
$10,000
(E) when a buyer decides to purchase a luxury car for $150,000

AP Microeconomics_p001-224.indd 38 22/12/20 5:00 PM


Basic Economic Concepts ❮ 39

84. Thompson Inc. has several potential companies which it can


cooperate with. The opportunity cost of not cooperating with
company A is $500, while the opportunity cost for not cooperating
with company B is $2,000. The marginal cost of cooperating with
company C is $2,500, while the marginal revenue would be $2,000.
Finally, the marginal revenue of cooperating with company D is
$3,000 and the marginal cost of cooperating with this company is
$5,000. Which company should Thompson Inc. cooperate with?
(A) company A because it has a low opportunity cost
(B) company B because it had a higher opportunity cost
(C) both company C and D because marginal cost is larger than
revenue
(D) company C because it has the lowest marginal cost
(E) company D because it has the largest marginal revenue

AP Microeconomics_p001-224.indd 39 22/12/20 5:00 PM


This page intentionally left blank

AP Microeconomics_p001-224.indd 40 22/12/20 5:00 PM


CHAPTER
2
Supply and Demand
85. What will most likely result if the price of apples decreases?
(A) The quantity of apples demanded will increase.
(B) The supply of apples will decrease.
(C) The demand for apples will increase.
(D) The quantity of apples supplied will decrease.
(E) none of the above

86. If Michael’s average yearly income increases and it is observed that


his demand for steak has increased, then steak must be considered
(A) an inferior good
(B) a normal good
(C) a determinant of demand
(D) a determinant of supply
(E) a necessity

87. If Peter’s average yearly income increases, and it is observed that his
demand for thrift-​store-​bought shoes decreases, then thrift-​store-​
bought shoes must be considered
(A) a normal good
(B) a shortage good
(C) a necessity
(D) an inferior good
(E) a determinant of demand

❮ 41

AP Microeconomics_p001-224.indd 41 22/12/20 5:00 PM


42 ❯ 500 AP Microeconomics Questions to Know by Test Day

88. Suppose it is necessary for tin to be used in the production of


guitar strings. If the price of tin decreases and all other variables are
constant, what will most likely result?
(A) The demand for silver will increase.
(B) The quantity demanded for guitars will decrease.
(C) The demand for guitars will increase.
(D) The supply of guitars will decrease.
(E) The supply of guitar strings will increase.

89. According to the law of demand,


(A) as the price of a good or service increases, the demand will shift
to the right
(B) as the price of a good or service increases, the demand will shift
to the left
(C) there is an inverse relationship between quantity demanded of a
good or service and the price of that good or service
(D) as prices for a good or service increase, consumers will begin to
use substitute goods
(E) as the price of a good or service increases, the quantity
demanded will increase

90. A hurricane destroys a significant supply of bananas in 2011. As a


result, the price of bananas increases. What prediction may be made
regarding the supply of apples, a substitute good, when its market is
in equilibrium?
(A) The apple price rises, and the apple supply will increase.
(B) The apple quantity supplied will increase.
(C) The apple price falls, and the apple supply will decrease.
(D) The apple price falls, and the apple quantity supplied will
decrease.
(E) Both the apple price and the apple quantity supplied are
undetermined.

91. What is likely to happen if the price of figs increases?


(A) The supply of figs will increase.
(B) The quantity of figs demanded will decrease.
(C) The demand for figs will increase.
(D) The quantity of figs supplied will increase.
(E) none of the above

AP Microeconomics_p001-224.indd 42 22/12/20 5:00 PM


Supply and Demand ❮ 43

92. If the demand for tennis rackets increases, which prediction can be
made regarding tennis balls?
(A) The demand for tennis balls will fall.
(B) The supply of tennis balls will remain the same.
(C) The price of tennis balls will remain the same.
(D) The supply of tennis balls will fall.
(E) The price and the quantity supplied of tennis balls will increase.

93. Which of the following situations will cause the demand curve for
chicken, a normal good, to shift to the left?
(A) Consumer incomes decrease.
(B) Consumer incomes increase.
(C) The price of steak decreases.
(D) There is a decrease in the cost of raising chickens on a farm.
(E) There is a scientific discovery that relates eating chicken to
lower blood pressure.

94. Mr. Harrington produces hot dog buns. Hot dogs and hot dog buns
are complementary goods. He is most likely to sell his hot dog buns
at a higher price if
(A) the price of hot dogs decreases
(B) there is an increase in consumer income
(C) the price of hot dogs increases
(D) a new technology is developed enabling an increase in hot dog
production
(E) a new technology is developed enabling an increase in hot dog
bun production

95. The equilibrium price is established


(A) at the next price above where the demand and supply curves
intersect
(B) when the quantity supplied equals the quantity demanded
(C) at the next price below where the demand and supply curves
intersect
(D) when you take the difference between the two lowest points
plotted on the demand and supply curves
(E) at the price where either the demand or supply curve becomes
horizontal

AP Microeconomics_p001-224.indd 43 22/12/20 5:00 PM


44 ❯ 500 AP Microeconomics Questions to Know by Test Day

Refer to the following demand curve for baseballs to answer question 96.

$5.00 A
Price

B
$3.00

10 20
Quantity

96. If the quantity demanded for baseballs shifts from point B to


point A, this represents
(A) an increase in the price of the good
(B) an increase in consumer income
(C) an increase in the price of a complementary good
(D) an increase in the price of a substitute good
(E) none of the above

97. Doherty Industries is a leading producer of an energy drink. Which


of the following will cause Doherty Industries to offer more of the
product at all possible sale prices?
(A) A competitor lowers the price of the product.
(B) The price of a key ingredient increases.
(C) The price of a key ingredient decreases.
(D) The demand for the energy drink decreases.
(E) The demand is perfectly elastic.

AP Microeconomics_p001-224.indd 44 22/12/20 5:00 PM


Supply and Demand ❮ 45

98. Suppose the demand for a product is inelastic. If a producer wishes


to increase total revenue, it should
(A) decrease prices
(B) decrease the quantity supplied
(C) increase the quantity supplied
(D) hire more workers
(E) raise prices

99. Which is an important factor to make the demand for a good


inelastic?
(A) It is a necessity.
(B) There are many substitutes.
(C) It is a luxury item.
(D) There are many cross-​price substitutes.
(E) A significant portion of consumers’ budgets goes to purchasing
the good.

100. There is a 10 percent rise in the price of bottled water. This creates
a 40 percent change in the quantity demanded. The demand for
bottled water is considered to be
(A) perfectly inelastic
(B) elastic
(C) inelastic
(D) perfectly elastic
(E) none of the above

101. If a 30 percent rise in gas prices creates a 0 percent decrease in the


quantity demanded, the demand is said to be
(A) inelastic
(B) perfectly elastic
(C) elastic
(D) perfectly inelastic
(E) none of the above

AP Microeconomics_p001-224.indd 45 22/12/20 5:00 PM


46 ❯ 500 AP Microeconomics Questions to Know by Test Day

102. Sally works at a clothing store and the manager has cut her weekly
shift from 30 to 20 hours, reducing her income. As a result, her
demand for fresh vegetables decreases and her demand for canned
vegetables increases. In this example, which type of good is a can of
vegetables?
(A) a shortage good
(B) an inferior good
(C) a necessity
(D) a capital good
(E) a normal good

Use the following graph to answer questions 103 and 104.

Supply 2

Supply 1
Price

Demand 1 Demand 2

Quantity

103. All of the following are factors that contributed to Demand 1


shifting to Demand 2 EXCEPT
(A) an increase in consumer income
(B) a decrease in the price of a substitute good
(C) an increase in the number of buyers in a market
(D) consumers expect the price of the good to increase in the future
(E) the product’s popularity with consumers increased

AP Microeconomics_p001-224.indd 46 22/12/20 5:00 PM


Supply and Demand ❮ 47

104. All of the following are factors that contributed to Supply 1 shifting
to Supply 2 EXCEPT
(A) the price of a key ingredient to the product decreased
(B) the number of sellers decreased
(C) sellers expect the price of the good to rise in the future
(D) the government increased a tax on the product
(E) the product is no longer subsidized by the government

105. An effective price ceiling is usually set


(A) above the equilibrium price and quantity
(B) at the intersection of the supply and demand curves
(C) by subtracting the highest price for supply and the lowest point
for demand on their respective curves
(D) below the equilibrium price
(E) none of the above

106. Which factor contributes to price elasticity of supply?


(A) time
(B) consumer surplus
(C) consumer expectations regarding future prices
(D) producer tastes and preferences
(E) the availability of a producer surplus

107. Consumer surplus is measured by


(A) the area above the supply curve but below the price equilibrium
(B) the sum of buyer and seller surplus
(C) the area above the supply curve and above the price equilibrium
(D) the quotient of percentage change in quantity supplied and
percentage change in price
(E) the quotient of percentage change in quantity demanded and
percentage change in price

108. A grape farm uses fertilizer to produce grapes. Which will occur if the
price of fertilizer increases?
(A) The demand for grapes will increase.
(B) The quantity of grapes demanded will increase.
(C) The supply of grapes will decrease.
(D) The quantity of fertilizer demanded will increase.
(E) The supply of grapes will increase.

AP Microeconomics_p001-224.indd 47 22/12/20 5:00 PM


48 ❯ 500 AP Microeconomics Questions to Know by Test Day

109. If the supply curve is a line with an upward slope, an increase in


demand will cause
(A) an increase in prices and a larger quantity sold
(B) an increase in prices and a smaller quantity sold
(C) a decrease in prices and a larger quantity sold
(D) a decrease in prices and a smaller quantity sold
(E) none of the above

110. A price increase in product X resulted in an increase in demand for


product Z. Product Z is most likely a(n)
(A) inferior good
(B) complementary good
(C) substitute good
(D) normal good
(E) factor of production

111. A limit on interest rates charged by a credit card company is an


example of a
(A) price floor
(B) price ceiling
(C) price support
(D) consequence of minimum wage law
(E) negative externality

112. When quantity demanded is greater than quantity supplied, there


is a(n)
(A) negative externality
(B) shortage in the market
(C) surplus in the market
(D) increase in government regulation
(E) decrease in unemployment

113. If the price of a good increases, the most likely result would be
for the
(A) quantity supplied to increase
(B) quantity supplied to decrease
(C) demand to increase
(D) supply to decrease
(E) demand to decrease

AP Microeconomics_p001-224.indd 48 22/12/20 5:00 PM


Supply and Demand ❮ 49

Use the following chart to solve questions 114 and 115.

Price

A
P1

Q1 Quantity

114. What is point A commonly known as?


(A) deadweight loss
(B) market equilibrium
(C) price discrimination
(D) Nash equilibrium
(D) marginal revenue
(E) inflated value

115. Which of the following could shift point A to the right?


(A) if a substitute good were subsidized by the government
(B) if a tariff was implemented on the product by the government
(C) if the price of a complementary good decreased
(D) if the purchasing power of consumers decreased
(E) if the good in question became less attractive to consumers

116. Another name for excess supply is


(A) disequilibrium
(B) equilibrium point
(C) Gini ratio
(D) surplus
(E) shortage

AP Microeconomics_p001-224.indd 49 22/12/20 5:00 PM


50 ❯ 500 AP Microeconomics Questions to Know by Test Day

117. If a market surplus exists, establishing an effective price floor would


(A) reduce a market surplus
(B) increase a market surplus
(C) remove the need for government regulation
(D) decrease demand
(E) increase government regulation

118. The number of substitutes for a good and time are known as
(A) determinants of supply
(B) determinants of demand
(C) determinants of elasticity
(D) factors of production
(E) elements of the Gini ratio

119. A warm winter results in a larger supply of wheat for the year. Corn
is a substitute good for wheat. What happens to wheat and corn
prices?
(A) The corn price stays the same and the wheat price falls.
(B) The corn price falls and the wheat price falls.
(C) The corn price rises and the wheat price falls.
(D) The corn price stays the same and the wheat price rises.
(E) The corn price and the wheat price both increase.

120. Suppose the price of iPhones increases 2 percent and the quantity
demanded for iPhones decreases by 4 percent, then
(A) elasticity is 2 and demand is price elastic
(B) elasticity is 0.5 and demand is price inelastic
(C) elasticity is 8 and demand is price elastic
(D) elasticity is .05 and demand is price inelastic
(E) answer cannot be determined without knowing the length of
time the product is on the market with the increase in price

AP Microeconomics_p001-224.indd 50 22/12/20 5:00 PM


Supply and Demand ❮ 51

121. Demand for hamburgers increases. Hamburgers and hamburger


buns are complementary goods. What happens to the demand curve
for hamburger buns and the price?
(A) The price of hamburger buns does not change and the demand
curve shifts to the left.
(B) The price of hamburger buns does not change and the demand
curve shifts to the right.
(C) The price of hamburger buns increases and the demand curve
remains the same.
(D) The price of hamburger buns increases and the demand curve
shifts to the right.
(E) The price of hamburger buns increases and the demand curve
shifts to the left.

122. Suppose the market is in equilibrium for razor blades where the
marginal benefit equals the marginal cost. If the government
imposes a tax on razor blades, what may result?
(A) deadweight loss
(B) an increase in demand
(C) an increase in supply
(D) a price ceiling
(E) a price floor

123. Deadweight loss refers to


(A) the lost benefit to society caused by the movement away from
the market equilibrium quantity
(B) the gained benefit to suppliers caused by the increase in the
market equilibrium quantity
(C) the incremental benefit or loss when the consumer increases
consumption by one additional unit (marginal utility)
(D) the legal maximum price above which a product cannot be sold
(price ceiling)
(E) the legal minimum price below which a product cannot be sold
(price floor)

AP Microeconomics_p001-224.indd 51 22/12/20 5:00 PM


52 ❯ 500 AP Microeconomics Questions to Know by Test Day

124. If you calculate the price elasticity of a good to be 0, you are correct
to assume
(A) it is a normal good
(B) it is a luxury good
(C) people will buy the good regardless of the price
(D) the good has few substitutes
(E) the good has few complements

125. If there is joint demand for olives and mushrooms, then olives and
mushrooms are
(A) substitute goods
(B) complementary goods
(C) normal goods
(D) inferior goods
(E) luxury goods

The following graph shows supply and demand for artichokes. Refer to it
for questions 126 and 127.

Artichoke
Price

D1 D2 D3

Artichoke Quantity

126. Bell peppers are a substitute for artichokes. If the price of bell
peppers increases, demand for artichokes could shift from
(A) D1 to D2
(B) D2 to D1
(C) D3 to D2
(D) D3 to D1
(E) The demand would not change.

AP Microeconomics_p001-224.indd 52 22/12/20 5:00 PM


Supply and Demand ❮ 53

127. Bell peppers are a substitute for artichokes. Tomatoes are also a
substitute for artichokes. If a 10 percent price drop for bell peppers
would shift artichoke demand from D2 to D1 and a 10 percent
price drop for tomatoes would shift artichoke demand from
D3 to D1, then
(A) tomatoes are a perfect substitute
(B) bell peppers are a perfect substitute
(C) tomatoes are a better substitute
(D) bell peppers are a better substitute
(E) tomatoes and bell peppers are equally substitutable

128. Consumer surplus is the


(A) difference between the price consumers would have been
willing to pay and the price they actually did pay
(B) price consumers would have been willing to pay minus total
variable costs
(C) price consumers would have been willing to pay in addition to
total variable costs
(D) price consumers would have been willing to pay in addition to
total fixed costs
(E) price consumers would have been willing to pay minus total
fixed and variable costs

129. The government establishes a price ceiling on good X above the


equilibrium price. The result would
(A) raise the price of the good
(B) raise the price of the good and decrease the quantity demanded
(C) lower the price of the good and increase the quantity demanded
(D) lower the price of the good
(E) have no effect on the price of the good or quantity demanded

AP Microeconomics_p001-224.indd 53 22/12/20 5:00 PM


54 ❯ 500 AP Microeconomics Questions to Know by Test Day

Refer to the following graph for question 130.

Grape Jelly
Price

S1 S2

Grape Jelly Quantity

130. If peanut butter and grape jelly are complementary goods, and the
supply of grape jelly moves from S1 to S2,
(A) the demand curve for peanut butter will shift to the right
(B) the demand curve for peanut butter will remain the same
(C) the demand curve for peanut butter will shift to the left
(D) quantity demanded will decrease for peanut butter on the same
demand curve
(E) quantity demanded will increase for peanut butter on the same
demand curve

131. All of the following explain the downward slope of the demand
curve EXCEPT
(A) income effect
(B) substitution effect
(C) diminishing marginal utility
(D) complement effect
(E) the relationship between marginal benefit and marginal cost

AP Microeconomics_p001-224.indd 54 22/12/20 5:00 PM


Supply and Demand ❮ 55

132. If an increase in technology in a perfectly competitive firm only


lowers the firm’s costs of production, the effect will be?
Price Quantity Profit
(A) decrease decrease decrease
(B) decrease increase increase
(C) no change decrease increase
(D) no change increase increase
(E) increase increase increase

133. If tacos are a better substitute for burritos than hamburgers, tacos
will have
(A) a higher cross-​price elasticity of demand
(B) a lower cross-​price elasticity of demand
(C) the same cross-​price elasticity of demand
(D) a negative cross-​price elasticity of demand
(E) a zero cross-​price elasticity of demand

134. Measuring allocative efficiency through allocation of resources is best


expressed in which formula?
(A) P = MC
(B) P = AR
(C) MB > MC
(D) MB = MC
(E) P = ATC

135. If corn oil is less substitutable for olive oil, then corn oil will have
(A) a higher cross-​price elasticity of demand
(B) a lower cross-​price elasticity of demand
(C) the same cross-​price elasticity of demand
(D) a negative cross-​price elasticity of demand
(E) an infinite cross-​price elasticity of demand

AP Microeconomics_p001-224.indd 55 22/12/20 5:00 PM


56 ❯ 500 AP Microeconomics Questions to Know by Test Day

136. Price elasticity of demand is an extremely useful tool in economics


because
(A) it indicates the equilibrium price
(B) it predicts the market forces of supply and demand
(C) it shows how consumer behavior is affected by price
(D) it predicts how much firms will produce until the
shutdown point
(E) it indicates a balance between long-​run and short-​run
production costs

137. If the government establishes a price less than the market


equilibrium price, then it is a(n)
(A) effective price floor
(B) effective price ceiling
(C) barrier to entry
(D) barrier to exit
(E) none of the above

138. How do economists know that a good is a viable substitute?


(A) Calculate cross-​elasticity and the result is a positive number.
(B) Calculate cross-​elasticity and the result is a negative number.
(C) The product is price inelastic.
(D) Wait to see if the market forces of supply and demand balance
to equilibrium for the potential substitute.
(E) none of the above

139. The income effect refers to


(A) the change in the quantity demanded because of a change in
price of a relative good
(B) the change in the quantity demanded because of a change in a
consumer’s purchasing power
(C) the change in a consumer’s total utility from the consumption
of a good
(D) economic profits will be zero in the long run for a perfectly
competitive market
(E) collusive pricing tactics that oligopolies use

AP Microeconomics_p001-224.indd 56 22/12/20 5:00 PM


Supply and Demand ❮ 57

140. The price of a golf ball increases from $1 to $1.25. As a result, golf
ball sales fall by 50 percent. Demand for golf balls is
(A) inelastic
(B) perfectly inelastic
(C) elastic
(D) perfectly elastic
(E) neither elastic nor inelastic

141. The federal government increases mass transit subsidies, and as a


result, the price of a train ticket from New York to Boston decreases
by 40 percent. Ridership increases from 10 million to 12 million as
a result. Demand for train tickets is
(A) inelastic
(B) perfectly inelastic
(C) elastic
(D) perfectly elastic
(E) neither elastic nor inelastic

142. Which of the following is the LEAST elastic good?


(A) oil
(B) cars
(C) insulin
(D) candy
(E) computers

143. A handbag designer sells luxury handbags for $2,000. The designer
raises the handbag price to $4,000 and demand for handbags
increases 50 percent. Which type of good is a handbag?
(A) a Giffen good
(B) a Veblen good
(C) an inferior good
(D) a substitute good
(E) a factor of production

AP Microeconomics_p001-224.indd 57 22/12/20 5:00 PM


58 ❯ 500 AP Microeconomics Questions to Know by Test Day

144. A worker at a shoe store buys five pounds of rice, which is less
expensive than other food items, each week. The rice farm increases
the price of rice and the shoe store worker starts buying six pounds
of rice each week. What type of good is the rice?
(A) a Veblen good
(B) a normal good
(C) a substitute good
(D) a Giffen good
(E) a luxury good

145. Firm A and firm B offer high-​speed Internet service. When firm B
increases the price of its Internet service by 20 percent, people stop
purchasing Internet service from firm B. In comparison with firm
B’s Internet service, firm A’s Internet service is a(n)
(A) complementary good
(B) perfect substitute
(C) imperfect substitute
(D) public good
(E) inferior good

146. The price elasticity of supply is shown in the table below. Which of
the following goods is inelastic?

Name of Good Elasticity of Supply


Oil 0.5
Apples 1.1
Oranges 3
Cars 4
Phones 6

(A) oil
(B) oil and apples
(C) apples, oranges, and cars
(D) apples, oranges, cars, and phones
(E) phones

AP Microeconomics_p001-224.indd 58 22/12/20 5:00 PM


Supply and Demand ❮ 59

Use the following graph to answer questions 147 and 148.

Supply 1

Supply 2

Price

Demand 1 Demand 2

Quantity

147. What factor would NOT contribute to a shift from Demand 2 to


Demand 1?
(A) The product becomes less popular.
(B) A substitute product leaves the market.
(C) A recession reduces consumer income.
(D) Researchers discover the product’s health risks.
(E) A substitute product enters the market.

148. What factors would contribute to a shift from Supply 1 to Supply 2?


(A) A key production input becomes less expensive.
(B) The factory becomes more efficient at producing this product.
(C) Production workers’ wages decrease.
(D) The government reduces taxes on the factory.
(E) All of these factors would shift supply from S1 to S2.

AP Microeconomics_p001-224.indd 59 22/12/20 5:00 PM


60 ❯ 500 AP Microeconomics Questions to Know by Test Day

Use the following charts to answer questions 149 and 150.

Price

Supply

Quantity

149. Which of the following is shown in the chart?


(A) unit elastic supply
(B) perfectly elastic supply
(C) perfectly inelastic supply
(D) relatively elastic supply
(E) relatively inelastic supply

150. How will the change in the supply by 10 percent impact the price in
the market conditions shown in the chart?
(A) The price will decrease by 10 percent.
(B) The price will increase by 10 percent.
(C) The price will remain the same.
(D) The price will increase by 20 percent.
(E) The price will decrease by 20 percent.

AP Microeconomics_p001-224.indd 60 22/12/20 5:00 PM


Supply and Demand ❮ 61

Use the following graph of rental prices and rental quantity to answer ques-
tions 151 and 152.

Apartment Supply

B1 A1

Rental C
Price

A2 B2

Apartment Demand

Rental Quantity

151. What moves on the apartment supply and demand curves would
represent the passage of rent control laws that limited the maximum
apartment rental price?
(A) Apartments supplied would move to A2 and apartments
demanded would move to B2.
(B) Apartments supplied would move to A1 and apartments
demanded would move to B1.
(C) Apartments supplied would move to A1 and apartments
demanded would move to B2.
(D) Apartments supplied would move to A2 and apartments
demanded would move to B1.
(E) Apartments supplied would be equal to apartments demanded
at point C.

152. A shortage of rental apartments would result in a deadweight loss.


Which area would represent the value of this loss?
(A) the area between points A2, B2, and C
(B) the area between points B1, C, and A2
(C) the area between points B1, C, and A1
(D) the area between points A1, C, and B2
(E) not enough information available to answer this question

AP Microeconomics_p001-224.indd 61 22/12/20 5:00 PM


62 ❯ 500 AP Microeconomics Questions to Know by Test Day

153. If the demand curve remains constant, and supply decreases,


the result will be
(A) a decrease in prices and a larger quantity sold
(B) an increase in prices and a smaller quantity sold
(C) a decrease in prices and a smaller quantity sold
(D) an increase in prices and a larger quantity sold
(E) none of the above

154. The price of sugar is currently $3 per pound. The government


establishes a price floor at $2 per pound. What happens to sugar
supply?
(A) There will be an oversupply of sugar.
(B) Quantity demanded will equal quantity supplied.
(C) There will be an undersupply of sugar.
(D) The supply curve for sugar will shift to the left.
(E) The supply curve for sugar will shift to the right.

155. Producer surplus is equal to


(A) the area to the left of the demand curve and below the price
equilibrium
(B) the area to the right of the demand curve and below the price
equilibrium
(C) the area to the left of the demand curve and above the price
equilibrium
(D) the area to the right of the demand curve and above the price
equilibrium
(E) the area to the right of the supply curve and to the left of the
demand curve

AP Microeconomics_p001-224.indd 62 22/12/20 5:00 PM


Supply and Demand ❮ 63

Refer to the following graph for questions 156 and 157.

S1
Auto
Price

D1 D2 D3

Auto Quantity

156. If auto demand shifted from D2 to D1 because of consumer


expectations, what could this indicate?
(A) Consumers expect auto prices to fall.
(B) Consumers expect auto prices to rise.
(C) Consumers expect auto prices to stay the same.
(D) Consumers expect an economic expansion.
(E) Consumers expect raw materials costs for autos to increase.

157. Which demand shift would indicate that consumers expect higher
auto prices?
(A) from D2 to D1
(B) from D3 to D2
(C) from D1 to D3
(D) from D3 to D1
(E) none of the above

AP Microeconomics_p001-224.indd 63 22/12/20 5:00 PM


64 ❯ 500 AP Microeconomics Questions to Know by Test Day

158. The price elasticity of demand is shown in the following table. For
which of the products would the change in the quantity demanded
be exactly proportional to the change in the price?

Name of Good Elasticity of Supply


Pizza –2
Olives –1.3
Ketchup 0
Laptops 1
Smartphones 1.4

(A) pizza and olives


(B) ketchup
(C) laptops
(D) smartphones
(E) laptops and smartphones

159. Which of the following BEST represents an example of how a price


floor can be set?
(A) when France decided to retaliate against Chinese tariffs by
limiting the number of Chinese cars that could be imported
(B) when the United States imposed a retaliatory tax against
Chinese solar panels
(C) when the United States imposed that all Canadian goods
needed to pass another additional health inspection
(D) when the United States imposed that the minimum wage
should be $10 per hour
(E) when France imposed a law that stated that you cannot charge
rent unless you provide minimal living conditions

160. When a government establishes an intervention price and the price


moves below this level,
(A) producer surplus decreases
(B) the demand curve shifts to the right
(C) the supply curve shifts to the left
(D) the equilibrium point shifts to the left
(E) consumer surplus increases

AP Microeconomics_p001-224.indd 64 22/12/20 5:00 PM


Supply and Demand ❮ 65

161. If the tomato price increases by 20 percent, and as a result quantity


demanded for lettuce increases by 40 percent, the cross-​price
elasticity of demand for tomatoes and lettuce is
(A) 2
(B) 0.5
(C) –2
(D) 0.5
(E) 0

162. Peanut butter and blackberry jam have a cross-​price demand


elasticity of –2. The price of peanut butter increases by 20 percent.
As a result, quantity demanded for blackberry jam will
(A) fall 20 percent
(B) fall 40 percent
(C) rise 20 percent
(D) rise 40 percent
(E) remain the same

Use the following graph to answer questions 163 and 164.

Supply

Price

Demand

Quantity

163. Which area represents a surplus created by a price floor?


(A) the area below line A and between the supply and demand curves
(B) the area above line B and between the supply and demand curves
(C) the area above line A and to the right of the supply curve
(D) the area below line B and to the right of the demand curve
(E) the area above line A and between the supply and demand curves

AP Microeconomics_p001-224.indd 65 22/12/20 5:00 PM


66 ❯ 500 AP Microeconomics Questions to Know by Test Day

164. Which area represents a shortage caused by a price ceiling?


(A) the area underneath line A and between the supply and
demand curves
(B) the area above line B and between the supply and
demand curves
(C) the area above line A and to the right of the supply curve
(D) the area below line B and to the right of the demand curve
(E) the area below line B and between the supply and
demand curves

165. Quantity demanded for pork is 10. The price of chicken increases
10 percent. If pork and chicken have a cross-​price elasticity of 5, the
new quantity demanded for pork will be
(A) 10
(B) 15
(C) 2
(D) 5
(E) 50

166. The price of bread increases by 50 percent. As a result, quantity


demanded for cheese falls 20 percent. The cross-price elasticity of
demand for bread and cheese is
(A) 0.4
(B) –0.4
(C) 2.5
(D) –2.5
(E) 1

167. Cross-​price elasticity for corn and wheat is 0.75. The corn price
rises 40 percent. As a result, the new quantity demanded for wheat
is 13. What was quantity demanded for wheat before the corn price
increased?
(A) 16
(B) 17.5
(C) 10
(D) 8
(E) 7.5

AP Microeconomics_p001-224.indd 66 22/12/20 5:00 PM


Supply and Demand ❮ 67

168. The price of peaches increases 30 percent. As a result, quantity


demanded for plums increases from 50 to 80. What is the price
elasticity of demand for peaches and plums?
(A) 2
(B) –2
(C) 0.5
(D) –0.5
(E) 1

169. The price of a peach falls from $2 to $1.40. Quantity supplied for
peaches falls from 100 to 85. Price elasticity of supply for peaches is
(A) –0.5
(B) 2
(C) 0.3
(D) 0.15
(E) 0.5

170. A fashion retailer can easily order new inventory and can employ
inexperienced workers in its stores. A motorcycle manufacturer must
wait several months for part shipments and workers need training
to work at the motorcycle plant. Price elasticity of supply for the
fashion retailer and the motorcycle manufacturer, respectively, are
likely to be
(A) high, high
(B) high, low
(C) low, high
(D) low, low
(E) not enough information provided

171. If the price of a good decreases, the most likely result would be
for the
(A) quantity supplied to increase
(B) quantity supplied to decrease
(C) demand to increase
(D) supply to decrease
(E) demand to decrease

AP Microeconomics_p001-224.indd 67 22/12/20 5:00 PM


68 ❯ 500 AP Microeconomics Questions to Know by Test Day

172. If a nation’s income increases by 5 percent and quantity demanded


for sports cars rises by 15 percent, income elasticity of demand for
sports cars is
(A) 3
(B) –3
(C) 0.33
(D) –0.33
(E) 0.5

173. If consumers demand more soda, but health care costs increase as
well, this indicates a
(A) negative externality
(B) price ceiling
(C) price support
(D) price floor
(E) decrease in production costs

174. The number of buyers in a market, consumer tastes, the availability


of substitute goods, and income are known as
(A) factors of supply
(B) determinants of demand
(C) consumer expectations
(D) factors of production
(E) marginal benefits

AP Microeconomics_p001-224.indd 68 22/12/20 5:00 PM


Supply and Demand ❮ 69

Refer to the following graph for question 175.

Apple
Price

$1.50 S1

$0.75

10 20 Apples Supplied

175. What is the price elasticity of supply for apples?


(A) 1
(B) 0.5
(C) 2
(D) 1.5
(E) 7.5

176. German consumers experience a 30 percent income rise. Quantity


demanded for oatmeal falls 15 percent in Germany. Income
elasticity of demand for oatmeal is
(A) 0.3
(B) 0.5
(C) –0.5
(D) 2
(E) –2

177. Income for New York consumers rises 20 percent. Quantity


demanded for heating oil doesn’t change. Heating oil is a
(A) sticky good
(B) inferior good
(C) luxury good
(D) normal good
(E) necessity good

AP Microeconomics_p001-224.indd 69 22/12/20 5:00 PM


70 ❯ 500 AP Microeconomics Questions to Know by Test Day

178. The government imposes a $50 tax on television sets. Price elasticity
of supply for television sets is 0.6 and price elasticity of demand for
television sets is –0.2. As a result of the tax increase, a consumer
who buys a television set will pay an additional
(A) $50
(B) $0
(C) $37.50
(D) $25
(E) $12

179. The government places a tariff on imported cars. If price elasticity of


supply for cars is 0.6, and price elasticity of demand for cars is –0.4,
what percentage of the tariff is paid by the car buyer?
(A) 100 percent
(B) 60 percent
(C) 40 percent
(D) 20 percent
(E) 0 percent

AP Microeconomics_p001-224.indd 70 22/12/20 5:00 PM


Supply and Demand ❮ 71

Use the following chart to solve questions 180 and 181.

Price
S
P2
A

B
P1

Q1 Q2
Quantity

180. What would a change from point B to point A represent?


(A) a shift in the supply curve to the right
(B) an expansion of supply
(C) a shift in the supply curve to the left
(D) a contraction of supply
(E) a change in the Nash equilibrium

181. Which of the following could be a cause of a shift from point B to


point A?
(A) a new technology that changes the field
(B) a lower quantity of the product supplied
(C) a higher quantity of the product supplied
(D) a decrease in the number of sellers
(E) a decrease in taxes

182. The government raises the tax on a truck by $10,000. The price
elasticity of supply for trucks is 1.5 and the price elasticity of
demand is –1.0. Truck manufacturers will absorb costs of
(A) $10,000
(B) $6,000
(C) $4,000
(D) $2,000
(E) $0

AP Microeconomics_p001-224.indd 71 22/12/20 5:00 PM


72 ❯ 500 AP Microeconomics Questions to Know by Test Day

183. The government places a tax on bicycle manufacturers. Bicycle


supply is inelastic and demand is elastic. A possible response by
bicycle manufacturers would be
(A) raise bicycle prices
(B) pass the tax hike to bicycle buyers
(C) reduce wages at the bicycle manufacturing plant
(D) reduce bicycle prices
(E) hire more employees

Use the following chart to solve questions 184 and 185.

Price

S1

A S2
P2

B
P1
C

Q1 Q2 Quantity

184. Which of the following could result in a change on the supply curve
from point A to point B?
(A) new technologies being invented that helped production
(B) an increase in the production costs
(C) government subsidies for that product
(D) higher government taxes
(E) a decrease in the price of the product

185. Which of the following could cause a shift in the supply curve from
Point B to Point C?
(A) a decrease in the price of the product
(B) an increase in the price of the product
(C) the government implementing a new tax
(D) a new technology that aids production
(E) a drought that caused difficult conditions for production

AP Microeconomics_p001-224.indd 72 22/12/20 5:00 PM


Supply and Demand ❮ 73

186. Tyson Generators operates in a market where there are many similar
products that are not fully identical. The buyers are completely
rational and aware of all of the relevant information. Companies can
easily enter or leave the market, and there are no barriers to entry.
Why is this NOT an example of perfect competition?
(A) because the products are identical in a perfectly competitive
market rather than being similar
(B) Because buyers are not fully rational in a perfectly competitive
market, it is assumed they are price takers.
(C) because buyers are not aware of the relevant information in a
perfectly competitive market, only producers are
(D) because companies are not supposed to ever exit the market in a
perfectly competitive market
(E) because a perfectly competitive market has significant barriers
to entry

187. The price of a product in market C is $150. If the price in market


A is $150, the price in market B is $200, the price in market D is
$90, and the price in market E is $600, which of the following is a
perfectly competitive market based on the price?

Price the Price of a Price the


Company Would Fair Market Consumer Would
Like to Set Equilibrium Like to Pay
Market A $150 $25 $35
Market B $200 $75 $30
Market C $325 $150 $60
Market D $400 $250 $90
Market E $600 $300 $220

(A) only market D is perfectly competitive


(B) markets A and B are perfectly competitive
(C) markets A, B, and E are perfectly competitive
(D) only market C is perfectly competitive
(E) only market E is perfectly competitive

AP Microeconomics_p001-224.indd 73 22/12/20 5:00 PM


This page intentionally left blank

AP Microeconomics_p001-224.indd 74 22/12/20 5:00 PM


CHAPTER
3
Production, Cost, and the
Perfect Competition Model
Use the following graph to answer question 188.

SRAC3
LRAC
SRAC1
$/Unit

SRAC2

A B C
Quantity

188. Sections A, B, and C of the short-​run and long-​run average cost


graph show, respectively,
(A) economies of scale, constant returns to scale, and diseconomies
of scale
(B) economies of scale, diseconomies of scale, and constant returns
to scale
(C) diseconomies of scale, constant returns to scale, and economies
of scale
(D) diseconomies of scale, economies of scale, and constant returns
to scale
(E) constant returns to scale, economies of scale, and diseconomies
of scale

❮ 75

AP Microeconomics_p001-224.indd 75 22/12/20 5:00 PM


76 ❯ 500 AP Microeconomics Questions to Know by Test Day

189. A farmer produces soybeans. Which of the following factors is


considered by accounting profit?
(A) the wages the farmer has to pay
(B) the cost of overhead
(C) the cost of fertilizer necessary to produce the soybeans
(D) the cost of the farmer’s gas when he goes to sell his products
(E) All of the answer choices are correct.

190. The following table shows how many chairs a company can produce
from hiring additional workers. Which worker is responsible for the
largest marginal return?

Number of Workers Number of Chairs Produced


0 0
1 2
2 10
3 21
4 23
5 27

(A) the first worker


(B) the second worker
(C) the third worker
(D) the fourth worker
(E) the fifth worker

AP Microeconomics_p001-224.indd 76 22/12/20 5:00 PM


Production, Cost, and the Perfect Competition Model ❮ 77

191. Details about the company’s finances are presented in the following
table. What is the company’s accounting profit?

Indicator Value
Revenue $30,000
Wages $5,000
Cost of raw materials $2,000
Overhead $4,000
Cost of accepting one project rather than another $1,500
Transportation costs $1,000

(A) $30,000
(B) $25,000
(C) $21,000
(D) $18,000
(E) $16,500

192. A company has to spend $10,000 on rent every month in order to


produce cars. The company also spends $5,000 to produce each
additional car. Which is the variable cost of producing five cars?
(A) $5,000
(B) $10,000
(C) $15,000
(D) $25,000
(E) $35,000

193. If marginal cost falls in the beginning and then gradually rises as
output increases, you would know that
(A) the law of diminishing returns is valid
(B) MC = MB
(C) this market structure is a perfect competition
(D) this market structure is a monopolistic competition
(E) you would have constant returns to scale

AP Microeconomics_p001-224.indd 77 22/12/20 5:00 PM


78 ❯ 500 AP Microeconomics Questions to Know by Test Day

194. All of the following are variable production inputs EXCEPT


(A) the amount of electricity used in a plant
(B) the total amount of raw materials utilized in a plant
(C) human capital
(D) the size of a firm’s labor force
(E) the water bill for a plant

195. If the price of a variable resource increases, the result would be


(A) a downward shift of MC
(B) an upward shift in AFC
(C) an upward shift in MC
(D) a downward shift in ATC
(E) an upward shift in MP

196. The difference between total revenue and total explicit and implicit
costs is known as
(A) economic profit
(B) accounting profit
(C) total fixed costs
(D) total variable costs
(E) total costs

197. As a result of increased growth, a firm may experience difficulty in


managing larger plants and may lose efficiency. This is known as
(A) constant returns to scale
(B) explicit costs
(C) law of diminishing marginal returns
(D) diseconomies of scale
(E) economies of scale

198. If the long-​run average cost curve is constant over a variety of plant
sizes, this is known as
(A) diseconomies of scale
(B) economies of scale
(C) explicit costs
(D) law of diminishing marginal returns
(E) constant returns to scale

AP Microeconomics_p001-224.indd 78 22/12/20 5:00 PM


Production, Cost, and the Perfect Competition Model ❮ 79

199. Specialization and lower costs of inputs will often result in


(A) a decrease in economic profits
(B) economies of scale
(C) a decrease in implicit costs
(D) constant returns to scale
(E) diseconomies of scale

200. The information about a company’s financial indicators are provided


in the following table. Which of the following is essential for the
company in making decisions about producing the adequate level of
output to maximize profit and whether to continue operating?

Indicator Value
Revenue $78,000
Debt-to-equity ratio 0.34
Total costs $50,000
Variable costs $22,000
Marginal cost $5
Marginal revenue $8

(A) By comparing marginal revenue with marginal cost, the


company should continue operating, as the difference is $3.
(B) By comparing total revenue and total costs, the company
should continue operating, as the difference is $28,000.
(C) By viewing the debt-to-equity ratio, the company should
continue operating, as it is below 1.
(D) By comparing variable and total costs, the company should stop
operating, as its variable costs are too high when compared with
total costs.
(E) By comparing variable costs with total revenue, the company
should stop operating, as it barely has enough profit to cover
total and variable costs.

AP Microeconomics_p001-224.indd 79 22/12/20 5:00 PM


80 ❯ 500 AP Microeconomics Questions to Know by Test Day

201. Mr. Ray is a business owner of a competitive firm. He is trying


to decide whether to shut down his firm in the short run. Which
factors should Mr. Ray analyze to help in his decision?
(A) average variable cost and marginal revenue
(B) total revenue and total cost
(C) total fixed cost and total revenue
(D) price and average total cost
(E) price and marginal benefit

202. In a perfect competition, if P > ATC, which of the following will


take place in the long run?
(A) Price decreases as more firms exit the market.
(B) Price decreases as more firms enter the market.
(C) Price increases as more firms enter the market.
(D) Profits decrease as price increases.
(E) none of the above

203. If prices fall below AVC, a company should


(A) differentiate products
(B) raise prices
(C) cut TFC
(D) lay off workers
(E) shut down

204. The profit-​maximizing rule states that to maximize profits a firm


should produce where
(A) MR = MC
(B) MR > MC
(C) MR < MC
(D) MB > MC
(E) MB = MC

205. When there is zero incentive for more firms to enter a perfectly
competitive market, it is said that the market has reached
(A) constant returns to scale
(B) economies of scale
(C) diseconomies of scale
(D) long-​run equilibrium
(E) none of the above

AP Microeconomics_p001-224.indd 80 22/12/20 5:00 PM


Production, Cost, and the Perfect Competition Model ❮ 81

206. If new firms enter the market, what is most likely to occur?
(A) Total revenue will decrease.
(B) Total revenue will increase.
(C) Market power increases.
(D) Market power decreases.
(E) none of the above

207. A perfect competition refers to


(A) the most competitive market structure
(B) the least competitive market structure
(C) extensive economies of scale and higher cost-​efficiency when
there is only one firm for the entire demand of a product
(D) a few small firms offering a differentiated product with easy
entry into the market
(E) a market structure with a small number of interdependent large
firms producing a standardized product

208. Kronhorst Inc. has a marginal revenue of $2,000, while its average
variable costs are $1,500. The company has roughly $1,800
depreciation of its office annually. How much would the average
variable costs have to be in order for the company to decide to shut
down operations?
(A) $300
(B) $500
(C) $1,500
(D) $1,800
(E) $2,000

209. What is the most likely reason perfectly competitive firms do not
make a profit in the long run?
(A) barriers of exit from the market
(B) the product being standardized with little differentiation
(C) the arrival of new firms on the market
(D) the firms being “price takers”
(E) all of the above

AP Microeconomics_p001-224.indd 81 22/12/20 5:00 PM


82 ❯ 500 AP Microeconomics Questions to Know by Test Day

210. If firms are exiting a market and price is rising as a result, then
(A) there is a consumer shortage
(B) the market structure is a monopolistic competition
(C) P < ATC in a perfect competition
(D) P > ATC in a perfect competition
(E) P = ATC in a perfect competition

211. The key difference between accounting profit and economic profit is
(A) accounting profit includes the opportunity cost of capital
(B) economic profit includes the opportunity cost of capital
(C) economic profit is a key component of GDP
(D) economic profit is usually higher than accounting profit
(E) accounting profit is always higher than economic profit

Use the following graph to answer questions 212 and 213.

MC

AC
Price

P D = MR = MC
C

0 QE
Quantity

212. The diagram represents which type of market structure?


(A) monopolistic competition
(B) perfect competition
(C) natural monopoly
(D) monopoly
(E) oligopoly

AP Microeconomics_p001-224.indd 82 22/12/20 5:00 PM


Production, Cost, and the Perfect Competition Model ❮ 83

213. At which point is the graph showing profit maximization?


(A) MR = MC
(B) AC = P
(C) MC = AC
(D) the area between MC and AC
(E) There is no point at which it shows profit maximization.

214. Which of the following shows the profit-​maximizing point for all
types of market structures?
(A) P < MC
(B) P = MC
(C) P = ATC
(D) P = MR
(E) MR = MC

215. How is monopolistic competition similar to perfect competition?


(A) It is easy for producers to enter and exit the market.
(B) There is only one producer in both market structures.
(C) No money is spent on advertising in either market structures.
(D) Companies are completely price takers in both scenarios.
(E) It is possible for the price to be more than the company’s break-
even cost in the long run.

216. If economic profit is zero, then


(A) a firm earned a normal profit
(B) a firm did not earn an accounting profit
(C) a firm has reached its shutdown point in the short run
(D) a firm has reached its shutdown point in the long run
(E) a firm must lobby the government for assistance in subsidies

217. The freedom of entry and exit for a perfectly competitive market
guarantees
(A) each firm knows how each other is planning its production
(B) no cheating on a firm’s part to produce an additional unit
of output
(C) economic profits will be zero in the long run
(D) economic profits will be greater than accounting profits
(E) economic profits will exceed marginal revenue

AP Microeconomics_p001-224.indd 83 22/12/20 5:00 PM


84 ❯ 500 AP Microeconomics Questions to Know by Test Day

218. Producing to where MR = MC in a perfectly competitive market


ensures
(A) the government will not intervene with regulations
(B) the ease of entry and exit in the market
(C) product differentiation
(D) earning an economic profit in the long run
(E) production efficiency in the long run

219. If a firm bases its decisions of pricing and production on the actions
of other firms, then it is most likely a(n)
(A) monopolistic competition
(B) natural monopoly
(C) monopoly
(D) perfect competition
(E) oligopoly

220. A cartel will maximize profit when


(A) MC = MR
(B) MC < MR
(C) MC > MR
(D) W = MPR
(E) none of the above

221. The significant purpose of a “barrier to entry” in a market is


(A) helping monopolies earn economic profit in the long run
(B) restricting the creation of a cartel
(C) restricting price discrimination
(D) allowing monopolistic competitive firms to differentiate their
product
(E) restricting collusive pricing practices

222. Long-​run profits for the perfectly competitive firm will always be
(A) greater than the marginal cost
(B) normal
(C) negative
(D) positive
(E) none of the above

AP Microeconomics_p001-224.indd 84 22/12/20 5:00 PM


Production, Cost, and the Perfect Competition Model ❮ 85

223. Which of the following BEST reflects the example of an economy of


scale?
(A) when a company receives diminishing returns for new workers
that it employs
(B) when a company is not able to get consumers to buy its
products
(C) when a company becomes more effective in decreasing unit
production costs as it grows
(D) when a larger company takes over a smaller company by
purchasing its shares
(E) when a company receives a larger government subsidy than a
smaller producer

224. Imagine a perfectly competitive market. If the market price of the


product is $15 and the marginal cost is also $15, which of the
following is true?
(A) The firm is making zero profit.
(B) Many firms will soon leave the market.
(C) Many firms will join together to increase profit through
collusive pricing.
(D) The firm is profit maximizing.
(E) all of the above

225. For a perfectly competitive market, the best way to maximize


profit is
(A) MR = MC
(B) MRP < W
(C) AFC = MB
(D) MC > MB
(E) none of the above

226. A tire factory buys rubber to make tires. The price of rubber
declines. As a result,
(A) marginal cost decreases
(B) fixed cost decreases
(C) average total cost increases
(D) average fixed cost decreases
(E) fixed cost increases

AP Microeconomics_p001-224.indd 85 22/12/20 5:00 PM


86 ❯ 500 AP Microeconomics Questions to Know by Test Day

227. Joe resigned from his engineering job, where he earned $50,000,
to open a restaurant. He earns a profit of $40,000 as the restaurant
owner. As the restaurant owner, Joe earns
(A) a normal profit
(B) an economic profit
(C) an accounting profit
(D) an abnormal profit
(E) both B and C

228. Jeff opens a shoe store. He earns $50,000 in the first year, but his
profit falls to $30,000 in the second year after more shoe stores open
up in town. He would earn $30,000 if he worked for another firm
as an employee. In the second year, the shoe store has reached
(A) economic profit
(B) short-​run equilibrium
(C) long-​run equilibrium
(D) market surplus
(E) deadweight loss

229. A furniture company opens up a plant and earns a profit of


$50,000 from the plant. After it opens up a second plant, doubling
production quantity, the first and second plants each earn the
furniture company $60,000 in profit. The furniture company
achieved
(A) diseconomies of scale
(B) economies of scale
(C) diminishing marginal returns
(D) a monopoly
(E) perfect competition

230. An auto manufacturer earns a profit of $100,000 from its plant. It


opens up second and third plants, tripling production quantity, and
profit rises to $300,000. This indicates
(A) economies of scale
(B) diseconomies of scale
(C) constant returns to scale
(D) diminishing marginal returns
(E) monopolistic competition

AP Microeconomics_p001-224.indd 86 22/12/20 5:00 PM


Production, Cost, and the Perfect Competition Model ❮ 87

231. Competition in a market system best helps a society because


(A) supply and demand establish the best price for a good or service
(B) all opportunity costs are heavily analyzed
(C) the total welfare is increased
(D) aggregate supply and aggregate demand are balanced
(E) within the production possibilities frontier, all resources are
maximized and used efficiently

232. An auto parts manufacturer starts buying large quantities of steel,


reducing its variable cost per unit. It also begins to specialize in car
door production instead of manufacturing every part of the car. As
a result, production increases and average cost decreases. The auto
parts manufacturer achieved
(A) economies of scale
(B) diseconomies of scale
(C) short-​run equilibrium
(D) declining returns to scale
(E) constant returns to scale

233. Bill leaves his job as a lawyer to open up a clothing store. He earned
$100,000 as a lawyer and earns $120,000 as owner of the clothing
store. His economic profit is
(A) $120,000
(B) $100,000
(C) $20,000
(D) $0
(E) $220,000

234. Nate opens up a restaurant and earns a profit of $60,000 per year.
He is offered a job as a factory manager paying $80,000 per year.
His economic profit is
(A) –$20,000
(B) $80,000
(C) –$80,000
(D) $0
(E) $140,000

AP Microeconomics_p001-224.indd 87 22/12/20 5:00 PM


88 ❯ 500 AP Microeconomics Questions to Know by Test Day

235. Company A is having trouble integrating a larger number of plants


so that it can be profitable. As the company expanded, its third plant
was very profitable and the company managed to significantly cut
down their per-unit costs. The company fired its lean management
operator, and when it opened its fourth plant, per-unit costs
increased significantly. This trend persisted as the company opened
its fifth plant and per-unit production costs continued to increase.
This is an example of
(A) a Nash equilibrium
(B) the law of supply and demand
(C) diseconomies of scale
(D) the economy of scale
(E) a consistent return to scale

236. If P < ATC in a market with perfect competition,


(A) firms will exit the market and the price will rise
(B) high exit barriers will prevent firms from leaving and the price
will not change
(C) low entry barriers allow new firms to enter the market and the
price will fall
(D) firms will not enter or exit the market because it is at long-​run
equilibrium
(E) none of the above

237. In a small rural town in Brazil, the local government is the only
employer of teachers. Assume that teachers are not able to work
online or start their own governments, and they are only able to
work for schools within their district. This would be an example of
(A) a monopoly
(B) a perfectly competitive market
(C) an oligopoly
(D) a monopsony
(E) a cartel

AP Microeconomics_p001-224.indd 88 22/12/20 5:00 PM


Production, Cost, and the Perfect Competition Model ❮ 89

238. An automaker is willing to sell a car for $30,000. A consumer is


willing to buy a car for $50,000. The equilibrium price for cars in
the market is $40,000. Producer surplus is
(A) $30,000
(B) $20,000
(C) $40,000
(D) $10,000
(E) $50,000

239. A banana farm will sell a banana for $2. A consumer will pay $2.60
for the banana. The market price of a banana is $2.25. Consumer
surplus is
(A) $2.00
(B) $2.25
(C) $2.60
(D) $0.25
(E) $0.35

240. A raspberry farm is selling boxes of raspberries. At the current


output level, a box has a marginal cost of $4. Marginal cost
would equal marginal revenue at $5. At this output level, a box
of raspberries would sell for $6 on the market. The raspberry
farm should
(A) increase raspberry output until marginal cost is $5
(B) increase raspberry output until marginal cost is $6
(C) shut down
(D) keep raspberry output the same
(E) lower raspberry output

241. Teran is a wine manufacturer in California. The company is


experiencing a troubling increase in its marginal costs that may
endanger its operations. What actions could the company take to
BEST decrease marginal costs in the long run?
(A) The company should hire a larger number of employees.
(B) The company should acquire its largest domestic competitor.
(C) The company should focus on training its employees so they
specialize in a certain area.
(D) The company should request a lump-sum subsidy from the
local government.
(E) The company should fire at least a fourth of its employees.

AP Microeconomics_p001-224.indd 89 22/12/20 5:00 PM


90 ❯ 500 AP Microeconomics Questions to Know by Test Day

242. The information in the following table show many video games
Tanaka Co. can produce, given a certain level of financial
investment. What is this an example of ?

Number of Video
Value of Investment Games Produced
$1,000 20
$1,500 30
$2,250 45
$3,375 67.5

(A) the law of diminishing returns


(B) market equilibrium
(C) Nash equilibrium
(D) an economy of scale
(E) a consistent return to scale

243. A wine manufacturing company operates in a state where it has


a consistent return to scale. Given the following information,
the value of X should be replaced with?

Number of Bottles of
Value of Investment Wine Produced
$1,000 10
$1,200 12
$1,440 X

(A) 2
(B) 14
(C) 14.4
(D) 15
(E) 16.2

AP Microeconomics_p001-224.indd 90 22/12/20 5:00 PM


Production, Cost, and the Perfect Competition Model ❮ 91

244. The profit of a monopolist by selling at a monopolist price is


$1,500,000 in market A. Market B is an oligopoly with the same
conditions, aside from the number of companies, where the
companies that operate in a cartel earn $1,100,000. If the same
market operated under the rules of a perfectly competitive market,
called market C, the companies would earn $300,000. What are the
deadweight costs for each market?
(A) The deadweight costs are $1,500,000 for market A, $1,100,000
for market B, and $300,000 for market C.
(B) The deadweight costs are $400,000 for market A, $800,000 for
market B, and 0 for market C.
(C) The deadweight costs are $1,100,000 for both markets A and B,
while the deadweight costs for C are not known, as there is no
valid reference point.
(D) The deadweight costs are $1,200,000 for market A, $800,000
for market B, and 0 for market C.
(E) The deadweight costs for markets A, B, and C are 0, as they are
all operating under different market structures.

245. Which of the following BEST defines the minimum efficient scale?
(A) It is the lowest point on the cost curve where a company can
produce its product at a competitive price.
(B) It is the points on the production curve where a monopolist can
no longer achieve higher-than-expected returns.
(C) It is the points on the cost curve where a producer no longer
gains higher returns by including additional production
capacity or capital.
(D) It is the level where producers can no longer earn any kind of
profit and should exit the market.
(E) It is the level where producers can no longer earn accounting
profit and should find ways to cut down costs.

AP Microeconomics_p001-224.indd 91 22/12/20 5:00 PM


92 ❯ 500 AP Microeconomics Questions to Know by Test Day

246. The levels of output for a company are shown in the following table.
If the company wants to maximize profits and it costs the company
$1,500,000 to open a plant, while the company can sell each of
its products for $20,000, what is the number of plants where the
company maximizes its profit?

Number of Plants Units Produced


0 0
1 100
2 120
3 140
4 160
5 210

(A) one production plant


(B) two production plants
(C) three production plants
(D) four production plants
(E) five production plants

247. What will happen to the marginal cost curve in case of the law of
diminishing returns?
(A) It will be downward sloping.
(B) It will be upward sloping.
(C) It will cause a shift in the marginal cost curve to the right.
(D) It will make the cost curve a horizontal line.
(E) It will make the cost curve a vertical line.

AP Microeconomics_p001-224.indd 92 22/12/20 5:00 PM


Production, Cost, and the Perfect Competition Model ❮ 93

248. The financial information about Torson’s Toys is provided in the


following table. The company earns $20 for each manufactured toy,
while the cost of hiring an additional employee is $800. What is the
least amount of profit that the company can make if it selects the
number of workers that will bring it the LEAST possible profit?

Number of Workers Toys Produced


0 0
1 80
2 110
3 200
4 310
5 440

(A) $1,300
(B) $800
(C) $600
(D) $200
(E) $4,800

249. Mera Inc. operates in a market that can best be described as perfectly
competitive. Which of the following conditions can it expect?
(A) There will be high barriers to entry.
(B) There will be no economic profit in the long run.
(C) The market will not be effective in the long run.
(D) The price in the long run will be determined only by
total costs – marginal revenue.
(E) There is one company that gains control of the market in the
long run.

AP Microeconomics_p001-224.indd 93 22/12/20 5:00 PM


94 ❯ 500 AP Microeconomics Questions to Know by Test Day

250. The following table shows how many tables a small company can
produce. Which of the following should be the value of X in order
for this to BEST illustrate the law of diminishing returns?

Number of Workers Tables Produced


0 0
1 2
2 5
3 8
4 11
5 X

(A) 12
(B) 15
(C) 17
(D) 18
(E) 19

251. The financial data for a small company called Peterson and
Associates is shown in the following table. What is the economic
profit of the company?

Indicator Value
Revenue $45,000
Wages $7,000
Cost of raw materials $4,000
Overhead $8,000
Cost of financial capital $5,000
Cost of missing a deadline to apply for $6,000
a government subsidy
Transportation costs $2,000

(A) $19,000
(B) $30,000
(C) $35,000
(D) $36,000
(E) $45,000

AP Microeconomics_p001-224.indd 94 22/12/20 5:00 PM


Production, Cost, and the Perfect Competition Model ❮ 95

252. Market A is characterized by low costs of entry, where companies


are unable to earn an economic profit in the long term. Market B is
one in which there are a few companies that work together to set the
market price and which causes consumers to purchase products at a
higher cost. How can these two markets be classified?
(A) Both markets A and B are an oligopoly.
(B) Both markets A and B are an example of monopolistic
competition.
(C) Market A is an example of a monopsony, while market B is an
example of a monopolistic competition.
(D) Market A is an example of an oligopoly, while market B is an
example of monopolistic competition.
(E) Market A is an example of perfect competition, while market B
is an example of oligopoly.

253. The following table shows the price and quantity for several different
markets of table manufacturers. Which of the following would
MOST likely be an example of perfect competition?

Quantity Produced Price per Table


1,000 $1,000 Market A
200 $5,000 Market B
550 $8,000 Market C
200 $20,000 Market D
1,200 $500 Market E

(A) Market A
(B) Market B
(C) Market C
(D) Market D
(E) Market E

AP Microeconomics_p001-224.indd 95 22/12/20 5:00 PM


96 ❯ 500 AP Microeconomics Questions to Know by Test Day

254. Mavericks International is considering whether it should remain in


business. Which of the following is accurate in how the company
should make its decision?
(A) It will only consider its accounting profit.
(B) It will consider both its accounting profit and total costs.
(C) It will primarily consider whether it is making an economic
profit or loss.
(D) It will look at the total costs it has, regardless of revenue.
(E) It will consider its variable costs, regardless of revenue.

255. Which of the following is a structure where companies are price


takers?
(A) a monopsony
(B) a cartel
(C) an oligopoly
(D) a perfectly competitive market
(E) a monopolistic market

256. The information for the long-term financial indicators of a company


competing in several different types of market structures is provided
in the following table. Which of the following markets is MOST
likely a perfectly competitive market?

Market A Market B Market C Market D Market E


Economic $2,000 $1,000 $0 $10,000 $45,000
profit
Accounting $7,000 $2,000 $1,500 $15,000 $60,000
profit
Revenue $45,000 $30,00 $30,000 $60,000 $130,000

(A) Market A
(B) Market B
(C) Market C
(D) Market D
(E) Market E

AP Microeconomics_p001-224.indd 96 22/12/20 5:00 PM


Production, Cost, and the Perfect Competition Model ❮ 97

257. Companies A, B, and C are the only companies participating in


a perfectly competitive market. Which of the following actions is
NOT possible in such market conditions?
(A) for companies A and B to produce a better product, causing
company C to decide to leave the market in the long term
(B) for company D to see that there is enough profit to be made in
entering the market and competing with these companies
(C) for companies A, B, and C to work together to keep the price
higher than it would be in fair market conditions
(D) for companies A, B, and C to all eventually leave the market as
several larger companies entered with stronger products
(E) All of the answer choices are possible in a perfectly competitive
market.

258. The financial details for Darian Corporation are shown in the
following table. Given the financial indicators provided, what is the
economic profit of the company?

Indicator Value
Revenue $60,000
Marketing costs $1,000
Wages $8,000
Cost of raw materials $6,000
Overhead $3,500
Cost of financial capital $4,500
Cost of not signing a contract with a new client $2,000
Transportation costs $1,000

(A) $18,000
(B) $34,000
(C) $36,000
(D) $40,500
(E) $41,500

AP Microeconomics_p001-224.indd 97 22/12/20 5:00 PM


98 ❯ 500 AP Microeconomics Questions to Know by Test Day

259. The financial details for Korean Oil Incorporated are shown in the
following table. What is the value of the difference between the
economic profit of the company and the accounting profit of the
company?

Indicator Value
Revenue $80,000
Sales costs $13,000
Wages $12,000
Marketing costs $1,500
Overhead $5,000
Cost of time owner spent having to file tax returns $2,500

(A) Economic profit if $1,500 higher than accounting profit.


(B) Economic profit is $2,500 lower than accounting profit.
(C) The economic profit is equal to accounting profit in this
example.
(D) Economic profit is $3,500 lower than accounting profit.
(E) Economic profit $6,000 lower than accounting profit.

260. Through the use of a lean management structure, Toyota could


cut down per-unit costs by expanding its operations. Toyota
continued to grow; it understood that by including more product
lines, it could further decrease per-unit costs. At one point, there
weren’t sufficient employees to operate the production line, and the
company’s per-unit costs significantly increased as it opened a new
plant. This is an example of
(A) overcrowding and mismanagement of resources that turned an
economy of scale into a decreasing return to scale
(B) overcrowding of manufacturing lines that caused a constant
return to scale to shift to an economy of scale
(C) ineffective use of resources that caused a diseconomy of scale to
be replaced with a decreasing return to scale
(D) bad inventory management that caused an ineffective scale to
become effective
(E) a change in the market equilibrium that was caused by a negative
externality over which the company had no control

AP Microeconomics_p001-224.indd 98 22/12/20 5:00 PM


Production, Cost, and the Perfect Competition Model ❮ 99

Use the following table to solve questions 261 and 262.

Indicator Value
Revenue $80,000
Cost of inventory $13,000
Wages $20,000
Cost of interns and other part-time employees $4,000
Marketing cost X
Cost of food $12,000
Transportation costs $3,000
Sales costs $2,500
Overhead $3,500
Cost of not paying owner a salary $1,000

261. Jim has recently opened a restaurant. The price of his marketing
efforts is one-third of all of his labor costs. What is the value of
marketing costs labeled as X?
(A) $6,666.67
(B) $7,500
(C) $8,000
(D) $1,333.33
(E) $12,000

262. Using the information from the table and from the previous
question, what are Jim’s accounting profits?
(A) $25,000
(B) $32,000
(C) $33,000
(D) $35,666.67
(E) $43,666.67

AP Microeconomics_p001-224.indd 99 22/12/20 5:00 PM


100 ❯ 500 AP Microeconomics Questions to Know by Test Day

263. Mark’s company produces grapes. While Mark has many


competitors, there is only one large company that manufactures
wines to which Mark can sell his product. Selling grapes directly to
consumers wouldn’t be profitable, so Mark can only sell to the wine
manufacturer. What is this an example of ?
(A) a monopsony
(B) a monopoly
(C) an oligopoly
(D) a cartel
(E) a perfectly competitive market

264. An apple farm is engaged in perfect competition with other apple


farms that sell the same cultivar. To earn an economic profit, the
apple farm should
(A) raise the price of its apples
(B) reduce the price of its apples to gain market share
(C) sell a unique apple cultivar
(D) produce apples until MC = MR
(E) hire more employees

265. In a market for microwave ovens, the equation for quantity supplied is
P = 10 + 4(Q) and the equation for quantity demanded is P = 80 – 3(Q).
What is the equilibrium microwave oven price and quantity?
(A) 80, 420
(B) 50, 90
(C) 80, 90
(D) 50, 10
(E) 80, 130

266. In a market for blenders, the equation for quantity supplied is


P = 80 + 6(Q) and the equation for quantity demanded is 400 – 10(Q).
What is the equilibrium blender price and quantity?
(A) 20, 420
(B) 200, 20
(C) 140, 15
(D) 20, 160
(E) 480, 60

AP Microeconomics_p001-224.indd 100 22/12/20 5:00 PM


Production, Cost, and the Perfect Competition Model ❮ 101

267. Bread becomes less popular with consumers who want wheat-​free
diets. Meanwhile, a flood damages wheat crops, so less wheat is
available for making bread. As a result,
(A) the market price of bread will increase
(B) the market price of bread will decrease
(C) equilibrium quantity for bread will increase
(D) equilibrium quantity for bread will decrease
(E) equilibrium quantity for bread will not change

268. The price of steel, an input for car manufacturing, falls. At the same
time, fewer consumers can afford to buy cars because of a weak
economy. As a result,
(A) the market price of cars will increase
(B) the market price of cars will decrease
(C) equilibrium quantity for cars will increase
(D) equilibrium quantity for cars will decrease
(E) the market price of cars will not change

269. In a market for vacuum cleaners, the equation for quantity supplied
is P = 575 + 8(Q) and the equation for quantity demanded is
920 – 15(Q). What is the equilibrium vacuum cleaner price and
quantity, respectively?
(A) 345, 23
(B) 575, 23
(C) 695, 15
(D) 1095, 10
(E) 1200, 8

270. In a market for refrigerators, the equation for quantity supplied is


1,000 + 3(Q) and the equation for quantity demanded is
5,000 – 5(Q). What is the equilibrium refrigerator price and
quantity, respectively?
(A) 1,150, 50
(B) 1,150, 4,000
(C) 4,000, 8
(D) 4,000, 400
(E) 400, 4,000

AP Microeconomics_p001-224.indd 101 22/12/20 5:00 PM


102 ❯ 500 AP Microeconomics Questions to Know by Test Day

271. The equilibrium price of apples would decrease if the supply and
demand curves for apples shifted, respectively, to the
(A) right, left
(B) left, right
(C) left, left
(D) right, right
(E) none of the above

272. The equilibrium quantity of pears would increase if the supply and
demand curves for pears shifted, respectively, to the
(A) right, left
(B) left, right
(C) left, left
(D) right, right
(E) none of the above

273. If the government established an effective price floor for grapes,


(A) quantity supplied would be more than quantity demanded
(B) quantity demanded would be more than quantity supplied
(C) quantity supplied would be equal to quantity demanded
(D) quantity supplied would be less than quantity demanded
(E) none of the above

274. A computer manufacturer needs $50,000 to pay for its factories.


The company also needs to pay $15,000 for the lease on the
manufacturing equipment it uses. The company needs to spend an
additional $500 on parts and $250 on labor for each new computer
produced. If the company produced 100 new computers in March,
which of the following is accurate?
(A) The company’s fixed costs for March are $50,000, while
variable costs are $15,750.
(B) The company’s fixed costs for March are $65,000, while total
costs are $140,000.
(C) The company’s variable costs are equal to its total costs and are
$140,000.
(D) The company’s fixed costs are $70,000, while the cost of labor is
the only variable cost.
(E) The company’s variable cost is $750, while the fixed costs are
$65,000.

AP Microeconomics_p001-224.indd 102 22/12/20 5:00 PM


Production, Cost, and the Perfect Competition Model ❮ 103

275. Zagreb International operates in a market where it works with


several other companies to set the market price which prevents an
effective market structure. On the other hand, GTI International
operates in a market where many companies of a similar size
produce computer chips. However, there is only one producer of
computers that purchases these computer chips. How can these
markets be compared?
(A) Zagreb operates in a perfect competition, while GTI operates in
a monopoly.
(B) Both Zagreb and GTI operate in an oligopoly, but there is a
larger number of sellers in GTI’s market.
(C) Zagreb operates in a monopolistic competition, while GTI
operates in a perfectly competitive market.
(D) Zagreb operates in a company that is an oligopoly, while GTI
operates in a monopsony.
(E) Zagreb is an example of a monopoly, while GTI operates in a
natural monopoly.

276. Floyd Pharmaceuticals has revenue of $450,000. The total costs of


the company are $200,000. If it takes the company only 0.1 percent
of its total costs to obtain the value of an additional unit of its
signature vaccine sold, what is the company’s marginal revenue and
why?
(A) $250,000, as this is the difference between revenue and total
costs.
(B) $450,000, as this is how much the company earns every year.
(C) $2,000, as this is how much the company could make per unit
if it was more effective in cutting down costs.
(D) $4,500, as this is the price of how much one employee can
produce.
(E) $200, as this is how much the company can earn from an
additional unit of vaccine.

AP Microeconomics_p001-224.indd 103 22/12/20 5:00 PM


This page intentionally left blank

AP Microeconomics_p001-224.indd 104 22/12/20 5:00 PM


CHAPTER
4
Imperfect Competition
277. Product differentiation is an essential part of which type of market
structure?
(A) oligopoly
(B) monopoly
(C) monopolistic competition
(D) natural monopoly
(E) perfect competition

278. A firm with a natural monopoly may be required by law to set prices
either on or relatively close to the costs that the firm incurred to
make the good or service. This idea is known as
(A) the average cost pricing rule
(B) the ability to pay rule
(C) an antitrust law
(D) law of diminishing marginal returns
(E) economies of scale

279. A monopolistic competition refers to


(A) extensive economies of scale and higher cost-​efficiency when
there is only one firm for the entire demand of a product
(B) many small firms offering a differentiated product with easy
entry into the market
(C) a market structure with a small number of interdependent large
firms producing a standardized product
(D) the most competitive market structure
(E) the least competitive market structure

❮ 105

AP Microeconomics_p001-224.indd 105 22/12/20 5:00 PM


106 ❯ 500 AP Microeconomics Questions to Know by Test Day

280. An oligopoly refers to


(A) a single firm offering the product and achieving economies
of scale
(B) the market structure where a firm has the most pricing power
(C) a small number of interdependent firms, high barriers to entry,
and significant pricing power
(D) many small firms offering a differentiated product with easy
entry into the market
(E) the market structure where a firm has the least pricing power

281. All of the following are part of a monopolistic competition market


structure EXCEPT
(A) differentiated products
(B) deadweight loss
(C) economic profits in the short run
(D) economic profits in the long run
(E) many firms

282. A movie theater offers tickets to the general public for $10. Students
can buy tickets for $8. This is an example of
(A) deadweight loss
(B) variable costs
(C) price discrimination
(D) market power
(E) fixed costs

283. A monopoly refers to


(A) the least competitive market structure
(B) the most competitive market structure
(C) a market structure with a small number of interdependent large
firms producing a standardized product
(D) extensive economies of scale and higher cost-​efficiency when
there is only one firm for the entire demand of a product
(E) the most competitive market structure

AP Microeconomics_p001-224.indd 106 22/12/20 5:00 PM


Imperfect Competition ❮ 107

284. Which of the following BEST reflects an example of an oligopoly?


(A) when there is only one company that is free to set the market price
(B) when there are two to three companies competing against each
other, causing a very low price
(C) when a company gets a patent and is legally the only company
that can fulfill a certain service
(D) when there are many small companies competing against one
another, keeping the price high
(E) when there are a few companies who work together to maintain
a higher market price

285. A company recently found a vaccine, and it now has a patent for it
for 10 years. Why is this an example of a natural monopoly?
(A) because the company will allow other companies to produce it
as well immediately, making the monopoly legal
(B) because there is only one company on the market and it
achieved this through non-market means
(C) because the company will be incentivized to provide a very low
price
(D) because there is only one company in the market, but the
company achieved this through regular market means
(E) because the company receives financial incentives from the
government whenever it gets a patent

286. Why is the demand for fruit less price elastic than the demand for
a boat?
(A) Consumption of fruit is greater.
(B) A boat is an inferior good.
(C) There are more suppliers of fruit than boat manufacturers.
(D) Fruit takes up less of a consumer’s budget.
(E) all of the above

287. All of the following are long-​run production decisions EXCEPT


(A) an auto-​repair shop decides to increase the size of its garage
(B) a school decides to hire more teaching assistants because of an
increase in the school population
(C) a firm increases the number of its plants
(D) a school decides to add more classrooms and a new performing
arts center
(E) a firm decides to close 5 percent of its plants

AP Microeconomics_p001-224.indd 107 22/12/20 5:00 PM


108 ❯ 500 AP Microeconomics Questions to Know by Test Day

288. The difference between a monopoly and a monopolistic


competition is
(A) differentiated products
(B) economies of scale
(C) long-​term pricing power
(D) government regulation
(E) the size of the market

289. If a monopoly and a perfect competition have the same costs, the
monopoly will always
(A) charge a lesser price than the perfect competition
(B) produce the same quantity as the perfect competition
(C) charge a higher price than the perfect competition and
produce less
(D) charge a higher price than the perfect competition and
produce more
(E) none of the above

290. No barriers to entry or exit, many firms, and a standardized product


are characteristics of which type of market structure?
(A) monopolistic competition
(B) oligopoly
(C) natural monopoly
(D) monopoly
(E) perfect competition

291. The market for smartphones may be considered a monopolistic


competition rather than a perfect competition because
(A) there is product differentiation
(B) there is no product differentiation
(C) profits in the long run decrease as more firms enter the market
(D) they are price takers
(E) all of the above

AP Microeconomics_p001-224.indd 108 22/12/20 5:00 PM


Imperfect Competition ❮ 109

292. The services of natural gas, water, and electricity brought into the
household are best consigned to which market structure?
(A) monopolistic competition
(B) oligopoly
(C) perfect competition
(D) natural monopoly
(E) monopoly

293. If the price of cigarettes, a normal good, is in equilibrium, which


choice will result in a price increase for cigarettes?
(A) consumer income to decrease
(B) consumer expectations to remain the same
(C) the price of tobacco to increase
(D) the government to decrease regulations
(E) the price of tobacco to decrease

294. Which of the following is a trait of monopolistic competition?


(A) a market where all of the products are the same
(B) a market where there is only one producer
(C) companies in that market do not maintain shared capacity
(D) a market where companies make more than normal profit in
the long run
(E) a market where there are few barriers to entry

295. Which pricing and output choices would a monopolist select?


(A) Output should be MR = MC and P > MC.
(B) Output should be MR = MC and P = MC.
(C) Output should be MR > MC and P > MC.
(D) Output should be MR < MC and P > MC.
(E) Output should be MR = MC and P < TRC.

296. Which market structures have ease of entry and exit in the long run?
(A) perfect competition and monopoly
(B) perfect competition and monopolistic competition
(C) monopolistic competition and oligopoly
(D) oligopoly and natural monopoly
(E) all market structures

AP Microeconomics_p001-224.indd 109 22/12/20 5:00 PM


110 ❯ 500 AP Microeconomics Questions to Know by Test Day

297. The government is needed to step in and regulate an electric natural


monopoly. Which of the following actions will ensure allocative
efficiency?
(A) Regulate the natural monopoly to the point where P < MC.
(B) Regulate the natural monopoly to the point where P > MC.
(C) Regulate the natural monopoly to the point where P = MC.
(D) Regulate the natural monopoly to the point where P > TFC.
(E) none of the above

298. An airline may identify a specific group of people and charge them a
different rate. This is known as
(A) a monopolistic competition
(B) price discrimination
(C) diseconomies of scale
(D) constant returns to scale
(E) illegal by current federal laws

299. Price discrimination might be successful if


(A) the firm can prevent resale to other consumers and identify and
separate groups of consumers
(B) the firm does not have a monopoly on the pricing power of the
good or service
(C) the firm has a monopoly on pricing power but cannot prevent
resale to other consumers
(D) the firm does not have economies of scale
(E) the firm is operating within government regulations

300. Suppose you pick up the latest edition of The Economist and read
that company Z, a producer of cigarettes, recently purchased 8 out
of the 10 biggest farms that produce tobacco. Since you are a very
good AP Economics student, you realize immediately that company
Z is attempting to
(A) increase profits
(B) establish an oligopoly through collusive pricing
(C) maximize profits where marginal revenue equals marginal cost
(D) establish a monopoly through majority control of a factor of
production
(E) none of the above

AP Microeconomics_p001-224.indd 110 22/12/20 5:00 PM


Imperfect Competition ❮ 111

Use the following graph to answer question 301.

MC

B ATC
P
Price

A
D

MR
Q
Quantity

301. The graph represents a short-​run monopolistic equilibrium. The


shaded area represents
(A) surplus
(B) shortage
(C) profit
(D) price ceiling
(E) price floor

302. In the long run, monopolistically competitive firms break even


because of
(A) government regulations
(B) price ceilings
(C) no entry or exit barriers
(D) exit of firms from the market
(E) non-​price competition

AP Microeconomics_p001-224.indd 111 22/12/20 5:00 PM


112 ❯ 500 AP Microeconomics Questions to Know by Test Day

303. Game theory fits best with which market structure?


(A) monopolistic competition
(B) perfect competition
(C) monopoly
(D) oligopoly
(E) natural monopoly

304. Price leadership fits best with which type of market structure?
(A) monopoly
(B) natural monopoly
(C) perfect competition
(D) oligopoly
(E) monopolistic competition

305. Implicit costs are


(A) direct, purchased, out-​of-​pocket costs
(B) costs that change with the level of output
(C) indirect costs or opportunity costs
(D) total variable costs divided by output
(E) none of the above

306. If a cartel comes into existence, the most likely outcome would
be that
(A) economic profits will be balanced among all cartel members
(B) each cartel member would attempt to cheat by producing more
(C) there is allocative efficiency
(D) prices will be established through the market forces of supply
and demand
(E) advertising will push consumers to one firm or the other

307. Monopoly deadweight loss is caused by


(A) P > MC
(B) P = MC
(C) MC = MB
(D) MC > MB
(E) none of the above

AP Microeconomics_p001-224.indd 112 22/12/20 5:00 PM


Imperfect Competition ❮ 113

308. If price equals marginal revenue, which equals marginal cost, which
equals average total cost, all in the long run, which type of market
structure would this be?
(A) monopolistic competition
(B) monopoly
(C) natural monopoly
(D) oligopoly
(E) perfect competition

309. All of the following are characteristic of an oligopoly EXCEPT


(A) price taking
(B) collusive behavior
(C) barriers to entry
(D) cheating on other firm members to produce more
(E) a few large firms

310. Jet Inc. is a car-making company that operates in a market that has
a very inelastic demand curve. Barriers to entering the market are
high, and the price is higher than the marginal cost in the long run.
Which of the following is this most likely an example of ?
(A) monopolistic competition
(B) perfect competition
(C) both A and B
(D) imperfect competition
(E) an atomistic market

AP Microeconomics_p001-224.indd 113 22/12/20 5:00 PM


114 ❯ 500 AP Microeconomics Questions to Know by Test Day

Use the following graph to answer question 311.

100

90

80

70
Total revenue

60

50

40

30

20

10

0 1 2 3 4 5 6 7 8 9 10 11 12
Quantity

311. For the firm in this graph, marginal revenue is


(A) constantly decreasing
(B) constantly increasing
(C) the same at every output level
(D) increasing rapidly, then decreasing, then negative
(E) increasing at a constant rate, then decreasing

312. If P = ATC, then


(A) economic profit is zero
(B) accounting profit is zero
(C) normal profit is unattainable
(D) firms are operating inefficiently
(E) all of the above

AP Microeconomics_p001-224.indd 114 22/12/20 5:00 PM


Imperfect Competition ❮ 115

313. Since the monopolist has many barriers to entry and no


competition, he or she has price-​setting ability. This is also known as
(A) purchasing power
(B) profit maximization
(C) market power
(D) efficiency
(E) none of the above

314. Firms engaged in a monopolistically competitive market have some


market power because
(A) the firms have few competitors
(B) the firms have differentiated products
(C) the government restricts the use of patents for the firms through
antitrust legislation
(D) advertising allows the firms to set any price they wish
(E) none of the above

315. When the government grants a patent to a firm, the market for the
patented product becomes a(n)
(A) monopoly
(B) oligopoly
(C) cartel
(D) monopolistic competition
(E) pure competition

316. Economies of scale refers to


(A) an increase in ATC as quantity increases
(B) a decrease in ATC as quantity increases
(C) a decrease in ATC as quantity decreases
(D) a firm maximizing profit through dominance of the market
(E) sensitivity to the determinants of supply and demand and
price level

AP Microeconomics_p001-224.indd 115 22/12/20 5:00 PM


116 ❯ 500 AP Microeconomics Questions to Know by Test Day

Refer to the following graph, which reflects the market for a soda
manufacturer, to answer questions 317–319.

Soda
Price

$5 A

MC
B
$2

MR D
200 400
Soda Quantity

317. What is the soda manufacturer’s economic profit if it has a


monopoly?
(A) $800
(B) $1,000
(C) $400
(D) $600
(E) $0

318. Which areas on the chart reflect the deadweight loss caused by the
monopoly?
(A) the triangle bounded by points A, B, and (200, $2)
(B) the triangle bounded by points A, B, and (200, $2) AND the
triangle bounded by point B, ($2, 200), and the point where
MR = MC
(C) the rectangle containing point A and (0, $2)
(D) the rectangle containing point A and (0, $0)
(E) the triangle containing the area between D and MR but
above MC

AP Microeconomics_p001-224.indd 116 22/12/20 5:00 PM


Imperfect Competition ❮ 117

319. If the soda price and quantity shifted from point A to point B, this
would indicate
(A) demand for soda increased
(B) economies of scale
(C) other soda manufacturers entered the market
(D) decreased price competition
(E) diseconomies of scale

320. A few laptop manufacturers are competing in a market. The


laptops have slightly different styles and similar prices. The laptop
manufacturers earn economic profits in the long run. The market
is a(n)
(A) oligopoly
(B) monopoly
(C) perfect competition
(D) monopolistic competition
(E) none of the above

321. Several oil companies form an industry organization that sets the
price of a barrel of oil. This is NOT
(A) a monopoly
(B) a cartel
(C) collusion
(D) an oligopoly
(E) market power

322. An agricultural community has four banana farms, five plum farms,
six grape farms, and seven apple farms. Demand is likely the MOST
price elastic for which fruit?
(A) apples
(B) bananas
(C) plums
(D) grapes
(E) not enough information

AP Microeconomics_p001-224.indd 117 22/12/20 5:00 PM


118 ❯ 500 AP Microeconomics Questions to Know by Test Day

323. Many farms are selling peanuts. The peanuts are not differentiated
and the peanut farms are not earning an economic profit. Demand
for a peanut farm is likely
(A) elastic
(B) perfectly elastic
(C) inelastic
(D) perfectly inelastic
(E) downward sloping

324. An auto manufacturer needs to reduce costs in the short run. It can
make any of these decisions EXCEPT
(A) lay off 10 percent of the auto workers
(B) shut down 5 percent of its plants
(C) order smaller quantities of metal and rubber
(D) shut down a production line
(E) refinance corporate bonds

325. A price increase in product B resulted in a price increase for product


C. Product C is most likely a(n)
(A) inferior good
(B) complementary good
(C) normal good
(D) substitute good
(E) factor of production

326. A toy manufacturer adds an employee and average cost decreases.


It continues to add more employees. At first, average cost decreases
even further, but with even more employees average cost begins to
rise. This reflects
(A) economies of scale
(B) diseconomies of scale
(C) constant economies of scale
(D) the U-​shaped average cost curve
(E) new firms entering the market

AP Microeconomics_p001-224.indd 118 22/12/20 5:00 PM


Imperfect Competition ❮ 119

327. A group of truck manufacturers are competing in a market. The


trucks they manufacture are differentiated products. If this market is
an oligopoly and NOT a monopolistic competition,
(A) truck manufacturers must consider other firms’ responses when
making business decisions
(B) the trucks would not be differentiated
(C) there would be many truck manufacturers
(D) truck manufacturers would have no incentive to collude
(E) truck manufacturers would have market power

328. An auto parts manufacturer has fixed costs of $40,000. It can shut
down or produce tires at a loss of $10,000. If it decides to produce
tires, its economic profit is
(A) –$10,000
(B) $40,000
(C) $30,000
(D) $50,000
(E) –$30,000

329. Four cable companies have 90 percent market share. These firms
may be
(A) price takers
(B) in collusion
(C) in perfect competition
(D) unconcerned with other cable companies’ pricing strategies
(E) earning a normal profit in the long run

330. Many small potato farms are growing potatoes. Three large French
fry manufacturers are buying all of the potatoes. The potato market
is best described as
(A) perfect competition
(B) an oligopoly
(C) an oligopsony
(D) a monopsony
(E) monopolistic competition

AP Microeconomics_p001-224.indd 119 22/12/20 5:00 PM


120 ❯ 500 AP Microeconomics Questions to Know by Test Day

331. When a steel manufacturer is buying all of the iron ore produced by
many mines, it is likely
(A) a price taker
(B) a monopsony
(C) not insulated from supply and demand fluctuations
(D) at a negotiating disadvantage to suppliers
(E) engaged in perfect competition

332. A movie studio charges $8 to watch one of its films over the Internet
in the United States, and $12 to watch the film from Australia. This
is an example of
(A) collusion
(B) price discrimination
(C) economies of scale
(D) a cartel
(E) game theory

The following table illustrates choices available to firm A and firm B.


Use this table to answer questions 333–336.
Raise Lower
Price Firm A Price

Raise
Price $10, $10 $25, –$5

Firm B

Lower –$5, $25 $0, $0


Price

333. Using the table, which decision would result in the best outcome for
firm A?
(A) Firm A lowers prices and Firm B raises prices.
(B) Firm B lowers prices and Firm A raises prices.
(C) Firms A and B both raise prices.
(D) Firms A and B both lower prices.
(E) none of the above

AP Microeconomics_p001-224.indd 120 22/12/20 5:00 PM


Imperfect Competition ❮ 121

334. Which outcome would be best for both firms?


(A) Firm A raises prices and Firm B raises prices.
(B) Firm A lowers prices and Firm B raises prices.
(C) Firm A raises prices and Firm B lowers prices.
(D) Firm A lowers prices and Firm B lowers prices.
(E) none of the above

335. In which type of market would a firm use a payoff matrix to set
pricing policy?
(A) a monopoly
(B) perfect competition
(C) monopolistic competition
(D) oligopoly
(E) natural monopoly

336. If Firm A and Firm B are in a prisoner’s dilemma situation, the Nash
equilibrium is
(A) Firm A defects and Firm B defects
(B) Firm A cooperates and Firm B defects
(C) Firm A defects and Firm B cooperates
(D) Firm A cooperates and Firm B cooperates
(E) none of the above

337. A group of oil producers have formed a cartel. The cartel is


restricting quantity supplied to set a higher price for oil. To
maximize its profit, an individual oil producer should
(A) raise the price of its oil
(B) lower the price of its oil
(C) increase quantity supplied for its oil
(D) decrease quantity supplied for its oil
(E) leave the oil price and quantity supplied unchanged

AP Microeconomics_p001-224.indd 121 22/12/20 5:00 PM


122 ❯ 500 AP Microeconomics Questions to Know by Test Day

This table illustrates choices available to Betty’s and Joey’s, two restaurants.
Refer to this table for questions 338 and 339.
Don’t Buy
Buy Joey’s Ads

Don’t
Buy $50, $50 $70, $30

Betty’s

Buy $30, $70 $40, $40


Ads

338. Which is the dominant strategy for Joey’s?


(A) Don’t buy ads, no matter what happens.
(B) Buy ads only if Betty’s buys ads.
(C) Don’t buy ads if Betty’s buys ads.
(D) Buy ads no matter what happens.
(E) none of the above

339. Which choice results in the lowest profit for Betty’s and Joey’s?
(A) Joey’s buys ads and Betty’s buys ads.
(B) Joey’s buys ads and Betty’s does not buy ads.
(C) Joey’s does not buy ads and Betty’s does not buy ads.
(D) Joey’s does not buy ads and Betty’s buys ads.
(E) none of the above

340. An auto parts manufacturer decides to produce car doors worth


$80,000. It could have produced car windows worth $60,000
instead. Its implicit costs were
(A) $20,000
(B) $60,000
(C) $80,000
(D) $0
(E) $140,000

AP Microeconomics_p001-224.indd 122 22/12/20 5:00 PM


Imperfect Competition ❮ 123

341. Several manufacturers of luxury cars work together in order to keep


the price of these products high. By doing so, the price of the car is
far higher had they not worked together. This is an example of
(A) marginal social benefit
(B) a monopoly
(C) a monopsony
(D) perfect competition
(E) an oligopoly

342. Which of the following is considered to be a trait of a monopoly?


(A) no legal barriers to entry
(B) high number of substitute goods
(C) downward-sloping demand
(D) no deliberate actions or attempts to control the market price
(E) high number of competitors

343. How are monopolies and oligopolies similar?


(A) Both market systems are characterized by high barriers to entry.
(B) Both are characterized by markets in which there is no possible
substitute.
(C) There are no selling costs in either market structure.
(D) Both markets are defined by having only one company which
sets the price.
(E) Companies are price takers.

344. Companies A, B, C, and D are the only companies in the electricity


supply market. Why do they have an incentive to form a cartel?
(A) because this allows them to cooperate in market conditions
(B) because this allows them to provide less quality to consumers
(C) because this allows them to increase the price that consumers
have to pay
(D) because it allows the consumers to increase the deadweight cost
of the market
(E) because it allows the consumers to decrease the deadweight cost
of the market

AP Microeconomics_p001-224.indd 123 22/12/20 5:00 PM


124 ❯ 500 AP Microeconomics Questions to Know by Test Day

345. In Market A, the market structure is that of a monopoly. In Market B,


the market structure is that of a perfectly competitive market.
Information about preferences by relevant actors is provided in the
following table. Which of the following would MOST likely be the
price of the product for each of these markets?

Price the Company Price of a Fair Market


Would Like To Set Equilibrium
Market A $200 $100
Market B $250 $50

(A) The price will be $200 for A and $250 for B because companies
are price setters regardless of market structure.
(B) The price will be $150 for both markets, as the price is
determined by the average of what companies require and what
consumers are expected to pay.
(C) The price will be $100 for A and $50 for B because fair market
regulations always determine the price.
(D) The price will be $200 for A and $50 for B because firms
are price setters in monopolies and price takers in a perfectly
competitive market.
(E) The price will be $100 for A and $250 for B because companies
have a higher level of market power in a perfectly competitive
market.

346. Suppose an ice cream producer buys a competing ice cream


producer. This is an example of
(A) vertical integration
(B) horizontal integration
(C) a monopoly
(D) an oligopoly
(E) a corporation

347. Many small farms are producing oranges. There is little


differentiation among the oranges, and farms enter and exit the
orange industry on a regular basis. As a result,
(A) the market is not very competitive
(B) the orange farms have low pricing power
(C) the orange farms have high pricing power
(D) the orange farms have established an oligopoly
(E) the orange farms have large economies of scale

AP Microeconomics_p001-224.indd 124 22/12/20 5:00 PM


Imperfect Competition ❮ 125

348. A tax software firm has 40 percent market share. Its main
competitor has 30 percent market share. If the first firm wants to
gain market share, it should
(A) invest in software patents to create stronger barriers to entry
(B) consider how the other tax software company will respond to its
business decisions
(C) attempt to become a price maker rather than a price taker
(D) reduce prices to attract customers from the other tax software
company
(E) buy out the other tax software company

349. Many clothing stores are competing for customers in a city by


offering unique styles. These clothing stores are likely to
(A) open for business or shut down infrequently
(B) spend much of their revenue on ads
(C) consider other clothing stores’ strategic plans before making
business decisions
(D) be unable to raise prices without losing all of their customers
(E) have detailed information about clothing buyers and other
clothing stores

350. A water company has 100 percent market share in a city. To make
the water market most efficient, the government should
(A) separate the water company into several firms to end its
monopoly
(B) eliminate price regulations on water to encourage competition
(C) grant the water company a natural monopoly
(D) allow any firm to build water pipes to homes
(E) shut down the water company for violating antitrust laws

351. How are monopolies and oligopolies similar?


(A) Companies are price takers in both market structures.
(B) Companies have perfectly differentiated products in both
market structures.
(C) Both are examples of ineffective market structures.
(D) Both are characterized by a perfectly inelastic demand curve.
(E) Both earn no economic profit in the long run.

AP Microeconomics_p001-224.indd 125 22/12/20 5:00 PM


126 ❯ 500 AP Microeconomics Questions to Know by Test Day

352. A small town has numerous small restaurants that have almost
identical menus. When surveyed about their satisfaction with the
culinary options in their town, many of the residents primarily
emphasized that they cannot stand so many different ads on their
TVs and radios. The restaurants do not work together to keep high
prices. What is this MOST likely an example of ?
(A) a monopoly
(B) a perfectly competitive market
(C) an oligopoly
(D) a monopolistic market
(E) a monopsony

353. How does an oligopoly compare to monopolistic competition?


(A) Companies are price takers in both, but only in oligopolies
can a change in the price by one company potentially cause a
price war.
(B) Companies are price setters in both, and in case one of the
companies changes the price, a price war can start in both
market structures.
(C) Companies are price takers in a monopolistic competition,
while they are price setters in an oligopoly. In case one company
changes the price, there is no impact in either market structure.
(D) Companies are price setters in a monopolistic competition,
while they are price takers in an oligopoly. In case one of the
companies changes the price in an oligopoly, nothing will
occur, but a price war will start in a monopolistic competition.
(E) Companies are price setters in both market structures. In case
one company changes the price, a price war can start in an
oligopoly, while this is not possible in the case of monopolistic
competition.

354. Which of the following is a trait of a monopolistic competition?


(A) Companies are price takers.
(B) Demand is highly elastic to price changes.
(C) Companies cannot make excess profit in the short run.
(D) There is only one company that operates in the market.
(E) There are numerous options for price differentiation.

AP Microeconomics_p001-224.indd 126 22/12/20 5:00 PM


CHAPTER
5
Factor Markets
355. A golf club manufacturer has a marginal cost of $50 and marginal
revenue of $50 for a golf club. Its average cost for a golf club is $60
and the price at the equilibrium price point is $80. If it sells 100
golf clubs at the equilibrium price point, what is its economic profit?
(A) Its economic profit is $8,000, or equilibrium price × quantity
demanded.
(B) Its economic profit is $5,000, or marginal revenue × quantity
demanded.
(C) Its economic profit is $2,000, or (equilibrium price − average
cost) × quantity demanded.
(D) Its economic profit is $3,000, or (equilibrium price – marginal
cost) × quantity demanded.
(E) Its economic profit is $6,000, or marginal cost × quantity
demanded.

356. A year later, the golf club manufacturer’s average cost is $70 and the
equilibrium price point is also $70. This likely happened because
(A) other firms saw the opportunity for an economic profit and
began manufacturing golf clubs
(B) consumers lost interest in playing golf
(C) equipment in the golf club factory broke down
(D) raw materials for golf club production became less expensive
(E) golf club manufacturers left the market

❮ 127

AP Microeconomics_p001-224.indd 127 22/12/20 5:00 PM


128 ❯ 500 AP Microeconomics Questions to Know by Test Day

357. Two software companies cross-​license their patents to each other,


giving both firms access to patents held by either company. This is
an attempt to create
(A) a monopoly
(B) an economy of scale
(C) a barrier to entry
(D) monopolistic competition
(E) D and C

358. A hydropower plant has a natural monopoly on supplying power to


a city and a pharmaceutical company has a monopoly on a patented
drug. The difference between these monopolies is
(A) the hydro plant has a temporary monopoly
(B) the pharmaceutical company has a temporary monopoly
(C) the pharmaceutical company has been granted its monopoly by
regulators
(D) both firms are violating antitrust laws
(E) the hydro plant has been granted its monopoly by regulators

359. A pharmaceutical firm is currently earning an economic profit on a


patented drug. The patent will expire soon. To continue earning an
economic profit, the pharmaceutical firm should
(A) invent a new drug
(B) raise the price of its existing drug
(C) lower the price of its existing drug
(D) stop manufacturing the drug immediately
(E) keep the price of the drug the same

360. An auto factory produces cars at an average cost of $10,000 per car.
It invests in a larger facility and its average cost rises to $12,000,
reducing its economic profit by $2,000 per car. The auto factory’s
profit decreased because of
(A) lower production efficiency
(B) new market entrants
(C) economies of scale
(D) a decrease in demand for cars
(E) an increase in demand for cars

AP Microeconomics_p001-224.indd 128 22/12/20 5:00 PM


Factor Markets ❮ 129

361. John is starting his own business after years of working in a factory.
In order to maximize his profit, how many workers should he hire?
(A) He should hire until the marginal cost equals marginal revenue.
(B) He should hire until the marginal product equals marginal
factor cost.
(C) He should hire until the marginal cost equals marginal
factor cost.
(D) He should hire until the marginal revenue product equals
marginal factor cost.
(E) He should hire until the marginal revenue product equals
marginal cost.

362. A popcorn factory makes popcorn with salt, cooking oil, and corn
kernels. Demand for popcorn decreases. As a result,
(A) demand for salt decreases
(B) demand for cooking oil increases
(C) the popcorn factory will hire more workers
(D) demand for corn kernels increases
(E) the popcorn factory will buy more production equipment

363. The MFC of labor rises in country A and falls in country B. As a


result,
(A) manufacturers will hire workers in country A and country B
(B) manufacturers will lay off workers in country A and country B
(C) manufacturers will hire workers in country A and lay off
workers in country B
(D) manufacturers will lay off workers in country A and hire
workers in country B
(E) employment will remain unchanged in both countries

364. Country X creates a new job training program for its citizens.
Country Y reduces its spending on job training for its citizens.
As a result,
(A) the MRP for labor will fall in country X and country Y
(B) the MRP for labor will rise in country X and fall in country Y
(C) the MRP for labor will rise in country X and country Y
(D) the MRP for labor will rise in country Y and fall in country X
(E) the MRP for labor in country X and country Y will not change

AP Microeconomics_p001-224.indd 129 22/12/20 5:00 PM


130 ❯ 500 AP Microeconomics Questions to Know by Test Day

365. A rancher can buy 10 bags of feed for $8 per bag. Buying 11 bags of
feed will cost $9 per bag. MFC for the eleventh bag of feed is
(A) $8
(B) $9
(C) $10
(D) $11
(E) $19

366. Demand for raw materials is a


(A) direct demand
(B) positive externality
(C) fixed cost
(D) derived demand
(E) negative externality

Refer to the following graph for questions 367 and 368. Wheat is an
input for bread.

S1

Bread
Price

D1 D2 D3

Bread Quantity

367. If demand for bread shifts from D2 to D3, what happens to the
market for wheat?
(A) Demand for wheat rises.
(B) Demand for wheat falls.
(C) Demand for wheat stays the same.
(D) Quantity demanded falls for wheat.
(E) The price of wheat falls.

AP Microeconomics_p001-224.indd 130 22/12/20 5:00 PM


Factor Markets ❮ 131

368. In this example, demand for wheat and bread can be described,
respectively, as
(A) derived demand and direct demand
(B) direct demand and derived demand
(C) both are derived demand
(D) both are direct demand
(E) unrelated to one another

369. If more workers moved to towns where automotive plants were


located, what will happen to the labor supply curve for auto
manufacturers?
(A) The labor supply curve will become horizontal.
(B) The labor supply curve will become vertical.
(C) The labor supply curve will shift to the left.
(D) The labor supply curve will shift to the right.
(E) There would be no change.

370. How would advancements in technology affect a firm’s labor


demand curve?
(A) It would become horizontal.
(B) It would become vertical.
(C) It would shift to the right.
(D) It would shift to the left.
(E) There would be no change.

371. A lawnmower factory sells lawnmowers for $200. With one worker,
it can make three lawnmowers. With two workers, it can make five
lawnmowers. With three workers, it can make six lawnmowers. If
the MFC for labor decreased from $400 to $200, the lawnmower
factory would
(A) hire one worker
(B) hire two workers
(C) leave its workforce unchanged
(D) lay off one worker
(E) lay off two workers

AP Microeconomics_p001-224.indd 131 22/12/20 5:00 PM


132 ❯ 500 AP Microeconomics Questions to Know by Test Day

372. A taco truck has no local competitors. With four workers, it can
make 100 tacos and sell them for $4 each. With five workers, it can
make 150 tacos, but the market price will fall to $3 per taco. The
taco truck will hire a fifth worker if the MFC of labor is equal to
(A) $100
(B) $50
(C) $150
(D) $200
(E) $400

373. A cupcake store has no local competitors. With three workers, it can
make 100 cupcakes and sell them for $2 each. If it hires a fourth
worker, it can make 200 cupcakes and sell them for $1.50 each.
MRP for the fourth worker is
(A) $200
(B) $150
(C) $100
(D) $50
(E) $0

374. If a community college canceled its nursing classes, what will


happen to the labor supply curve for nearby hospitals?
(A) It will become vertical.
(B) It will become horizontal.
(C) It will shift to the left.
(D) It will shift to the right.
(E) It will not change.

375. A food processing company uses onions to make onion rings. If the
price of onions increases and onions and labor are complementary
resources, what would happen to the food processing company’s
labor demand curve?
(A) It will become vertical.
(B) It will become horizontal.
(C) It will shift to the left.
(D) It will shift to the right.
(E) There would be no change.

AP Microeconomics_p001-224.indd 132 22/12/20 5:00 PM


Factor Markets ❮ 133

376. A motorcycle factory uses machinery and labor to produce


motorcycles. If the motorcycle factory purchases more efficient
machinery, its demand for labor will decrease if
(A) machinery and labor are complementary resources
(B) machinery and labor are substitute resources
(C) the price of motorcycles rises
(D) demand for motorcycles rises
(E) the price of labor decreases

377. An airplane factory uses machinery and labor to produce airplanes.


If the airplane factory buys more efficient machinery, its demand for
labor will increase if
(A) the price of airplanes falls
(B) demand for airplanes falls
(C) the price of airplanes increases
(D) machinery and labor are complementary resources
(E) machinery and labor are substitute resources

378. A soda manufacturer produces soda using sugar and carbonated


water. If soda has a high elasticity of demand, the soda
manufacturer’s demand for carbonated water will be
(A) more elastic
(B) less elastic
(C) inelastic
(D) perfectly inelastic
(E) none of the above

379. A restaurant uses wheat tortillas to make tacos. If corn tortillas are
available in the market and can be used as a substitute for wheat
tortillas, the restaurant’s demand for wheat tortillas will be
(A) more elastic
(B) less elastic
(C) inelastic
(D) perfectly inelastic
(E) none of the above

AP Microeconomics_p001-224.indd 133 22/12/20 5:00 PM


134 ❯ 500 AP Microeconomics Questions to Know by Test Day

380. A canned soup company uses 10 types of beans to produce


minestrone soup and one type of bean to produce chili. The
firm’s demand for a single type of bean used in minestrone soup
production, compared with its demand for the chili bean, will be
(A) more elastic
(B) less elastic
(C) just as elastic
(D) inelastic
(E) perfectly inelastic

381. Trucking company A has trucks available for new drivers. Trucking
company B does not have any more trucks for new drivers, although
new workers can still help unload boxes, which has a smaller
impact on productivity than what a new driver would contribute.
In comparison with trucking company B, trucking company A’s
demand for labor will be
(A) more elastic
(B) less elastic
(C) just as elastic
(D) inelastic
(E) perfectly inelastic

382. If firms hire 15 percent fewer production workers when the wage of
production workers increases by 45 percent, the price elasticity of
labor demand is
(A) 0.33
(B) –0.33
(C) 3
(D) –3
(E) 0

383. If wages rise 40 percent, and 20 percent more production workers


enter the market, the price elasticity of labor supply is
(A) 0.5
(B) –0.5
(C) 2
(D) –2
(E) 0

AP Microeconomics_p001-224.indd 134 22/12/20 5:00 PM


Factor Markets ❮ 135

384. If the MFC of labor decreases,


(A) employment increases
(B) unemployment increases
(C) labor demand decreases
(D) labor supply remains unchanged
(E) labor demand remains unchanged

385. Demand for production inputs is a


(A) derived demand
(B) product of positive externalities
(C) product of negative externalities
(D) direct demand
(E) function of the elasticity of demand

386. Consumers demand hamburgers. A restaurant purchases beef to


produce hamburgers. Demand for hamburgers is a(n)
(A) derived demand
(B) direct demand
(C) factor of production
(D) cost of production
(E) indirect demand

387. Consumers demand trucks from a truck manufacturer. Because of


derived demand, the truck manufacturer will
(A) hire production workers
(B) shut down truck manufacturing plants
(C) create differentiated trucks
(D) offer a discount on last year’s truck model
(E) buy radio ads

388. A cookie factory uses wheat and sugar to produce cookies. Wheat
and sugar are complementary resources. If the MFC of wheat
increases,
(A) sugar demand will fall
(B) sugar demand will rise
(C) sugar demand will stay the same
(D) the price of cookies will fall
(E) the cookie factory will increase production

AP Microeconomics_p001-224.indd 135 22/12/20 5:00 PM


136 ❯ 500 AP Microeconomics Questions to Know by Test Day

389. A suit manufacturer has eight employees who can produce 20


suits. The suits sell for $100 each. With nine employees, the firm
can produce 24 suits. Marginal revenue product for the ninth
employee is
(A) $100
(B) $400
(C) $2,000
(D) $2,400
(E) $800

390. A refrigerator manufacturer has five employees who can produce 10


refrigerators. Each refrigerator sells for $500. With six employees,
the firm can produce 12 refrigerators. Marginal revenue product for
the sixth employee is
(A) $500
(B) $5,000
(C) $1,000
(D) $6,000
(E) $2,000

391. A saw manufacturer has eight workers who can produce 30 saws.
With nine workers, it can produce 25 saws. Each saw sells for $10
on the market. Marginal revenue product for the ninth worker is
(A) $300
(B) $250
(C) $50
(D) –$50
(E) $100

392. A golf club factory has five workers. It pays wages of $500 per day
and makes 20 golf clubs, which sell for $50 each. With six workers,
it would pay wages of $600 per day and make 24 golf clubs. The
golf club factory should hire another worker because
(A) marginal product is positive
(B) marginal revenue product is less than marginal factor cost
(C) marginal product is increasing
(D) total revenue would increase
(E) marginal revenue product is positive

AP Microeconomics_p001-224.indd 136 22/12/20 5:00 PM


Factor Markets ❮ 137

393. A waffle truck sells waffles for $2.50 each. With three employees, its
revenue is $250. With four employees, its revenue is $375. Marginal
product for the fourth employee is
(A) 50
(B) 150
(C) 100
(D) 250
(E) 200

394. A table manufacturer uses wood to produce tables. Consumers start


demanding more tables. Because of derived demand,
(A) the supply of wood will increase
(B) the supply of wood will decrease
(C) the price of wood will increase
(D) the price of wood will decrease
(E) none of the above

395. If employees were required to attend training courses, how would


this affect labor productivity?
(A) It would increase.
(B) It would decrease.
(C) It would remain the same.
(D) It would shift the supply curve to the left.
(E) It would have no effect on supply.

396. The financial indicators for a company are in the following table.
Based on the information, which of the following is the marginal
revenue product of labor?

Indicator Value
Marginal revenue $8
Total revenue $7,000
Marginal cost $7
Deadweight cost $400
(A) $50
(B) $56
(C) $500
(D) $3,200
(E) $5,000

AP Microeconomics_p001-224.indd 137 22/12/20 5:00 PM


138 ❯ 500 AP Microeconomics Questions to Know by Test Day

397. The marginal revenue of a company is three times larger than the
company’s marginal cost. If the company’s marginal cost is $5
and the marginal product is $6, what is the value of the marginal
revenue product of labor?
(A) $15
(B) $50
(C) $65
(D) $75
(E) $90

398. Marginal product is


(A) the total value of all goods produced
(B) the value of goods produced by adding one more
production input
(C) the quantity of goods produced by adding one more
production input
(D) the total quantity of goods produced
(E) also known as scrap or waste

399. Which of the following would result in higher demand for soda
factory workers?
(A) The price of sugar used in soda production increases.
(B) Consumers buy more soda.
(C) Consumers begin drinking tea instead of drinking soda.
(D) Manufacturers shut down soda factories.
(E) Energy drinks gain popularity as an alternative to soda.

400. Which example illustrates derived demand?


(A) A new tomato harvester increases tomato production.
(B) Tomato farm workers go on strike and tomato supply falls.
(C) Ketchup demand increases, and tomato demand increases as a
result.
(D) A new pesticide reduces pest damage to the tomato crop.
(E) Scientists discover health benefits of tomatoes.

AP Microeconomics_p001-224.indd 138 22/12/20 5:00 PM


Factor Markets ❮ 139

401. MRP for a hot dog truck’s second, third, and fourth workers is
$150, $140, and $130, respectively. The city requires the hot dog
truck to pay for health benefits for its workers, increasing MFC for a
worker from $130 to $150. The hot dog truck should
(A) hire one more worker
(B) lay off one worker
(C) shut down
(D) lay off two workers
(E) hire two more workers

402. A cabinet manufacturer has seven employees. Each employee earns


a wage of $25,000. To attract an eighth employee, it would have to
offer a wage of $30,000 to the new employee and current employees
would receive a raise to $30,000 as well. MFC for the eighth
employee is
(A) $175,000
(B) $65,000
(C) $40,000
(D) $5,000
(E) $240,000

403. A smoothie shop uses bananas to make smoothies. The price of


bananas increases. As a result,
(A) MFC increases
(B) profit increases
(C) MP increases
(D) labor demand increases
(E) fixed cost increases

404. A truck manufacturer can produce one more truck if it hires another
employee for $40,000. It can also produce one more truck if it
buys $50,000 worth of machinery. A truck sells on the market for
$60,000. The truck manufacturer should
(A) hire employees until the cost of the next employee equals the
cost of buying machinery to produce another truck
(B) buy machinery until the cost of buying machinery to produce
another car equals the cost of hiring another employee
(C) sell machinery and lay off employees
(D) lay off employees and buy machinery
(E) none of the above

AP Microeconomics_p001-224.indd 139 22/12/20 5:00 PM


140 ❯ 500 AP Microeconomics Questions to Know by Test Day

405. Seth finally realized his dream to open a 24-​hour bagel café, which
sells wheat bagels. Which of the following scenarios would increase
Seth’s demand for labor?
(A) The price of wheat increases.
(B) Wheat farmers go on strike.
(C) The price of wheat decreases.
(D) A blight destroys thousands of acres of farmland.
(E) The government establishes a tax on wheat.

406. Derived demand refers to which of the following?


(A) Labor demand for a business is the MRP curve.
(B) Demand for labor comes from the demand for the product
produced.
(C) The combination of labor and capital helps minimize
total costs.
(D) A business hires workers to the point where MRP = MFC.
(E) Certain firms are wage setters.

407. Wages fall in Arizona and rise in Nevada. As a result,


(A) manufacturers will hire workers in Arizona and Nevada
(B) manufacturers will lay off workers in Arizona and Nevada
(C) manufacturers will hire workers in Arizona and lay off workers
in Nevada
(D) manufacturers will lay off workers in Arizona and hire workers
in Nevada
(E) employment will remain unchanged in Arizona and Nevada

408. New York offers free university education for state residents, while
Nebraska increases tuition for state universities. As a result,
(A) the MRP for labor will rise in New York and Nebraska
(B) the MRP for labor will rise in New York and fall in Nebraska
(C) the MRP for labor will fall in New York and Nebraska
(D) the MRP for labor will fall in New York and rise in Nebraska
(E) the MRP for labor in New York and Nebraska will not change

AP Microeconomics_p001-224.indd 140 22/12/20 5:00 PM


Factor Markets ❮ 141

409. Which of the following is an example of derived demand?


(A) People desire food to eat, so farms invest in tractors and hire
workers.
(B) The wages of farmhands increase, so supply decreases and
demand increases.
(C) The wages of farmhands decrease, so supply increases and
demand decreases.
(D) People desire food to eat, so farms raise the price of food.
(E) none of the above

410. All of the following will increase the demand for doctors EXCEPT
(A) the government requires firms to pay for health care for their
employees
(B) all citizens are enrolled in a national health care program
(C) a cure is found for the common cold
(D) flu sweeps across the nation and sickens thousands
(E) venture capitalists invest more money in health care firms

411. If the output effect is greater than the substitution effect, what will
happen to the demand for labor?
(A) It increases.
(B) It decreases.
(C) It increases and then gradually decreases.
(D) It decreases and then gradually increases.
(E) It remains the same.

412. An increase in MRP for labor will result in


(A) a decrease in labor demand
(B) an increase in labor demand
(C) a shift of the labor supply curve to the left
(D) no change in labor demand
(E) none of the above

AP Microeconomics_p001-224.indd 141 22/12/20 5:00 PM


142 ❯ 500 AP Microeconomics Questions to Know by Test Day

413. The marginal revenue product of labor refers to which of the


following?
(A) It measures the cost a business must pay for using one more
unit of a factor of production.
(B) It is the value that the next unit of labor brings to the firm.
(C) The combination of labor and capital helps minimize
total costs.
(D) A business hires workers to the point where MRP = MRC.
(E) Certain firms are wage setters.

414. If the wage of the last worker hired by a firm is greater than MRP of
labor, the firm should
(A) hire more workers
(B) increase supply
(C) decrease supply
(D) lay off workers
(E) do nothing

415. Immigration has which of the following effects?


(A) It will move the labor supply curve from their country of origin
to the right. In the country to which they moved, the curve will
shift to the left.
(B) It will move the labor supply curve from their country of origin
to the right. In the country to which they moved, the curve will
shift to the right.
(C) It will move the labor supply curve from their country of origin
to the left. In the country to which they moved, the curve will
shift to the right.
(D) It will increase revenue for firms.
(E) It will decrease the labor demand curves.

416. If the price of raw materials increases, what will happen to labor
demand?
(A) Labor demand will increase.
(B) Labor demand will remain the same.
(C) Labor demand will decrease.
(D) Labor demand will drive prices lower.
(E) Labor demand will have no effect on price.

AP Microeconomics_p001-224.indd 142 22/12/20 5:00 PM


Factor Markets ❮ 143

417. What will happen to labor demand if the price of a complementary


resource rises?
(A) Labor demand will decrease.
(B) Labor demand will remain the same.
(C) Labor demand will increase.
(D) Labor demand will force the price of the product lower.
(E) Labor demand will have no effect on price.

418. The least-​cost hiring rule refers to


(A) firms that are wage setters
(B) a business hiring workers to the point where MRP = MFC
(C) the combination of labor and capital helping minimize total costs
(D) measuring the cost a business must pay for using one more unit
of a factor of production
(E) the value that the next unit of a resource brings to the firm

419. All else equal, as the MFC of labor increases,


(A) unemployment decreases
(B) employment decreases
(C) the supply of labor remains the same
(D) demand remains the same
(E) none of the above

420. Diners wish to eat food at a restaurant. Because of derived demand,


the restaurant will
(A) spend money on television ads
(B) buy cooking ingredients
(C) reduce prices because of a decline in rent costs
(D) petition regulators to reduce its taxes
(E) create a differentiated menu

421. An auto parts manufacturer has five workers who can produce 10
auto parts. These parts sell for $3 each. If it hires a sixth worker, it
can produce 15 auto parts. Marginal revenue product for the sixth
worker is
(A) $45
(B) $30
(C) $5
(D) $15
(E) $50

AP Microeconomics_p001-224.indd 143 22/12/20 5:00 PM


144 ❯ 500 AP Microeconomics Questions to Know by Test Day

422. An ice cream stand has eight employees. They can produce 20 ice cream
cones, which sell for $2 each. If it hires a ninth employee, it can
produce 24 ice cream cones. If it hires a tenth employee, it can
produce 27 ice cream cones. Marginal revenue product for the tenth
employee is
(A) $40
(B) $48
(C) $8
(D) $6
(E) $54

423. A fishing boat has six crew members, who can catch 40 fish. The
fish can be sold at the market for $4 each. If the boat hires a seventh
crew member, the total catch will be 35 fish. Marginal revenue
product for the seventh crew member is
(A) $160
(B) $140
(C) $20
(D) –$20
(E) $0

424. A hot dog stand has four workers. It pays wages of $400 per day and
makes 100 hot dogs, which it sells for $5 each. With five workers, it
would pay an additional $120 in wages and make 20 more hot dogs.
The hot dog stand should not hire the fifth worker because
(A) marginal product would be negative
(B) total revenue would fall
(C) marginal product is declining
(D) marginal revenue product is less than marginal factor cost
(E) marginal factor cost is rising

425. A golf club manufacturer uses graphite to produce golf clubs.


Consumers start demanding more golf clubs. Because of derived
demand,
(A) the price of graphite will increase
(B) the price of graphite will decrease
(C) the supply of graphite will increase
(D) the supply of graphite will decrease
(E) none of the above

AP Microeconomics_p001-224.indd 144 22/12/20 5:00 PM


Factor Markets ❮ 145

426. A taco truck sells tacos for $3 each. With four employees, its revenue
is $450. If it hires a fifth employee, revenue will be $600. Marginal
product for the fifth employee is
(A) 200
(B) 150
(C) 100
(D) 50
(E) 30

427. A surfboard manufacturer adds a seventh worker. Total revenue


increases from $900 to $1,000. Production increases from 18 to
20 surfboards. Marginal revenue for a surfboard is
(A) $100
(B) $50
(C) $45
(D) $55
(E) $20

428. Marginal resource cost is equal to


(A) marginal revenue multiplied by the cost of a unit of output
(B) total cost for producing every unit of output
(C) the cost of producing the next unit of output
(D) the average cost of producing a unit of output
(E) total revenue divided by the market price of a unit of output

429. Which of the following would NOT result in higher demand for car
factory workers?
(A) The cost of steel used in car manufacturing decreases.
(B) Consumers purchase more cars.
(C) Consumers begin taking the train to work instead of driving
their cars.
(D) Firms construct more car manufacturing plants.
(E) Bus ridership declines as a result of economic expansion.

AP Microeconomics_p001-224.indd 145 22/12/20 5:00 PM


146 ❯ 500 AP Microeconomics Questions to Know by Test Day

430. Which example illustrates derived demand?


(A) Consumers increase their purchases of corn tortillas, so the
price of corn rises.
(B) Technology makes wheat production more efficient, so the
output of wheat flour increases.
(C) Workers move from country A to country B in search of
economic opportunities.
(D) A truck factory shuts down after workers go on strike.
(E) A heat wave causes the sugar beet price to rise on the market.

431. MRP for a hot dog stand’s fifth, sixth, and seventh workers is $140,
$120, and $100, respectively. If the wage for a hot dog stand worker
is $120, the hot dog stand should
(A) employ four workers
(B) employ five workers
(C) employ six workers
(D) employ seven workers
(E) employ eight workers

432. MRP for a shaved ice stand’s first, second, and third workers is
$95, $90, and $85 per day, respectively. The shaved ice stand
hires workers until MRP = MFC. The government increases the
minimum daily wage from $85 to $90. The shaved ice stand should
(A) hire one more worker
(B) lay off one worker
(C) shut down
(D) lay off two workers
(E) hire two more workers

433. A skateboard manufacturer is the only employer in a small town. It


has 10 workers who each earn $40,000. It can hire another worker
at a wage of $50,000 and produce skateboards worth $60,000. This
would make its current workers demand a wage of $50,000 as well.
If the firm hired another worker,
(A) profit would increase because MFC < MRP
(B) profit would be unchanged because MFC = MRP
(C) profit would fall because MFC > MRP
(D) profit would increase because the skateboard manufacturer has
a monopoly
(E) none of the above

AP Microeconomics_p001-224.indd 146 22/12/20 5:00 PM


Factor Markets ❮ 147

434. An auto-body shop has six employees, who each earn $30,000. If it
hired a seventh employee, it would have to pay all of its employees
$40,000 each. MFC for the seventh employee is
(A) $10,000
(B) $30,000
(C) $40,000
(D) $100,000
(E) $280,000

435. A soft drink factory can buy 10 tons of sugar for $200 per ton. If
it buys 11 tons of sugar, it must pay $250 per ton. MFC for the
eleventh ton of sugar is
(A) $200
(B) $250
(C) $500
(D) $550
(E) $750

436. A sandwich shop uses roast beef to make sandwiches. The price of
roast beef decreases. As a result,
(A) MFC decreases
(B) profit decreases
(C) MP decreases
(D) labor demand decreases
(E) fixed cost decreases

437. If a labor market is an oligopsony, in comparison with perfect


competition it will have
(A) lower wages
(B) lower unemployment
(C) higher output
(D) less market power
(E) more firms in the market

AP Microeconomics_p001-224.indd 147 22/12/20 5:00 PM


148 ❯ 500 AP Microeconomics Questions to Know by Test Day

438. A bicycle manufacturer purchases aluminum to make bicycles.


Demand for aluminum is a
(A) negative externality
(B) fixed cost
(C) derived demand
(D) negative externality
(E) factor of production

439. A cupcake manufacturer uses frosting to make cupcakes. If demand


for cupcakes increases,
(A) frosting demand will shift to the right
(B) frosting demand will shift to the left
(C) frosting demand will not change
(D) frosting price will decrease
(E) frosting quantity will decrease

AP Microeconomics_p001-224.indd 148 22/12/20 5:00 PM


CHAPTER
6
Market Failure and the
Role of Government
440. A company which produces computers emits carbon dioxide, which
causes $15,000 in social costs for the community. The company
earns $1,000,000 annually and considers itself to be a corporate
socially responsible entity. What would be the BEST way to make
the social benefit of the company operating equal to the marginal
costs of its operation for society?
(A) if the company was prohibited from operating anymore
(B) if the company had to pay a fine of $10,000 every year
(C) if the company were subject to a tax of 15 percent
(D) if the company were subject to a tax of 1.5 percent
(E) if the company were to pay a fine of $25,000 every year

441. Why are antitrust laws used by governments?


(A) to prevent inequality between the rich and the poor
(B) to prevent a company from using dangerous and potentially
untested products
(C) to prevent rapid inflation and the Federal Reserve from making
independent decisions
(D) to prevent corruption and government officials being bribed by
lobbyists
(E) to prevent monopolies forming and to ensure fair market
standards

❮ 149

AP Microeconomics_p001-224.indd 149 22/12/20 5:00 PM


150 ❯ 500 AP Microeconomics Questions to Know by Test Day

442. A good that is excludable to other consumers and rivals is known


as a
(A) public good
(B) quasi-​public good
(C) positive externality
(D) common resource
(E) private good

443. If a positive externality emerges in regard to the demand curve,


(A) MEB is not represented
(B) MEC is represented
(C) MSC is represented
(D) marginal private benefits are not represented
(E) none of the above

444. If the U.S. government increases a tax to help redistribute income


from the wealthy to the poor, it is following a
(A) proportional tax system
(B) tariff system
(C) progressive tax system
(D) regressive tax system
(E) neutral tax system

445. A negative externality will exist when


(A) all consumers receive an equal share of economic resources
(B) spillover costs are given to others not directly involved in the
consumption or production of a good or service
(C) the production of a good or service creates a spillover benefit
(D) members of a community pay for a good or service while others
do not yet benefit
(E) there is a large unequal distribution of income in a society

446. The private sector will NOT provide for a public good or service
because of
(A) the income effect
(B) the interest rates effect
(C) the free-​rider problem
(D) spillover costs
(E) all of the above

AP Microeconomics_p001-224.indd 150 22/12/20 5:00 PM


Market Failure and the Role of Government ❮ 151

447. A positive externality will exist when


(A) all consumers receive an equal share of economic resources
(B) spillover costs are given to others not directly involved in the
consumption or production of a good or service
(C) the production of a good or service creates a spillover benefit
(D) members of a community pay for a good or service while others
do not yet benefit
(E) there is an equal distribution of income in a society

448. If firms overproduce a good or service and charge a price that is too
low, it is likely that
(A) a negative externality exists
(B) a positive externality exists
(C) spillover costs are subsidized by the government
(D) the firm is at the shutdown point
(E) A, C, and D

449. A coal company has had a record-breaking quarter while operating


in Indiana. Which of the following would be an example of a
negative externality of production?
(A) the company deciding to hire additional employees
(B) the company’s factories causing environmental harm
(C) the company paying higher taxes because of additional revenue
(D) the company paying its suppliers more because of higher
demand
(E) the company earning more money through increased
production

AP Microeconomics_p001-224.indd 151 22/12/20 5:00 PM


152 ❯ 500 AP Microeconomics Questions to Know by Test Day

450. Samantha’s earnings are shown in the following table. Based on her
earnings and the amount of taxes paid, which kind of taxing system
is shown?

Year Revenue Taxes Paid


2017 $25,000 $2,500
2018 $45,000 $9,000
2019 $72,000 $36,000
2020 $150,000 $80,000

(A) flat
(B) progressive
(C) regressive
(D) incentivized
(E) supplemental

451. Tera International has discovered a permanent vaccine for the


common flu. The company filed a patent which grants it a natural
monopoly for this product. What can the government do to ensure
that Tera produces at the allocatively efficient quantity?
(A) The government should regulate a minimum amount of
vaccines the company needs to produce.
(B) The government should provide a lump-sum subsidy.
(C) The government should implement a per-unit tax.
(D) The government should implement a lump-sum tax.
(E) The government should set a price ceiling.

AP Microeconomics_p001-224.indd 152 22/12/20 5:00 PM


Market Failure and the Role of Government ❮ 153

452. Tim’s earnings are shown in the following table. How much taxes
would he have to pay in case he earned $135,000 in case the
country he lives in has a flat tax?

Year Revenue Taxes Paid


2018 $50,000 $5,000
2019 $75,000 $7,500
2020 $135,000 X

(A) $5,000
(B) $12,000
(C) $13,500
(D) $15,000
(E) $16,500

453. If the supply curve is shifted to S2, which of the following is TRUE?
(A) Marginal social costs are not reflected in the supply curve.
(B) Marginal social costs are reflected in the supply curve.
(C) Output has reached the socially optimum level.
(D) The demand curve will also shift to the left.
(E) both B and C

454. A private business will NOT produce a good like a public radio
station because
(A) MC is greater than MB
(B) there is zero profit for this good
(C) the government owns a monopoly for this type of service
(D) the free-​rider problem exists
(E) all of the above

AP Microeconomics_p001-224.indd 153 22/12/20 5:00 PM


154 ❯ 500 AP Microeconomics Questions to Know by Test Day

455. The information about the Gini coefficient for several countries
is provided in the following table. Which has the FEWEST issues
concerning inequality?

Country Gini Coefficient Value


United Kingdom 0.52
Baluchistan -0.4
Belgium 0.61
Slovenia 0.23
Canada 1

(A) the United Kingdom, as its value is closest to 0.5


(B) India, as it has the lowest value of the Gini coefficient
(C) Belgium, as its value is in the middle of the table
(D) Slovenia, as it has the lowest possible value within the table
(E) Canada, as its value is equal to 1

456. The reason a general sales tax is regressive is because


(A) low-​income families pay a higher portion of their income than
high-​income families do
(B) the tax is paid for by the seller, not the buyer
(C) the tax is paid for by the buyer, not the seller
(D) low-​income families pay less of the tax than households with
higher incomes do
(E) tax rates do not vary as compared with income

457. The fact that a firefighter’s job is more dangerous than a college
professor’s job is an example of
(A) compensating differential
(B) a negative externality
(C) the free-​rider problem
(D) the ability to pay principle
(E) the benefits received principle

AP Microeconomics_p001-224.indd 154 22/12/20 5:00 PM


Market Failure and the Role of Government ❮ 155

458. Of the following, which is a public good?


(A) a limousine
(B) tickets to a baseball game
(C) a subscription to the New York Times
(D) a fighter jet
(E) a bowl of ice cream

459. A “flat tax” is also known as


(A) ability to pay tax
(B) benefits received tax
(C) proportional tax
(D) income tax
(E) property tax

460. The information about earnings for Bill and Caroline is provided
in the following table. They do not live in the same country. Which
BEST describes the tax structure of these countries?

Bill’s Taxes Paid Caroline’s Taxes Paid


Year Revenue by Bill Revenue by Caroline
2017 $50,000 $10,000 $55,000 $11,000
2018 $65,000 $13,000 $100,000 $15,000
2019 $80,000 $16,000 $150,000 $15,000
2020 $100,000 $20,000 $200,000 $17,500

(A) Bill lives in a country that has a flat tax of 10 percent, while
Caroline lives in a country that has a regressive tax.
(B) Both Bill and Caroline live in a country that has a progressive
tax system, as they have paid more in taxes as their incomes
increased.
(C) Bill lives in a country that has a progressive tax rate, while
Caroline lives in a country that has a flat tax rate.
(D) Bill lives in a country that has a flat tax rate of 20 percent, while
Caroline lives in a country that has a regressive tax rate.
(E) Bill lives in a country that has a regressive tax rate, while
Caroline lives in a country with a progressive tax rate.

AP Microeconomics_p001-224.indd 155 22/12/20 5:00 PM


156 ❯ 500 AP Microeconomics Questions to Know by Test Day

461. Last year Anna earned $50,000 in gross income and paid $10,000 in
taxes. She recently received a promotion. So next year she will earn
$75,000 in gross income and pay $15,000 in taxes. This increase in
taxes is a result of
(A) a regressive tax
(B) a progressive tax
(C) a proportional tax
(D) an ability to pay tax
(E) none of the above

462. Fire protection provided by a fire department is an example of a(n)


(A) private good
(B) inferior good
(C) normal good
(D) public good
(E) complementary good

463. Nation A has a higher Gini ratio than nation B. You can conclude
that nation A
(A) has a highly unequal distribution of income compared with
nation B
(B) has a more equal distribution of income compared with
nation B
(C) has an equal distribution of income compared with nation B
(D) is allocating its resources to full efficiency
(E) is at full employment with no inflation

AP Microeconomics_p001-224.indd 156 22/12/20 5:00 PM


Market Failure and the Role of Government ❮ 157

Use the following graph to answer questions 464 and 465.

% of Income

% of Families

464. The graph represents


(A) the percentage of income tax families pay in the United States
(B) the percentage of families paying a regressive tax on their
incomes
(C) the percentage of families paying a flat tax on their incomes
(D) the percentage of families paying a progressive tax in the
United States
(E) the distribution of income among families in the United States

465. The arrow in the graph is pointing to the area between a perfect
distribution of income and the actual distribution of income. If the
Gini ratio is 0.9, it is said that this graph represents
(A) a highly unequal distribution of income
(B) a very equal distribution of income
(C) most families having the same socioeconomic status
(D) a society benefiting from egalitarianism
(E) a negative externality

AP Microeconomics_p001-224.indd 157 22/12/20 5:00 PM


158 ❯ 500 AP Microeconomics Questions to Know by Test Day

466. Suppose Anna earns $50,000 a year and pays $10,000 in taxes. Her
friend Sarah earns $100,000 a year and pays only $12,000 a year in
taxes. This is an example of
(A) a progressive tax
(B) a proportional tax
(C) a negative externality
(D) being in a higher tax bracket
(E) a regressive tax

467. Why are published official government statistics an example of a


public good?
(A) because only a select few can have access to them
(B) because they are easily excludable
(C) because they are developed by private entities
(D) because no individual can be excluded from seeing them
(E) because they are regularly published

468. If income increases and the marginal tax rate falls, this is also known
as a
(A) regressive tax
(B) proportional tax
(C) flat tax
(D) progressive tax
(E) tax bracket

469. As income increases, the marginal tax rate increases. This is known
as a
(A) progressive tax
(B) regressive tax
(C) flat tax
(D) proportional tax
(E) tax bracket

470. A ride in a taxicab is a form of


(A) a public good
(B) a private good
(C) a government subsidy
(D) a positive externality
(E) the free-​rider problem

AP Microeconomics_p001-224.indd 158 22/12/20 5:00 PM


Market Failure and the Role of Government ❮ 159

471. NASA, an organization run by the federal government, sends a


satellite into space. The satellite broadcasts information about the
solar system to people around the world at no charge. The satellite
represents
(A) a private good
(B) a government subsidy
(C) a positive externality
(D) a public good
(E) the free-​rider problem

472. The government helps with the redistribution of income


(A) only in a command economy
(B) only in a mixed economy
(C) through taxes
(D) through proportional taxes only
(E) only in a market economy

473. Why are efficiency-enhancing taxes used?


(A) to correct for a negative market outcome or inefficiency
(B) to subsidize industries that are trying to enter foreign markets
(C) to subsidize companies that are producing oil and similar
products
(D) to tax companies that do not pay federal taxes
(E) to compensate for the impact of flat corporate taxes

474. The U.S. government has decided to provide an efficiency-enhancing


subsidy for companies that want to use solar panels instead of
nonrenewable sources of energy. What is the goal of this subsidy?
(A) to ensure that more companies continue to use nonrenewable
energy sources
(B) to decrease the supply of solar panels
(C) to make solar panels more expensive
(D) to prevent corporate manufacturers from setting the price of
solar panels
(E) to make the marginal social benefit of solar panels equal supply

AP Microeconomics_p001-224.indd 159 22/12/20 5:00 PM


160 ❯ 500 AP Microeconomics Questions to Know by Test Day

475. A company in Portland has developed a new open-source software


for companies to use in database management. This software can
also be used by health care companies to storage data about patients
and work on analysis. What is this an example of ?
(A) the Nash equilibrium
(B) a positive externality
(C) a negative externality
(D) collective bargaining
(E) diminishing marginal returns

476. Which of the following is NOT an example of an efficiency-


enhancing tax?
(A) a tax on an oil company for emitting too much carbon dioxide
(B) a tax on an alcohol producer due to an increase in alcoholics
(C) a tax on tobacco to try and minimize the number of smokers
(D) a tax on foreign products that may compete with domestic
producers
(E) a tax on fast-food manufacturers due to an increase in the
number of obese people

477. A shopkeeper earns $100,000 and pays $15,000 in pension taxes.


The next year, the shopkeeper earns $150,000 and pays $15,000 in
pension taxes. The pension tax is
(A) progressive
(B) based on ability to pay
(C) proportional
(D) regressive
(E) none of the above

478. Five gas stations sell 100,000 barrels of fuel. Economists calculate
that burning this fuel results in air pollution that costs society
$1,000,000. If regulators want the marginal social cost of fuel to
equal the marginal social benefit of fuel, they should
(A) pay a $10 per barrel subsidy to gas stations
(B) levy a $10 per barrel tax on gas stations
(C) make each gas station pay $200,000 in taxes
(D) shut down the gas stations
(E) none of the above

AP Microeconomics_p001-224.indd 160 22/12/20 5:00 PM


Market Failure and the Role of Government ❮ 161

479. The government decides to encourage bus travel by paying a subsidy


to bus companies for each mile traveled. As a result of the subsidy,
(A) bus supply will shift to the right
(B) bus demand will shift to the right
(C) bus supply will shift to the left
(D) bus demand will shift to the left
(E) none of the above

480. The government encourages consumers to buy efficient refrigerators


by paying a rebate to consumers. As a result of the rebate,
(A) demand for efficient refrigerators will shift to the right
(B) supply for efficient refrigerators will shift to the right
(C) demand for efficient refrigerators will shift to the left
(D) supply for efficient refrigerators will shift to the left
(E) none of the above

481. Which item is a public good?


(A) transport provided by a personal automobile
(B) law enforcement provided by a police car
(C) delivery provided by a pizza delivery car
(D) transport provided by a rental car
(E) transport provided by a personal truck

482. Country X had a Gini ratio of 0.40 last year. This year, country X
reported that its Gini ratio decreased to 0.35. We can conclude that
(A) the income distribution in country X has become more equal
(B) the income distribution in country X has become less equal
(C) country X is experiencing a recession
(D) country X is experiencing an economic expansion
(E) country X has improved its freedom of the press

483. Country Y had a Gini ratio of 0.25 last year. This year, its Gini ratio
was 0.30. This indicates that
(A) the income distribution in country Y has become more equal
(B) the income distribution in country Y has become less equal
(C) country Y is experiencing a recession
(D) country Y is experiencing an economic expansion
(E) country Y reduced its business regulations

AP Microeconomics_p001-224.indd 161 22/12/20 5:00 PM


162 ❯ 500 AP Microeconomics Questions to Know by Test Day

484. Country A has a Gini ratio of 0.20. Country B has a Gini ratio of
0.25. Country C has a Gini ratio of 0.30. This indicates that
(A) country C has a more equal income distribution than country B
(B) country B has a more equal income distribution than country A
(C) country C has a more equal income distribution than
country A
(D) country B has a more equal income distribution than country C
(E) none of the above

485. Countries D and E have the same Gini ratio. Country D


implements a proportional tax and country E implements a
progressive tax. As a result,
(A) country E will have a higher Gini ratio than country D
(B) country D will have a higher Gini ratio than country E
(C) country D will still have the same Gini Ratio as country E
(D) country D will have the same Gini ratio as country E
(E) none of the above

486. Cities B and C have the same Gini ratio. City B implements a flat
tax and city C implements a regressive tax. As a result,
(A) city B will have a higher Gini ratio than city C
(B) city C will have a higher Gini ratio than city B
(C) city C will still have the same Gini Ratio as city B
(D) both cities will have the same Gini ratio
(E) none of the above

487. The government allows all citizens to hunt birds on federal land. The
birds are an example of a
(A) public good
(B) common resource
(C) quasi-​public good
(D) private good
(E) none of the above

AP Microeconomics_p001-224.indd 162 22/12/20 5:00 PM


Market Failure and the Role of Government ❮ 163

488. A city charges drivers a toll to cross a bridge. The bridge has enough
capacity to handle normal traffic. The bridge is a
(A) common resource
(B) public good
(C) quasi-​public good
(D) private good
(E) none of the above

489. A freeway is open to all citizens and does not charge a toll. At rush
hour, the freeway does not have enough room for all drivers. During
rush hour, the freeway is
(A) a public good
(B) a private good
(C) a quasi-​public good
(D) a common resource
(E) none of the above

490. According to the Coase theorem, consumers can reach an


agreement when
(A) negotiating costs are low
(B) marginal cost does not equal marginal benefit
(C) the government intervenes
(D) positive externalities are present
(E) negative externalities are not present

491. The Coase theorem would not apply to a factory that was polluting
a river near a city because
(A) the pollution affects many people
(B) the factory’s liabilities are very clear
(C) negotiating costs are low
(D) the negotiation process is efficient
(E) there are no negative externalities

492. If firms underproduce a good or service and charge a price that is


too high, it is likely
(A) a negative externality exists
(B) a positive externality exists
(C) spillover costs are high
(D) the economy is expanding
(E) there is a government subsidy

AP Microeconomics_p001-224.indd 163 22/12/20 5:00 PM


164 ❯ 500 AP Microeconomics Questions to Know by Test Day

493. When a government subsidizes student grants to attend university


for all students,
(A) it is creating a consumer subsidy
(B) it is creating a producer subsidy
(C) university education is a public good
(D) university education is a common resource
(E) both A and D

494. When a government grants a property tax exemptions to universities


but many students still cannot afford to attend universities,
(A) it is creating a consumer subsidy
(B) it is creating a producer subsidy
(C) university education is a public good
(D) university education is a private good
(E) both B and D

495. The government provides elementary school education to all


students, and once the students graduate they begin work at local
businesses. Productivity increases at these businesses. This is an
example of
(A) spillover costs
(B) spillover benefits
(C) negative externalities
(D) the Gini ratio
(E) market failure

496. Phantom demand refers to


(A) goods and services that consumers purchased in the past
(B) unaffordable goods and services
(C) market failure
(D) demand for public goods that consumers cannot purchase
directly
(E) demand that would exist if the economy expanded

497. The demand curve for a public good is


(A) the horizontal sum of each consumer’s demand curve
(B) the vertical sum of each consumer’s demand curve
(C) not measurable
(D) always horizontal
(E) always vertical

AP Microeconomics_p001-224.indd 164 22/12/20 5:00 PM


Market Failure and the Role of Government ❮ 165

Use the following graph to answer questions 498 and 499.

MC

MB1

MB2

MB3

MB SUM

498. The graph shows demand curves for a(n)


(A) private good
(B) inferior good
(C) luxury good
(D) public good
(E) capital good

499. The equilibrium price of this public good will be


(A) MB1 = MC
(B) MB2 = MC
(C) MB3 = MC
(D) MB SUM = MC
(E) none of the above

500. In comparison to the demand curve for all consumers, the slope
of the demand curve for an individual consumer of a public good
will be
(A) more steep
(B) more flat
(C) the same
(D) horizontal
(E) vertical

AP Microeconomics_p001-224.indd 165 22/12/20 5:00 PM


This page intentionally left blank

AP Microeconomics_p001-224.indd 166 22/12/20 5:00 PM


ANSWERS

Chapter 1
21. (D) Choice (D) is the correct answer because Deirdre can make only one decision
regarding how she will spend her time. An opportunity cost represents the next best thing
missed out on. In this case, it is missing out on the $10 per hour babysitting job and
not the $7 per hour library job. All other choices are incorrect because they incorporate
the two choices, whereas opportunity costs represent only the next best alternative.
22. (A) Choice (A) is the correct answer because with any choice people make, they
must look at the relationship between marginal benefit and marginal cost. If a decision,
whether it be purchasing an item or going for a walk, costs more than the benefit, then
the person should not do it.
23. (E) Choice (E) is the correct answer because factors of production refer to the
inputs actually used to manufacture the product, while money is used to purchase
these inputs.
24. (A) Choice (A) is correct because the production possibilities curve represents the
maximum output between two goods using scarce resources. As such, it represents
the increasing opportunity costs incurred when the production shifts to more of one
product than the other. The more a producer chooses to make of a product, the more
the opportunity cost increases for the product not being produced. This represents the
law of increasing costs.
25. (E) The correct choice is (E) because trade will exist between two countries if there
is a comparative advantage between the two countries or each country is producing its
good at a lower opportunity cost than the other.
26. (D) The correct choice is (D) because if a society is overallocating, then it is not
utilizing its resources to allocative efficiency. This would imply that the marginal benefit
(MB) for producing goods and services with these resources is less than the marginal
cost (MC). In any economic decision, it is best to have either MB > MC or MB = MC.
27. (C) Choice (C) is the best answer because it best reflects the idea of comparative
advantage. If a producer can make a good at a lower opportunity cost than all other
producers, it is said that he or she has a comparative advantage. Richard can mow the
lawn at a lower opportunity cost than Michele, so it would be mutually beneficial if he
mowed the lawn and Michele walked the dog.
28. (B) Choice (B) is the correct answer because opportunity cost refers to the value of
what is given up. In this scenario, Joe decided to maximize hot dog production, giving
up the opportunity to produce 400 hamburgers with his supplies. The prices of hot
dogs and hamburgers are not needed for this calculation.

❮ 167

AP Microeconomics_p001-224.indd 167 22/12/20 5:00 PM


168 ❯ Answers

29. (A) Choice (A) is correct because factors of production refer to all the units that
are used in the production of goods and services in hopes of making a profit. They are
categorized as land, labor, capital, and entrepreneurship.
30. (B) Choice (B) is correct because the law of increasing costs refers to how the more
a good or service is produced, the more its opportunity costs increase. If there is an
increase in the production of good A, then the opportunity cost, the next most valuable
alternative, increases for good B.
31. (A) Choice (A) is correct because trade will exist between two countries if there is
a comparative advantage between the two countries or each country is producing its
good at a lower opportunity cost than the other.
32. (A) Choice (A) is correct. According to the law of comparative advantage, two
nations should specialize production by producing goods at the lowest possible cost.
These nations should then engage in trade for the products they stopped producing
to specialize. Since Spain has a lower opportunity cost for producing soccer balls as
compared to Germany, it should stop producing golf balls and focus on soccer balls.
33. (B) The correct choice is (B). Because the potential output could increase, the pro-
duction possibilities curve would shift to the right.
34. (D) The correct choice is (D). While all products are scarce, there are some prod-
ucts where our consumption of them does not significantly impact supply; these are
non-rivalrous goods.
35. (C) Choice (C) is correct because opportunity cost is what is given up when
someone makes a decision. If your school decides to construct a new performing arts
center, all of the resources that could have been used in another project are given up.
The resources must go into the construction of the performing arts center.
36. (A) The correct choice is (A). Whenever there is a cost to consuming a good and
it diminishes the supply of the good, it can be considered scarce.
37. (D) Choice (D) is the best answer because marginal analysis refers to the decision-​
making process based on marginal benefits versus marginal costs.
38. (B) Choice (B) is the best answer because a market system has complete decen-
tralization of decision making for firms and consumers. A mixed economy refers to a
combination of capitalism and socialism, where there are elements of centralized plan-
ning and a free market. Economies with market systems do not currently exist because
every nation’s government makes some economic decisions. A command economy is
the opposite of a free market (where prices and the answers to the fundamental ques-
tions of economics are answered by consumers and producers). A command economy
is also known as a planned economy.
39. (C) The correct choice is (C). While France has an absolute advantage in both
products, the economies of these countries would be most efficient if France focused
on producing cheese, where it is more effective, while Italy should produce olive oil.

AP Microeconomics_p001-224.indd 168 22/12/20 5:00 PM


Answers ❮ 169

40. (B) The correct choice is (B) which best reflects the concept of absolute advantage,
while (A) reflects the concept of comparative advantage.
41. (C) Choice (C) is the best answer. At the point on the production possibilities curve
where MB = MC, production is both productively efficient and allocatively efficient.
42. (D) Choice (D) is correct because a market system is characterized by a decentral-
ization of decision making for firms and consumers.
43. (A) Choice (A) is correct because this point is located inside the current produc-
tion curve, not along the curve. The curve would have to move left to reach this point,
shrinking inward. This would indicate fewer resources and less production capacity
available, which could indicate a recession.
44. (D) The correct choice is (D) because point E and point D are both on the same
production possibilities curve, so resource availability and production capacity have not
changed and there is no evidence of recession or economic expansion.
45. (D) The correct choice is (D). If Choice (C) occurs, production would not change,
and all other outcomes would shift the curve to the right.
46. (E) The correct choice is (E) is accurate because a recession is technically classified
by a decline of economic activity in at least two consecutive quarters.
47. (C) The correct answer is (C). A nation with a mixed economy often lets the
free market handle decisions for most industries and uses a command approach for
selected industries, such as industries related to food production or national security.
Most nations have mixed economies; a completely free market economy would have
no public sector, and a completely command economy would have no private sector.
48. (D) The correct answer is (D). A ruby is not necessary for survival while people
need water to remain alive, yet a ruby is much more expensive than a bottle of water.
This is the paradox of value. It occurs because rubies are scarcer than bottles of water,
and scarcity results in a higher price.
49. (B) The correct answer is (B), the truck. A truck can be driven regularly for more
than three years, while the other items will wear out or be used up in less than three
years so they are considered nondurable goods.
50. (C) The correct choice is (C), a barter economy. This type of economy preceded
the market economy and is very rare now. Barter economies can still exist under special
conditions, such as when buyers and sellers wish to evade high taxes or a nation’s
currency is experiencing hyperinflation.
51. (C) The best choice is (C). Marginal utility declines as additional units of a good
are purchased. After buying three cans of soda, the consumer considers the fourth can
less valuable.

AP Microeconomics_p001-224.indd 169 22/12/20 5:00 PM


170 ❯ Answers

52. (B) The best choice is (B). The law of comparative advantage indicates that France
should specialize in producing cheese and Switzerland should specialize in producing
chocolate and both countries should trade with one another. If a trade embargo stops
this exchange, France would need to produce its own chocolate and Switzerland would
need to produce its own cheese. As a result, France would produce less cheese and
Switzerland would produce less chocolate.
53. (B) The correct choice is (B). A typical production possibility curve is concave,
indicating increasing opportunity costs. As the nation moves toward maximizing pro-
duction of one crop, its production of the other crop falls more rapidly. That isn’t the
case here. This lack of increasing opportunity costs indicates that the nation’s resources
are equally suitable for producing either crop.
54. (E) Choice (E) is the best choice. Specialization creates a concave shape for the
production possibilities curve. Specialization can also increase total production by mak-
ing workers and factories more efficient. An unspecialized economy probably isn’t as
productive in overall terms as a specialized economy.
55. (D) Choice (D) is correct. The bakery can produce either 300 cookies and 140 loaves
of bread at point A, or zero cookies and 160 loaves of bread at point B. Cookie produc-
tion fell from 300 units to zero, so 300 cookies is the opportunity cost.
56. (C) Choice (C) is correct. At point A the bakery produces 140 loaves of bread, and
at point D the bakery produces 80 loaves of bread. Bread production fell by 60 loaves.
Cookie production remained unchanged at 300 cookies.
57. (D) Choice (D) is the best answer. A biased technology improves production effi-
ciency for only one good. Choice (A) is incorrect because an economic expansion
would increase the maximum production quantity for cars as well.
58. (E) Choice (E) is the best answer. If resources allocated to bread and car produc-
tion do not change, car quantity will remain the same and bread quantity will increase.
59. (A) Choice (A) is correct. If the spinach quantity doesn’t change after the biased
technology increases the efficiency of spinach production, the nation must have allo-
cated more resources to lettuce production so the lettuce quantity will increase.
60. (A) Choice (A) is correct. The law of diminishing marginal utility states that as
a person increases consumption of a good, satisfaction or utility decreases with each
marginal consumption of the good. For example, say you really liked hot dogs and you
started eating them every day. With consumption of each hot dog, there would come
a point where you would like them less and less.
61. (A) Choice (A) is the best answer. Marginal utility is defined as the additional sat-
isfaction or utility a consumer receives from consuming one additional unit of a good.
Choice (B) is incorrect because it refers to the marginal revenue cost. Choice (C) is incor-
rect because it refers to the marginal propensity to save. Choice (D) is incorrect because it
describes marginal cost. Choice (E) is incorrect because it describes marginal cost.

AP Microeconomics_p001-224.indd 170 22/12/20 5:00 PM


Answers ❮ 171

62. (B) Choice (B) is the best answer. The law of diminishing marginal utility states
that as a person increases consumption of a good, satisfaction or utility decreases with
each marginal consumption of the good.
63. (B) Item I is not a part of a market system because it describes the operations of
a traditional economy. Item III is not part of a market system because it describes a
command economy. Choice (B) is correct because Item II describes the interactions
between consumers and producers.
64. (E) The correct choice is (E). Tariffs are carried over to consumers and, as a result,
U.S. consumers would likely have to pay for the increase caused by the tariff.
65. (B) Choice (B) is the best answer. Choice (C) is correct, but this describes the
income effect.
66. (C) Choice (C) is the best answer. Choice (B) describes the substitution effect.
67. (D) Choice (D) is the best answer. If car manufacturers exited the market, the price
of cars would rise. A higher car price would result in a lower quantity demanded, but
demand for cars would not fall. The other choices would reduce car demand.
68. (C) Choice (C) is correct. Increased supply would reduce sugar beet prices, increas-
ing quantity demanded. The other choices would result in higher demand for sugar
beets.
69. (A) Choice (A) is correct. The number of buyers is not a determinant of supply
and the number of producers is not a determinant of demand, but both factors affect
elasticity.
70. (B) Choice (B) is correct. Perceived health benefits would increase demand for
mushrooms, resulting in a higher quantity supplied, but wouldn’t increase supply.
71. (C) Choice (C) is correct. The demand curve for strawberries would shift left,
resulting in a lower quantity supplied, but supply wouldn’t change.
72. (A) Choice (A) is correct. Dividing 50 percent by 10 percent results in a price elas-
ticity of 5, which is greater than 1, so television set demand is price elastic.
73. (C) Choice (C) is correct. Demand for sunscreen will shift to the right, but sun-
screen supply won’t increase.
74. (C) Choice (C) is correct. Higher supply and higher demand will increase the
equilibrium quantity, but the change in market price can’t be determined.
75. (A) Choice (A) is correct. Dividing 10 percent by 40 percent results in a price
elasticity of 0.25.
76. (E) Choice (E) is the best answer. All of these regulations could result in a dead-
weight loss, creating either a shortage or a surplus in the market.

AP Microeconomics_p001-224.indd 171 22/12/20 5:00 PM


172 ❯ Answers

77. (C) Choice (C) is correct. If the market equilibrium price and quantity match, the
market does not have an effective price floor/ceiling or undersupply/oversupply. An
ineffective price floor or ceiling could still be present though.
78. (D) The best answer is Choice (D), because the buyer is willing to pay less for each
additional apple. While Choice (B) and Choice (E) would result in the same total price
for three apples, these answers provide no evidence of declining utility.
79. (B) The correct choice is (B). Tariffs are generally defined as a tax on foreign prod-
ucts that then carries over to domestic consumers. Because domestic consumers have
to pay this added cost, this results in deadweight costs.
80. (D) The correct choice is (D). Because 10 percent of $80 is $8, this means that the
total price of oil once a tariff has been placed on it is $88.
81. (C) The correct choice is (C). This is the most precise response, as any quantitative
restrictions to trade are considered to be quotas.
82. (A) The correct choice is (A). Assuming that a consumer is perfectly rational, he
or she would make the decision based on the largest increase to the consumer’s total
utility.
83. (D) The correct choice is (D). This choice best reflects an example of a sunk cost, as
this is a cost the company needs to do regardless of the level of output it has. It is also
something the company cannot recover.
84. (B) The correct choice is (B). Because the opportunity is the cost of a missed oppor-
tunity, company B is a better option than company A.

Chapter 2
85. (A) According to the law of demand, all else being equal, when the price of a
good or service increases, the quantity demanded decreases. Choice (A) is the best
answer because it is the inverse of the question being asked.
86. (B) Choice (B) is correct because a normal good is a good that increases in demand
as income increases. Since Michael received an increase in his yearly salary, there was
an increase in demand for steak, and steak is an expensive meat product, it is a normal
good. Choice (A) is incorrect because an inferior good is a good for which demand
decreases as income increases. Choice (E) is incorrect because eating steak is not a
necessity. Choices (C) and (D) are incorrect because eating steak does not affect the
determinants of supply and demand.
87. (D) Choice (D) is correct. An inferior good is a good for which demand decreases
as income increases. Since Peter received an increase in salary, he can afford better shoes
than thrift-​store-​bought shoes.

AP Microeconomics_p001-224.indd 172 22/12/20 5:00 PM


Answers ❮ 173

88. (E) Choice (E) is correct. Tin is a factor of production used in the production of
guitar strings. Firms will produce more guitar strings if the costs related to the produc-
tion of the product decrease. If the price of tin decreases, then producers are willing and
able to produce more because of a decrease in a factor of production.
89. (C) Choice (C) is correct. Remember that people will buy more of a good if the
price is low and less of a good if the price is high. This illustrates an inverse relationship.
According to the law of demand and all else being equal, when the price of a good or
service increases, the quantity demanded decreases.
90. (B) The question asks for only the quantity supplied, so any question that refers to
price may be eliminated. Choice (B) is the best answer because if the price of a good
increases, its substitutes will experience an increase in the quantity demanded.
91. (B) The correct choice is (B) because according to the law of demand, if the price
of figs increases, consumers will demand fewer figs.
92. (E) Tennis rackets and tennis balls are complementary goods. You cannot use a ten-
nis racket without tennis balls. Therefore, if the price of one good increases or decreases,
the price of the complementary good increases or decreases. Choice (E) is the best
answer because it illustrates the complementary effect.
93. (A) Choice (A) is correct. Chicken is a normal good. Remember that a normal good
is a good that increases in demand as income increases. If the demand curve for chicken
shifted to the left, then consumer income would show an overall decrease.
94. (C) Choice (C) is correct. If the price of hot dogs increases, then the price of hot
dog buns will increase as well.
95. (B) Choice (B) is correct. All else being equal, in a free market system, the price
of a good or service is determined at the equilibrium point where supply and demand
intersect.
96. (A) Choice (A) is correct. The graph represents the inverse relation stated in the law
of demand. The graph represents movement from point B to point A. This illustrates
that the quantity demanded decreased because of an increase in price.
97. (C) Choice (C) is correct. If the price of a key ingredient decreases, then the cost of
production will decrease. This will lead Doherty Industries to offer more of the product
at all possible price levels. This illustrates the law of supply.
98. (E) Choice (E) is correct. The producer would gain more revenue by raising the
price for an inelastic good. For example, if the gasoline price increased 20 percent and
quantity demanded fell 10 percent, gasoline would be inelastic and the revenue from
higher-​priced gasoline would outweigh sales lost because of the higher price. If the
price increased 20 percent and quantity demanded did not change, the good would be
considered perfectly inelastic.
99. (A) Choice (A) is correct. Most necessity goods are price inelastic.

AP Microeconomics_p001-224.indd 173 22/12/20 5:00 PM


174 ❯ Answers

100. (B) The elasticity formula is percent change in quantity divided by percent change
in price. If the answer is greater than 1, the good is said to be price sensitive, or elastic.
If the answer is less than 1, the good is said not to be price sensitive. Therefore,
Choice (B) is the best answer because 40/10 = 4.
101. (D) Refer to question 69. Therefore, Choice (D) is the best answer because
0/30 = 0.
102. (B) Choice (B) is correct. Sally’s income decreased and her demand for canned
vegetables increased as a result, so canned vegetables are an inferior good.
103. (B) The movement from D1 to D2 represents an increase in demand. Choice (B)
is the best answer because a decrease in the price of a substitute good would decrease
demand for this product and increase demand for the substitute good.
104. (A) The movement from S1 to S2 represents a decrease in supply. Choice (A) is the
best answer; if the price of a key ingredient to the production of the product decreased,
then production would increase because of a decrease in the costs of production.
105. (D) A price ceiling refers to the maximum price at which a producer is allowed
to sell a good or service. This is usually instituted by law from the government and
helps ensure fair business practices. An effective price ceiling is set at a point below the
equilibrium price.
106. (A) Choice (A) is the best answer because over a longer period of time, producers
have a better chance to adjust to a change in price: hire more workers and build more
plants. Over time, elasticity of supply will be greater.
107. (C) Choice (C) is correct. Consumer surplus is measured by calculating the dif-
ference between what consumers are willing and able to pay for a good or service and
its relative market price. A surplus occurs if the consumer is willing to pay more than
the market price for a good or service.
108. (C) Because fertilizer is a factor of production for grapes, production costs for
grapes will increase and thus Choice (C) is correct; the supply of grapes will decrease.
109. (A) Choice (A) is correct. An increase in demand will signal to suppliers to raise
the prices of their products. If demand rises, then suppliers will raise their prices and
a higher quantity will be sold.
110. (C) Choice (C) is correct. Since there was an increase in price for product X, a
decrease in demand should result. If the demand for product Z increases as a result of
an increase in the price of product X, then product Z is a substitute good.
111. (B) Choice (B) is correct. A price ceiling refers to the maximum price at which
a producer is allowed to sell a good or service. This is usually instituted by law from
the government and helps to ensure fair business practices. If credit card companies
are regulated to institute a maximum amount on how much interest they can charge,
then it is a price ceiling.

AP Microeconomics_p001-224.indd 174 22/12/20 5:00 PM


Answers ❮ 175

112. (B) Choice (B) is correct. A shortage is defined as a situation where the quantity
demanded exceeds the quantity supplied. More and more people are demanding a
particular good, but producers do not have an adequate supply of the good.
113. (A) Choice (A) is the best answer because it reflects the law of supply: producers
are more willing and able to produce more of a good as price increases.
114. (B) The correct choice is (B). The point where the supply and demand curve
intersect is called market equilibrium.
115. (C) The correct choice is (C). If the price of a complementary good decreased, this
good may be seen as more valuable and thus the demand curve would shift to the right.
This would result in a new point of market equilibrium that would be shifted to the
right. All of the other remaining answer choices would shift the demand curve to the left.
116. (D) Choice (D) is correct. A surplus exists when the quantity supplied is greater
than the quantity demanded. A surplus is another way of saying there is an excess
of supply. Choice (B) is incorrect because the equilibrium point is the intersection
between supply and demand, where the supply equals the demand. Choice (C) is incorrect
because it refers to the percentage of income distribution in a society.
117. (B) Choice (B) is correct. Establishing an effective price floor will create a market
surplus because suppliers will produce more than consumers demand. If a market surplus
already exists, the size of the surplus will increase.
118. (C) Choice (C) is the best answer because if there are too few substitutes and
little time for changes to production to be made, then demand is said to be inelastic.
Therefore, substitutes and time are determinants of elasticity.
119. (B) Choice (B) is correct. The supply of wheat increases, so if demand doesn’t
change, the wheat price falls. With inexpensive wheat on the market serving as a sub-
stitute good for corn, the corn price also falls.
120. (A) Choice (A) is correct. The elasticity formula is percent change in quantity
divided by percent change in price. If the answer is greater than 1, the good is said to
be price sensitive. If the answer is less than 1, the good is said not to be price sensitive.
Because 4/2 = 2, the good would be price elastic.
121. (D) Choice (D) is correct. An increase in demand for hamburgers shifts the
demand curve for buns to the right and the price of buns rises.
122. (A) Choice (A) is correct. Deadweight loss refers to the lost net benefit to society
caused by a movement from competitive market equilibrium. When price increases
above the equilibrium because of taxes, a deadweight loss occurs.
123. (A) Choice (A) is correct. Deadweight loss refers to the lost net benefit to society
caused by a movement from competitive market equilibrium. When the quantity pro-
duced is above the market equilibrium quantity, deadweight loss occurs.

AP Microeconomics_p001-224.indd 175 22/12/20 5:00 PM


176 ❯ Answers

124. (C) Choice (C) is correct. The price elasticity formula is percent change in quan-
tity divided by percent change in price. If the answer is greater than 1, the good is said
to be price sensitive. If the answer is less than 1, the good is said not to be price sensitive.
However, if the answer is zero, then the demand is completely independent from the
price. Consumers will purchase the good regardless of the price.
125. (B) Choice (B) is correct. If joint demand exists for two goods, they are comple-
mentary goods.
126. (A) Choice (A) is correct. The price increase for bell peppers would shift the
demand curve for artichokes to the right. A demand shift from D1 to D2 is a move to
the right, and the other choices are moves to the left.
127. (C) Choice (C) is correct. A price drop for tomatoes causes a larger shift in arti-
choke demand, so tomatoes are a better substitute for artichokes.
128. (A) Choice (A) is the best answer because a consumer surplus is the difference
between what a consumer is willing and able to pay and the actual market price. If the
consumer was willing to pay more for the product than the established market price,
then there is a consumer surplus.
129. (E) Choice (E) is the best answer because establishing an ineffective price ceiling
will not influence quantity demanded or supplied. It only ensures that producers will
not exceed a specific price for their product, thus protecting consumers.
130. (A) Choice (A) is correct. If the supply curve for grape jelly shifts to the right,
the price of grape jelly will fall. This will increase demand for peanut butter because
it is a complementary good.
131. (D) Choice (D) is the best answer because the complement effect refers to an
increase or decrease in demand for a product that works with another product. For
example, if the demand for tennis rackets increases, so will the demand for tennis
balls. This reflects a direct relationship, whereas all the other choices reflect an inverse
relationship.
132. (D) Choice (D) is correct. An increase in technology is a major change in the
factors of production. If technology increases, then production output will increase as
well. This will result in greater profits. However, since this is a perfectly competitive
market, price will remain the same.
133. (A) Choice (A) is correct. Tacos are a better substitute for burritos, so the cross-​
price elasticity of demand will be higher.
134. (A) Choice (A) is the best answer because if a firm is utilizing its resources to
allocative efficiency, then price (P) will equal marginal cost (MC). Keep in mind the
opposite notion: since monopolies are notoriously inefficient, they operate where P is
greater than MC.

AP Microeconomics_p001-224.indd 176 22/12/20 5:00 PM


Answers ❮ 177

135. (B) Choice (B) is correct. Corn oil is less substitutable for olive oil so its cross-​
price elasticity of demand will be lower.
136. (C) Price elasticity of demand is extremely useful because it helps firms and econo-
mists predict consumer behavior if there is a change in price. Choice (C) is the best
answer.
137. (B) Choice (B) is correct. Remember, an effective price ceiling will always be
demarcated below the market equilibrium price on a supply-​and-​demand chart.
138. (A) Calculating the cross-​elasticity refers to the measurement taken of how
sensitive the consumption of one good is to the change in price of another good.
Choice (A) is the best answer.
139. (B) The correct choice is (B) because the income effect describes the relation-
ship between income and demand: as a person’s income increases or decreases, it will
increase or decrease demand for a product. Choice (A) is incorrect because it describes
the substitution effect. Choice (C) is incorrect because it describes marginal utility.
Choices (D) and (E) are incorrect because they are not related to the income effect.
140. (C) Choice (C) is correct. A price increase from $1 to $1.25 is a 25 percent increase.
Quantity demanded fell 50 percent. 50%/25% = 2, which is above 1, so demand for
golf balls is elastic.
141. (A) Choice (A) is correct. The price fell 40 percent and train ridership (quantity
demanded) rose 20 percent. 20%/40% = 0.5, which is below 1, so demand for train
tickets is inelastic.
142. (C) The correct choice is (C). All of the remaining answer choices are very elastic
goods, while it is very difficult to find a substitute for insulin.
143. (B) Choice (B) is the best answer. Demand for Giffen goods does increase when
the price rises, but Giffen goods are inferior goods. Veblen goods are luxury goods used
for conspicuous consumption. Consumers buy them to show off their wealth. Thus, a
luxury handbag is a Veblen good if a higher price results in higher demand.
144. (D) Choice (D) is the best answer. If an inferior good is a necessity and substitutes
are not available, a worker may demand more of the inferior good and fewer normal
goods if the price of the inferior good increases. For example, the shoe store worker
could buy more rice and give up hot sauce used to season the rice. Therefore, the
income effect affects demand more strongly than the substitution effect, indicating the
presence of a Giffen good.
145. (B) Choice (B) is correct. If a perfect substitute is available for a good, increasing
the price of the good will cause quantity demanded to fall to zero.
146. (A) The correct choice is (A). Only oil would be considered inelastic, as the value
of elasticity is lower than 1. In all of the remaining answer choices, the value is higher
than 1, making them unit elastic.

AP Microeconomics_p001-224.indd 177 22/12/20 5:00 PM


178 ❯ Answers

147. (B) Choice (B) is correct. If fewer substitute products were available, the demand
curve would shift to the right, not the left.
148. (E) Choice (E) is correct. The first four factors would reduce production costs,
shifting the supply curve to the right.
149. (A) The correct choice is (A). Unit elastic supply occurs when a change in the
supply provided causes an equally corresponding change in the price.
150. (B) The correct choice is (B). As supply is unit elastic, it is clear that a 10 percent
increase in quantity would result in the price increasing by 10 percent.
151. (A) Choice (A) is the best answer. In a market economy without rent control laws,
apartments supplied would equal apartments demanded at point C. Rent control laws
would cause apartments supplied to decrease toward A2, while apartments demanded
would increase toward B2. Applicants would outnumber available rental units, result-
ing in a housing shortage.
152. (A) The size of a deadweight loss can be calculated by measuring the area between
quantity demanded, quantity supplied, and equilibrium price/quantity. Choice (A) is
correct, the deadweight loss caused by a housing shortage would be represented by the
area inside the triangle bounded by points A2, B2, and C.
153. (B) Choice (B) is the best answer. The supply curve will shift to the left and a
smaller quantity will be sold at higher prices.
154. (B) The best choice is (B). A price floor will have no effect if it’s set below the
equilibrium price.
155. (A) Choice (A) is correct. Producer surplus is the area to the left of the supply
curve and below the price equilibrium. The area above the price equilibrium and to the
left of the demand curve is consumer surplus.
156. (A) Choice (A) is the best answer. If consumers expect lower auto prices, demand
shifts to the left.
157. (C) Choice (C) is correct. If consumers expect higher auto prices, demand shifts
to the right.
158. (C) The correct choice is (C) is the only one where the value of the elasticity of
demand is equal to 1. In such a case, demand is unit elastic, and increase in the quantity
demanded is equal to the price.
159. (D) The correct choice is (D). Most of the answer choices are related to trade,
while a price floor is the minimal price mandated by the government that can be set. A
common example is the minimum wage.
160. (B) Choice (B) is correct. If the price falls below the intervention price, the
government begins buying goods in the market. This shifts the demand curve to the
right.

AP Microeconomics_p001-224.indd 178 22/12/20 5:00 PM


Answers ❮ 179

161. (A) Choice (A) is correct. Dividing a 40 percent increase by a 20 percent increase
results in a cross-​price demand elasticity of 2. This result indicates that tomatoes and
lettuce are substitute goods.
162. (B) Choice (B) is correct. If quantity demanded for blackberry jam divided by 20
percent equals –2, then quantity demanded must be –40 percent. This result indicates
that blackberry jam and peanut butter are complementary goods.
163. (A) Choice (A) is correct. The price floor causes the price to shift from the equi-
librium point to the point where the supply curve meets line A, resulting in a surplus.
The size of the surplus is the area of the triangle bounded by the points where line A
crosses the supply and demand curves and the equilibrium price point. This is consid-
ered a deadweight loss.
164. (B) Choice (B) is correct. A price ceiling represented by line B would result in
a shortage represented by the area above line B and between the supply and demand
curves. This is considered a deadweight loss.
165. (B) Choice (B) is correct. If the price of chicken increases 10 percent and cross-​
price elasticity of demand is 5, then quantity demanded for pork will increase 50
percent. The new quantity demanded for pork will be 10 + (10 × 50 percent), or 15.
166. (B) Choice (B) is correct. Dividing –20 percent by 50 percent results in cross-​
price demand elasticity of –0.4. Thus, bread and cheese are complementary goods.
167. (C) Choice (C) is correct. If the corn price rose 40 percent and cross-​price elas-
ticity of demand is 0.75, quantity demanded for wheat rose by 30 percent. If the new
quantity demanded is 13, then the original quantity demanded was 10.
168. (A) Choice (A) is correct. Quantity demanded increased from 50 to 80, which is
a 60 percent increase. Dividing 60 percent by 30 percent results in a price elasticity of
demand of 2. These are substitute goods.
169. (E) Choice (E) is correct. Quantity supplied fell 15 percent and price fell 30 percent,
so price elasticity of supply for peaches is 0.5.
170. (B) Choice (B) is the best answer. The fashion retailer can easily adjust output, but
the motorcycle manufacturer needs more time to adjust output. Thus, price elasticity
of supply will be low for the fashion retailer and high for the motorcycle manufacturer.
171. (B) Choice (B) is the best answer. According to the law of supply, quantity sup-
plied should decrease because suppliers will be less willing to produce the good.
172. (A) Choice (A) is correct. Income elasticity of demand is the percentage change
in quantity demanded divided by the percentage change in income.
173. (A) Choice (A) is the best answer. The increased health care costs are an example
of a negative externality resulting from increased soda demand. There is no evidence
that production costs decreased or the government regulated the price of soda.

AP Microeconomics_p001-224.indd 179 22/12/20 5:00 PM


180 ❯ Answers

174. (B) Choice (B) is correct. Choice (C), consumer expectations, is a determinant
of demand as well.
175. (A) Choice (A) is correct. The quantity of apples supplied doubles from 10 to 20
when the price doubles from $0.75 to $1.50, so apple supply is unitary elastic.
176. (C) Choice (C) is correct. Income elasticity of demand for oatmeal is –15 percent
divided by 30 percent, or –0.5.
177. (A) Choice (A) is correct. When income elasticity of demand is zero, the good is
a sticky good.
178. (C) Choice (C) is correct. The formula for the tax paid by the consumer is PES
divided by (PES minus PED). This is 0.6/(0.6 − –0.2), or 0.6/0.8, which comes out to
0.75. Since the tax was $50, the consumer pays $50 × 0.75, or $37.50.
179. (B) Choice (B) is correct. The formula is 0.6/(0.6 − –0.4). This equals 0.6/1.0, or
0.6, so the car buyer pays 60 percent of the tariff.
180. (B) The correct choice is (B). This would represent an expansion of supply where
both the price and quantity were increased.
181. (C) The correct choice is (C). This is the only choice that would cause an increase
in the price on the existing curve. (A), (C), and (D) would result in a shift of the curve,
while a lower quantity could only move a point lower down along the existing curve.
182. (C) The truck manufacturer absorbs a percentage of the tax increase equal to
–PED/(PES - PED). In this example, the percentage is 1.0 (1.5 - –1.0), or 1.0/2.5,
which is equal to 0.4. Because 0.4 × $10,000 = $4,000, Choice (C) is correct.
183. (C) The best answer is (C), reduce wages. If supply is inelastic and demand is elastic,
a manufacturer can’t force buyers to pay the tax by raising prices. The manufacturer
would have to absorb the tax hike by reducing other expenses, such as wages.
184. (E) The correct choice is (E). As this is movement along the same supply curve,
the only factor that could have caused this decrease is a change in the price of the prod-
uct, which also causes a shift in the quantity supplied.
185. (D) The correct choice is (D). This is the only option that would actually cause
an increased quantity, as (A) and (B) would only cause shifts along the existing curve.
On the other hand, (C) and (E) would shift the supply curve to the left.
186. (A) The correct choice is (A). While the remaining elements mentioned in the
scenario are all integral elements of a perfectly competitive market, one of the main
assumptions of this market structure is that all companies produce identical products.
187. (D) The correct choice is (D). A perfectly competitive market is one where
the price would be determined by a fair market equilibrium. This is shown only in
market D.

AP Microeconomics_p001-224.indd 180 22/12/20 5:00 PM


Answers ❮ 181

Chapter 3
188. (A) Choice (A) is correct. Economies of scale refers to the downward part of the
long-​run average total cost (LRAC) curve where LRAC falls as plant size rises; constant
returns to scale refers to changes in output by the same proportion as a change in
input; and diseconomies of scale refers to the upward sloping part of the LRAC where
firms see an increase in marginal cost as output increases. Choice (A) is the best answer
because it reflects these definitions.
189. (E) The correct choice is (E). All of these answer choices are considered by
accounting profit.
190. (C) The correct choice is (C). The third worker can generate a marginal return of
11 additional chairs.
191. (D) The correct choice is (D). The final answer can be received by deducting all
of the values from revenue aside from the cost of accepting another project. This is an
example of opportunity costs and is not considered by accounting profit.
192. (D) The correct choice is (D). Variable costs are those that change as output
increases. As rent is a fixed cost, the answer is $5,000 multiplied by five cars, which
equals $25,000.
193. (A) Choice (A) is correct. The law of diminishing returns states that as more and
more units are added to production, there is a point where the marginal revenue prod-
uct declines. In the short run, marginal cost is low, but as more and more is produced,
marginal cost increases.
194. (C) Choices (A), (B), (D), and (E) are all variable production inputs because they
can easily be influenced and changed in the short run. Choice (C) is the best answer
because human capital refers to the skills and education a worker brings to the produc-
tion of a product. This is not as easily manipulated by producers in a firm.
195. (C) Choice (C) is correct. This question is very straightforward. If a variable pro-
duction input—an input that is not a part of fixed costs—increases, then the marginal
cost will increase as well. For example, suppose a plant adds a new machine that uses
more electricity than the other machines in the plant. This is an increase in the factors
of production, and the firm will respond by increasing the price of the product to cover
the additional cost in electricity.
196. (A) Choice (A) is correct. Economic profit is the difference between total rev-
enue and total economic cost. Economic profit incorporates opportunity costs, whereas
accounting profit does not.
197. (D) Choice (D) is correct. Diseconomies of scale exist on the upward sloping
part of the LRAC curve. In the long run, firms see increasing costs as production
increases.

AP Microeconomics_p001-224.indd 181 22/12/20 5:00 PM


182 ❯ Answers

198. (E) Choice (E) is the best answer. Constant returns to scale refers to changes in
output by the same proportion as a change in input. It is the point between economies
of scale and diseconomies of scale.
199. (B) Choice (B) is the best answer. Economies of scale refers to an increase in the
efficiency of a production as production levels increase. Choice (A) is incorrect because
specialization usually does not result in a decrease in profits. Choice (C) is incorrect
because specialization coincides with a decrease in production costs. Choice (D) is
incorrect because both changes in input are the same as a change in output, so it tends
to be a straight line on the LRAC curve. Choice (E) is incorrect because diseconomies
of scale refers to an increase in marginal cost when production increases.
200. (A) The correct choice is (A). The company can continue operating, as its mar-
ginal revenue is $3 higher than its marginal costs.
201. (A) Choice (A) is correct. In general, economists say that firms do not go out of
business in the short run. They must pay their fixed costs. However, firms may shut
down and not pay their variable costs. If Mr. Ray is considering shutting down his
business, then the two most important factors he should analyze are average variable
cost (AVC) and marginal revenue (MR). The firm will remain open if MR = AVC or
MR > AVC. If marginal revenue, the same as price, is equal to average variable costs,
then firms could shut down and pay only their fixed costs. If MR < AVC, then firms
may shut down.
202. (B) Choice (B) is correct. At the point where price is greater than average total
costs, more firms will enter the market because of the attractiveness of the business
venture. This is so because both fixed and variable costs are being covered by the price.
As a result, prices will go down as more and more firms enter the market. Remember,
in a perfectly competitive market, economic profit in the long run is zero.
203. (E) Choice (E) is correct. If an owner of a firm is trying to decide whether to shut
down, then analyzing marginal revenue (MR) and average variable cost (AVC) will
help. The firm will stay open if MR = AVC or MR > AVC. If marginal revenue, the
same as price, is equal to average variable costs, then firms could shut down and pay
only their fixed costs. If MR < AVC, then firms may shut down.
204. (A) Choice (A) is correct. In any competitive market, the profit-​maximizing point
is where MR = MC. If MR is greater than MC, then the firm would make a profit.
If MC is greater than MR, then the firm would be losing money.
205. (D) A perfectly competitive firm in the long run shows a profit of zero after
more and more firms enter the market. Choice (D) is the best choice because as more
and more firms enter the market, and existing markets adjust their size, the price will
decrease and all firms will only be making a normal profit.
206. (D) If new firms enter a market, the firms already competing in the market will
lose market power, so Choice (D) is the best answer.

AP Microeconomics_p001-224.indd 182 22/12/20 5:00 PM


Answers ❮ 183

207. (A) Choice (A) is correct. A perfect competition is a market structure where many
firms are producing very similar products, prices are established through supply and
demand, and there is freedom to enter and exit the market.
208. (E) The correct choice is (E). When average variable costs is equal to marginal
revenue, the most profitable option for the company is to cease operations.
209. (C) Choice (C) is correct. The goal of any firm is to make a profit with the products
it is producing and selling. In a perfectly competitive market, there are no barriers to
entry into the market. When a product is profitable for a perfectly competitive firm,
it exists only in the short run. In the long run, profits are zero because of other firms
entering the market.
210. (C) If more and more firms are exiting the market, then costs must be greater than
price. Choice (C) is the best answer because it shows that price is less than average total
cost (ATC) in a perfectly competitive market structure.
211. (B) When most people say the word profit, they are referring to accounting profit,
calculated as the difference between total revenue and total explicit cost. Economic
profit, however, calculates the opportunity cost (implicit cost). Choice (B) is the best
answer.
212. (B) Choice (B) is the best answer because the diagram illustrates price equal to
marginal revenue. This is true in a perfect competition.
213. (A) Choice (A) is the best answer because at any other level the firm runs the pos-
sibility of producing too much or too little, which would raise revenue or incur a cost.
214. (E) Choice (E) is correct. Remember that for all market structures, the profit-​
maximizing point is where MR = MC.
215. (A) The correct choice is (A). All of the other claims are not accurate. Monopo-
listic competition and perfect competition are structures with a significant number of
competitors, but companies in a monopolistic competition have some level of market
power.
216. (A) Choice (A) is correct. A normal profit is described as another way of saying
a firm is earning an economic profit of zero. It earned the same amount that it would
earn from the next best use of its resources, so it also earned an economic profit. This is
because normal profit and economic profit consider opportunity costs.
217. (C) Choice (C) is the best answer because if producing a product is profitable,
more and more firms will enter the market because of the ease of entry. Remember,
in the long run, the economic profits of a perfectly competitive firm will be zero.
218. (E) Choice (E) is the best answer because MR = MC represents the profit-​
maximizing point for all market structures. If a firm is operating at this point, then all
resources are being utilized efficiently.

AP Microeconomics_p001-224.indd 183 22/12/20 5:00 PM


184 ❯ Answers

219. (E) Choice (E) is the best answer because it describes price leadership, where one
firm is a leader among its competitors and sets the market price, and others will follow
along because they wish to hold on to their market share. This is very true for oligopolies.
220. (A) Choice (A) is correct. Remember, the profit-​maximizing point will always be
MR = MC for any market structure, even a cartel.
221. (A) Choice (A) is the best answer because a restriction to how many firms enter the
market would allow a greater opportunity for profit. Choices (B) and (E) are incorrect
because the opposite is true because of barriers to entry. Choice (C) is incorrect because
price discrimination is unrelated. Choice (D) is incorrect because it is unrelated.
222. (B) Remember that a perfectly competitive firm in the long run will have eco-
nomic profits of zero. Normal profit will equal economic profit.
223. (C) The correct choice is (C). This is the only answer choice that reflects the
concept of economies of scale. One of the reasons this can happen is that the company
can specialize its workers or integrate more advanced technology to boost production.
224. (D) Choice (D) is correct. In any market structure, the profit-​maximizing point is
where marginal revenue equals marginal cost. In a perfectly competitive market, price
will equal marginal revenue. Choice (A) is incorrect, although deceiving, because only
in the long run will a perfect competition have a zero profit. This is because more and
more firms enter the market, cutting price and profit. Choice (B) is incorrect because
the question shows that the firm is profit maximizing and would result in more firms
entering, not leaving, the market. Choice (C) is incorrect because this is a business
tactic of an oligopoly.
225. (A) In any market structure the profit-​maximizing point is where marginal rev-
enue equals marginal cost. Choice (B) is incorrect because if the revenue product of
labor (MRP) is less than the wages paid to workers, then the firm will respond by laying
off workers. Choice (C) is incorrect because average fixed costs are costs that must be
paid regardless if the firm is making a profit or losing money. Choice (D) is incorrect
because economic decisions are not followed through if the marginal cost is greater
than the marginal benefit.
226. (A) Choice (A) is correct. Rubber is a variable production input, not a capital
good included in fixed costs, so a change in its price doesn’t affect fixed costs.
227. (C) Choice (C) is correct. Joe is earning an accounting profit of $40,000 but
would earn $50,000 as an engineer, so he incurs an economic loss of $10,000.
228. (C) Choice (C) is the best answer. In the second year, the shoe store is earning
zero economic profit because Joe’s profit as an owner equals the wage he would earn at
a company. Thus, the market has likely reached long-​run equilibrium.
229. (B) Choice (B) is correct. The furniture company achieved economies of scale
by opening up the second plant because its first plant became more profitable as well.

AP Microeconomics_p001-224.indd 184 22/12/20 5:00 PM


Answers ❮ 185

230. (C) Choice (C) is the best answer. The auto manufacturer earns $300,000
from three plants, so each plant produces $100,000 in profit. Its first plant was
producing $100,000 in profit by itself, so the auto manufacturer achieved constant
returns to scale.
231. (B) Choice (C) is correct. Competition ensures that prices will be controlled
through the market forces of supply and demand. If a monopoly exists, that firm may
set a much higher price than in a perfectly competitive market. If competition exists,
then it will increase the total welfare and benefit society.
232. (A) Choice (A) is the best answer. The auto parts manufacturer achieved economies
of scale. It also achieved increasing returns to scale, so Choices (D) and (E) are incorrect.
233. (C) Choice (C) is correct. Economic profit is equal to accounting profit minus
the profit that would be earned if resources (in this case, the owner’s skills) were put to
the next best possible use.
234. (A) Choice (A) is correct. The next best possible use of Nate’s skills pays $80,000
so opening a restaurant that earns an economic profit of $60,000 results in an economic
profit of –$20,000, or an economic loss.
235. (C) The correct choice is (C). As per-unit costs consistently increased, it is clear
that the company was becoming less effective in producing the good, meaning this is
an example of diseconomies of scale.
236. (A) Choice (A) is the best answer. If P < ATC, firms won’t earn a profit. A per-
fectly competitive market has low exit barriers, so firms will exit the market and the
price will rise.
237. (D) The correct choice is (D). This would be an example of a monopsony, as the
teachers do not really have other realistic opportunities (aside from those that were
eliminated by the question prompt) to find other sources of income. As a single entity,
the local government has strong control over this aspect of the market, so this would
be a monopsony.
238. (D) Choice (D) is correct. The difference between market price ($40,000) and the
price the automaker will accept for the car ($30,000) is $10,000.
239. (E) Choice (E) is correct. Consumer surplus is the price the consumer is willing to
pay for a banana ($2.60) minus the market price of a banana ($2.25) which is $0.35.
240. (A) Choice (A) is correct. For the raspberry farm, MR = MC at $5. Thus, it should
increase production to that output level to maximize profit. It should not increase
production until MC equals the market price of $6 because MC would rise above MR
if that were to happen.

AP Microeconomics_p001-224.indd 185 22/12/20 5:00 PM


186 ❯ Answers

241. (C) The correct choice is (C). Specialization would help minimize production
costs. There is no guarantee that just changing the quantity of employees would have
any impact, and a lump-sum subsidy would not make changes in the long run if not
properly implemented.
242. (E) The correct choice is (E). As an increase in the value of money invested always
is consistently 50 percent, this corresponds to a 50 percent increase in investment. This
is an example of a consistent return to scale.
243. (C) The correct choice is (C). As it can be clearly calculated that an increase always
results in a 20 percent increase, increasing investments from $1,200 to $1,400 would
result in an increase in the number of wines produced of 20 percent. The value of the
increase is 2.4, meaning that the total number of wines produced would be 14.4.
244. (D) The correct choice is (D). Deadweight costs occur when the price is kept
artificially high through the monopolist or a cartel setting the price of the product. The
difference between the price set by either the monopolist or the oligopoly compared
with the market price is the deadweight cost for each of these markets.
245. (A) The correct choice is (A). The minimum efficient scale, often abbreviated as
MES, is considered to be the lowest level on the cost curve on which the company can
produce a product at a competitive price.
246. (A) The correct choice is (A). At the level of one production plant, the company
can earn $20,000 times 100, which equals $2,000,000. When deducting the cost of
opening the plant from this value, its profit is still $500,000. Despite the fact that the
company could theoretically make more cars with a rising number of plants, the costs
outweigh the benefits. For example, with five production plants, the company would
earn $4,200,000, but it would cost the company $6,000,000 just to open that many
plants, so it would be operating with a loss of $1,800,000.
247. (B) The correct choice is (B). When the production function is marked by dimin-
ishing returns, the marginal cost curve will be upward sloping.
248. (C) The correct choice is (C). When you are calculating the levels of profit for each
worker, consider that the profit the company can earn with one worker is 20 × 80 - 800,
which equals $800. The corresponding levels of profit, using the same procedure as just
described, are $600 for two workers, $1,600 for three workers, $3,000 for four workers,
and $4,800 for five workers. Given these facts, it is clear that the least the company can
make is if it hires two workers, at which point it will only earn $600, as reflected in (C).
249. (B) The correct choice is (B). This is the only accurate response, as this is a
defining characteristic of a perfectly competitive market in which there are many com-
petitors and low barriers to entry. This results in no companies’ making an economic
profit in the long run.

AP Microeconomics_p001-224.indd 186 22/12/20 5:00 PM


Answers ❮ 187

250. (A) The correct choice is (A). The law of diminishing returns shows that the mar-
ginal use of each additional worker decreases over time, so an increase of less than three
(the marginal return preceding it) would be ideal.
251. (A) The correct choice is (A). Economic profit is gained by deducting all of the
costs, including implicit costs. This means that the costs of financial capital and oppor-
tunity would also be considered. When deducting all of these values from revenue, you
will find that the final result is $19,000. (B), $30,000, is the accounting profit of the
company, which does not take into account the implicit costs of doing business.
252. (E) The correct choice is (E). This is the most accurate response, as perfect com-
petition is generally characterized by companies being unable to make profit in the
long run, while a few companies working together to keep the price high is an essential
trait of oligopolies.
253. (E) The correct choice is (E). Because a perfect competition would lead to a large
number of companies competing, this would likely drive down the price of the product.
254. (C) The correct choice is (C). Companies primarily make such decisions by con-
sidering whether they are making an economic profit, as this accounts for both their
implicit and explicit costs. If a company is making an economic loss, it is likely that it
may decide to shut down its operations.
255. (D) The correct choice is (D). In a perfectly competitive market, there are no
barriers to entry and companies are price takers who have to accept the market price.
256. (C) The correct choice is (C). Neither revenue nor accounting profit are actually
relevant to showing whether any of these market structures are a perfectly competitive
market. The key element is that economic profit is $0 for market C, which is what
companies make in the long run in a perfectly competitive market.
257. (C) The correct choice is (C). Colluding to keep the price higher than fair market
conditions is a trait of an oligopoly, and this is specifically not possible in a market that
is perfectly competitive.
258. (B) The correct choice is (B). Economic profit can be calculated by considering
revenue and then subtracting all of the implicit and explicit costs of doing business.
Failing to sign a contract with a client and the cost of financial capital are implicit costs
of doing business, so while the accounting profit is $40,500, economic profit is equal
to $34,000.
259. (B) The correct choice is (B). The owner losing time filling his tax returns is an
implicit cost of doing business that is measured by economic profit by economic profit
but not by accounting profit. Therefore, accounting profit would be worth $2,500
more than economic profit.

AP Microeconomics_p001-224.indd 187 22/12/20 5:00 PM


188 ❯ Answers

260. (A) The correct choice is (A). Prior to the companies’ implementing these changes,
its per-unit costs of production were decreasing as it opened new plants. The scenario
makes it clear that overcrowding by adding redundant production lines caused an
increase in the per-unit cost, which can be characterized as a decreasing return to scale.
261. (C) The correct choice is (C). Labor costs include the costs of both full-time and
part-time employees, so Jim’s total labor costs are $24,000. When dividing this value
by 3, you find that the result is that Jim’s marketing costs are $8,000.
262. (C) The correct choice is (C). All of the costs mentioned in the table, aside from
the cost of Jim not paying himself as the owner a salary, are explicit costs and should be
deducted from the total value of the revenue. By including all of these costs, the total
accounting profit will equal $33,000.
263. (A) The correct choice is (A). Because the wine manufacturer is the only buyer,
this would make this type of market structure a monopsony.
264. (C) Choice (C) is correct. The apple farm would lose its customers if it raised
the price of apples because its apples are a commodity. The apple farm can sell all of
its apples, so cutting the price wouldn’t improve sales; MC should already equal MR.
A unique apple cultivar would be a differentiated product, resulting in monopolistic
competition instead of perfect competition, which would give the apple firm an oppor-
tunity to earn an economic profit.
265. (D) Choice (D) is correct. The equilibrium price and quantity can be calculated
with algebra. The quantity supplied will equal quantity demanded at equilibrium, and
the supply price will equal the demand price. Let 10 + 4Q = 80 – 3Q, then 10 = Q.
P = 10 + (4 × 10) = 50.
266. (B) Choice (B) is correct. Let 80 + 6Q = 400 – 10Q. Then 16Q = 320, so Q = 20.
P = 80 + (6 × 20) = 200.
267. (D) Choice (D) is correct. Lower supply and lower demand will reduce equilib-
rium quantity, but the change in market price can’t be determined.
268. (B) Choice (B) is correct. The price of an input fell, which will increase supply.
Demand also fell. This will result in a lower market price, but the change in equilibrium
quantity can’t be determined.
269. (C) Choice (C) is correct. The equilibrium price and quantity can be calculated
with algebra. The quantity supplied will equal quantity demanded at equilibrium,
and the supply price will equal the demand price. Let 575 + 8Q = 920 – 15Q, then
345 = 23Q, so Q = 15. P = 575 + (8 × 15) = 695.
270. (A) Let 1,000 + 3Q = 5,000 – 5Q. Then 8Q = 4,000, so Q = 50. P = 1,000 +
(3 × 50) = 1,150.

AP Microeconomics_p001-224.indd 188 22/12/20 5:00 PM


Answers ❮ 189

271. (A) Choice (A) is correct. If the supply curve for apples shifted to the right and
the demand curve for apples shifted to the left, the equilibrium price for apples would
decrease.
272. (D) Choice (D) is correct. If the supply curve for pears shifted to the right and
the demand curve for pears shifted to the right as well, the equilibrium quantity of
pears would increase.
273. (A) Choice (A) is correct. If an effective price floor was established for grapes,
quantity supplied would be more than quantity demanded.
274. (B) The correct choice is (B). This is the only answer that contains both of the
accurate values. Fixed costs are $65,000, while variable costs are $750 × 100, which
equals $75,000. This means that total costs are $140,000.
275. (D) The correct choice is (D). When several companies work to set the market
price, this is evidence of an oligopoly. When a market has only one buyer, by definition
this is an example of a monopsony.
276. (E) The correct choice is (E). 0.1 percent of $200,000 is $200. Marginal revenue
can best be defined as the value of an additional unit of vaccine that Floyd Pharma-
ceuticals sells.

Chapter 4
277. (C) Choice (C) is correct. In a monopolistic competition, product differentiation
is utilized to make a product stand out from its competitors. For example, there are
many similarities between iPhone and Galaxy smartphones, but there are very subtle
differences that appeal to different consumers.
278. (A) Choice (A) is the best answer. Regulators may require average cost pricing in
exchange for granting a natural monopoly to a firm.
279. (B) Choice (B) is correct. A monopolistic competition refers to a market structure
with many firms producing a differentiated product with easy entry into the market.
Choice (A) is wrong because it refers to a natural monopoly. Choice (C) is incorrect
because it refers to an oligopoly. Choice (D) is incorrect because it refers to a perfect
competition. Choice (E) is incorrect because it refers to a monopoly.
280. (C) Choice (C) is correct. An oligopoly is a market structure in which there are a
small number of interdependent firms producing either a standardized or differentiated
product with barriers to entry. Firms in an oligopoly have significant pricing power.
Choice (A) is incorrect because it refers to a natural monopoly. Choice (B) is incorrect
because it refers to a monopoly. Choice (D) is incorrect because it refers to a monopo-
listic competition. Choice (E) is incorrect because it refers to a perfect competition.

AP Microeconomics_p001-224.indd 189 22/12/20 5:00 PM


190 ❯ Answers

281. (D) Choice (D) is the best answer. Monopolistic competition allows firms to earn
short-​run economic profits but not long-​run economic profits.
282. (C) Choice (C) is correct. The movie theater is using price discrimination to
maximize profit by setting different ticket prices for two groups of customers.
283. (A) Choice (A) is correct. A monopoly is a market structure in which one firm is
the producer for a good or service with very few substitutes and many barriers to entry.
There is virtually no competition in a monopoly market structure.
284. (E) The correct choice is (E). When there is an oligopoly, there is usually some
coordination between the few companies to maintain high prices.
285. (D) The correct choice is (D). Any monopoly that occurs through market-allowed
means is an example of a natural monopoly.
286. (D) If the price of a good increases, consumers lose purchasing power. If the good
is a significant part of a consumer’s income or budget, then a change in price will be
greatly felt. Choice (D) is the best answer because purchasing fruit does not make up
a significant percentage of a person’s income or budget as compared with buying and
maintaining a boat.
287. (B) Choice (B) is the best answer because a school can hire teaching assistants very
easily in the short run. Long-​run adjustments are major and significant changes to
a firm, such as plant size and investing in human capital.
288. (C) Choice (C) is correct. A monopoly can earn an economic profit in the long
term because other firms cannot enter the market. In monopolistic competition, firms
can easily enter the market so profit will be normal in the long term.
289. (C) Choice (C) is correct. Since a monopoly is the sole producer of a product,
the firm will charge an increased price as compared with a perfect competition, where
price is established through supply and demand. Remember, in a monopoly, there is
no competition, so firms will always charge a higher price.
290. (E) Choice (E) is the best answer because a perfect competition is a market
structure where many firms are producing very similar products, prices are established
through supply and demand, and there is freedom to enter and exit the market.
291. (A) Choice (A) is correct. There are many different types of smartphones on the
market produced by many different firms. Although they are similar in many ways
(i.e., you can make a phone call, send a text, and surf the Internet), product differenti-
ation offers variety. Remember, if you see product differentiation, then a monopolistic
competition or an oligopoly is being discussed.
292. (D) Choice (D) is correct. A natural monopoly is a market structure where it is
beneficial for one firm to control the production of a good or service. For example,
natural gas and the water authority work most efficiently and with the greatest benefit if
there is a natural monopoly. It would be too complicated and at a great cost if competi-
tive markets began running separate lines to households for gas and water.

AP Microeconomics_p001-224.indd 190 22/12/20 5:00 PM


Answers ❮ 191

293. (C) Choice (C) is the best answer because tobacco is a cost of production for
cigarettes. If the price increases for a cost of production, then firms must increase their
price to keep making a profit on producing the good.
294. (E) The correct choice is (E). All of the other answer choices are not accurate for
monopolistic competition. (B) and (D) would be true for monopolies.
295. (A) Choice (A) is correct. Remember that a monopolist is the only producer of a
good or service and has no competition. Therefore, the firm may raise the price higher
than marginal costs. All firms will maximize profit where marginal revenue (MR) equals
marginal cost (MC), but a monopolist will support a higher price than the MC.
296. (B) Choice (B) is correct. An oligopoly and a monopoly have barriers to entry
into the market. A perfect competition and a monopolistic competition are market
structures that have ease of entry into the market.
297. (C) Choice (C) is the best answer because allocative efficiency is achieved if
price (P) equals marginal cost (MC). Choice (B) is incorrect because the government
would not allow P to be greater than MC. Choices (A) and (D) represent allocative
inefficiency.
298. (B) Choice (B) is correct. Price discrimination refers to a systematic pricing system
that charges different prices to different groups of people. However, the firm will still
ensure that the maximum price that these different people are willing to pay is utilized.
299. (A) Choice (A) is correct. Price discrimination allows firms to receive every dol-
lar available from their customers. If a firm can successfully identify specific groups of
people and ensure that customers do not resell their product to other consumers, then
price discrimination would be successful. An example would be selling movie tickets
in a theater. There are different prices for shows at different times, and different prices
for different types of customers: adult, child, and senior citizen.
300. (D) Choice (D) is correct. Even though company Z does not own all the farm-
land used in the production of tobacco, it owns a significant majority. It now controls
most of the farmland used in the production of tobacco. This shows that company Z
is attempting to establish a monopoly by controlling one of the factors of production
(land). Choice (A) is incorrect because all businesses wish to increase profits, and it does
not fully answer the question. Choice (B) is incorrect because an oligopoly refers to a
small number of very large firms; this question refers only to company Z. Choice (C)
is incorrect because even though it is correct that the profit-​maximizing point is where
MR = MC, it does not relate to the scenario in the question.
301. (C) Choice (C) is correct. You should be aware of the graphical representation of
profit for all market structures. This graph represents the profit of a monopolistically
competitive firm. The shaded area represents profit because the price level is more than
the average total cost (ATC). Since the ATC represents the total cost divided by the
output, if the price is higher than this number, then the firm is making a profit.

AP Microeconomics_p001-224.indd 191 22/12/20 5:00 PM


192 ❯ Answers

302. (C) Choice (C) is correct. With no entry or exit barriers, firms will enter the market
under monopolistic competition until all firms are earning a normal economic profit.
303. (D) Choice (D) is correct. Game theory examines the relationships between par-
ticipants in a specific model structure and attempts to predict their actions. This is
useful for studying oligopolies because they consist of very large firms that are inter-
dependent companies, producing either a standardized or differentiated product with
barriers to entry.
304. (D) Price leadership refers to a firm that is a leader among its competitors and sets
the market price; others will follow along because they wish to hold on to their market
share. Choice (D) is the best answer because an oligopoly fits this description.
305. (C) Choice (C) is the best answer because if something is implicit, then it is not
blatantly seen outright. An opportunity cost is an excellent example of an implicit
cost. Choice (A) is incorrect because it describes explicit costs. Choice (B) is incorrect
because it describes total variable costs. Choice (D) is incorrect because it describes
average variable costs.
306. (B) Choice (B) is correct. A cartel, which is a form of an oligopoly, is a market
structure where there are a small number of interdependent firms, producing either a
standardized or differentiated product with barriers to entry. Since these firms work
together to establish prices and control supply and demand, there is always an incentive
for a firm to cheat to earn a greater profit than its competitors.
307. (A) Choice (A) is correct. Deadweight loss refers to the lost net benefit to society
caused by a movement from competitive market equilibrium. In other words, the
people who would purchase the good who have more marginal benefit than marginal
price are not purchasing the product. For a monopoly, this occurs when price is greater
than marginal cost.
308. (E) Choice (E) is the best answer because P = MR = MC = ATC represents a
perfectly competitive firm.
309. (A) Choice (A) is correct. Price taking refers to firms that establish their price
through the market forces of supply and demand. Since oligopolies are very large firms
and work together to control supply and demand, they are price setters.
310. (D) The correct choice is (D). As the company is making a profit in the long run
and the barriers to entering the market are high, this must mean that it is an example of
imperfect competition. An atomistic market is synonymous with perfect competition,
so the remaining answer choices are not accurate.
311. (D) Choice (D) is correct. This graph illustrates the total revenue curve and also
provides information about changes in marginal revenue.
312. (A) Choice (A) is correct. Average total cost (ATC) is the total cost divided by the
output. If the ATC equals price (P), then firms will make an economic profit of zero.

AP Microeconomics_p001-224.indd 192 22/12/20 5:00 PM


Answers ❮ 193

313. (C) Choice (C) is correct. Market power refers to the ability of a firm to set prices
above the perfectly competitive level. A perfectly competitive firm does not have market
power because prices are determined through supply and demand. Therefore, a perfect
competition is a price taker. A monopoly market structure fits perfectly with the idea
of market power because as the only producer of the good, it increases prices above
where MR = MC.
314. (B) Choice (B) is correct. When monopolistic competition exists, firms sell differ-
entiated products. Since these products aren’t complete replacements for one another,
the firms have some market power.
315. (A) Choice (A) is correct. A patent is a grant given by the government allowing
the patent holder the sole right to produce and sell a product for a set period of time.
If a producer is the only producer of a product, then it is a monopoly.
316. (B) Choice (B) is correct. Economies of scale refers to a decrease in ATC as quan-
tity increases. Choice (A) is incorrect because it describes diseconomies of scale. Choice
(C) is incorrect because if quantity decreases, ATC will not necessarily decrease. Choice
(D) is incorrect because it describes the profit-​maximizing point and is unrelated to
economies of scale in the short run. Choice (E) is incorrect because it refers to elasticity.
317. (D) The correct choice is (D). The soda manufacturer set the price of a soda can at
$5 and limited quantity to 200, while the equilibrium price would be $2. So the soda
manufacturer collects an economic profit of $600 by limiting quantity.
318. (B) The correct choice is (B). Choice (C) is the soda manufacturer’s economic
profit.
319. (C) Choice (C) is correct. The shift would indicate that the soda manufacturer
lost its monopoly.
320. (A) This market is an oligopoly. Few manufacturers and long-​run economic prof-
its are the distinguishing features of this market. Product differentiation could also
indicate monopolistic competition and does not always exist in an oligopoly.
321. (A) Choice (A) is correct. A group of firms in an oligopoly may form a cartel to
set the price for their goods, which is collusion. Because these firms set the price, they
have market power. This is not an example of a monopoly, because more than one oil
company is in the market.
322. (A) Choice (A) is the best answer. There are more apple farms so substitutes for
any farm’s apples are more widely available, thus demand should be more price elastic.
323. (B) Choice (B) is the best answer. This market can be described as a perfect com-
petition. While demand is likely downward sloping for the market as a whole, there are
many peanut farms selling undifferentiated peanuts so an individual farm’s decisions
don’t affect the market price. Thus, demand for an individual peanut farm is perfectly
elastic.

AP Microeconomics_p001-224.indd 193 22/12/20 5:00 PM


194 ❯ Answers

324. (E) Choice (E) is the best answer. To reduce costs, a firm can shut down produc-
tion in the short run, lay off workers, or reduce its raw material purchases. Refinancing
corporate bonds would change the firm’s capital structure, which is considered a long-​
run decision.
325. (B) Choice (B) is the best answer. If a good increases in price, a complementary
good will also increase in price.
326. (D) The correct choice is (D). On the left side of the cost curve, the toy manufac-
turer’s average cost decreases as production rises because fixed cost is applied to a greater
production quantity. When the toy manufacturer’s production quantity increases
beyond the trough of the cost curve, diminishing marginal returns have a larger impact
on average cost than the decrease in fixed cost and average cost rises.
327. (A) Choice (A) is the best answer because firms in an oligopoly consider other
firms’ responses to business decisions, while monopolistic competition includes many
small firms whose decisions don’t greatly impact one another.
328. (C) Choice (C) is correct. The auto manufacturer’s next best choice is shutting
down for an accounting profit of –$40,000. If it produces tires, its accounting profit is
–$10,000. Thus, it earns an economic profit of $30,000.
329. (B) Choice (B) is correct. The cable companies have an oligopoly and may be
colluding to control the market.
330. (C) Choice (C) is the best answer. When there are many sellers and few buyers,
the market is an oligopsony.
331. (B) Choice (B) is the best answer. The steel manufacturer is a monopsony.
332. (B) Choice (B) is the best answer. Charging different prices to viewers in different
countries is price discrimination.
333. (A) Choice (A) is correct. Firm A would earn a profit of $25 if it lowered prices
and firm B raised prices. Firm B would then lose $5.
334. (A) Choice (A) is correct. If firms A and B both raise prices, each firm earns a
profit of $10.
335. (D) Choice (D) is the best answer. In an oligopoly, a firm may use a payoff matrix
to consider other firms’ decisions when setting prices.
336. (A) Choice (A) is the best answer. Defecting always results in a higher payoff than
cooperating, so a firm always has an incentive to defect. Thus, the Nash equilibrium is
for both firms to defect.
337. (C) Choice (C) is the best answer. An individual oil producer can increase its
profit by increasing quantity supplied because the cartel is supplying less oil than
the market is willing to buy. Thus, the oil producer has an incentive to defect from the

AP Microeconomics_p001-224.indd 194 22/12/20 5:00 PM


Answers ❮ 195

cartel, although the cartel may respond by ejecting the oil producer for violating
the agreement.
338. (D) Choice (D) is the best answer. Buying ads has a payoff of $70 or $40,
while not buying ads has a payoff of $50 or $30, so the dominant strategy for Joey’s
is buying ads.
339. (A) Choice (A) is correct. Joey’s and Betty’s both earn $40 if they each buy ads.
They would each earn $50 if neither restaurant bought ads.
340. (B) Choice (B) is correct. The auto manufacturer can’t produce car windows if
it produces car doors, so the money it would have earned by selling car windows is an
implicit cost.
341. (E) The correct choice is (E). As this is a small number of manufacturers that work
together to maintain a price that would be higher than if there were regular market
conditions, this is an example of an oligopoly.
342. (C) The correct choice is (C). A monopoly is characterized by a downward slope
of the demand curve.
343. (A) The correct choice is (A). This is the only accurate answer and a trait of both
monopolies and oligopolies. (B), (C), and (D) are only correct for monopolies, while
(E) is not accurate for either of these market structures.
344. (C) The correct choice is (C). Because there is a limited number of producers, the
main reason why they would collude is to raise the market costs.
345. (D) The correct choice is (D). Because a monopolist can simply set the price, as
it is the only producer of that good, that company is a price setter. On the other hand,
in a perfectly competitive market, companies are price takers, so the price would be
$200 for A and $50 for B.
346. (B) The best answer is choice (B). Acquiring a competitor does not necessarily
create a monopoly or an oligopoly, although it is a step in that direction. A sole propri-
etorship or a partnership may acquire a direct competitor.
347. (B) Choice (B) is the best answer. The question describes the orange farm industry
as a market with perfect competition. Thus, the orange farms have low pricing power
(they are price takers).
348. (B) The market for tax software is an oligopoly. Barriers to entry are already
high, and the tax software companies are already price makers. Oligopolies typically
don’t involve price competition. Buying the other firm would create a monopoly,
and regulators could block the acquisition on antitrust grounds. When a market is
an oligopoly, firms often respond to competitors’ business decisions, so Choice (B) is
the best answer.

AP Microeconomics_p001-224.indd 195 22/12/20 5:00 PM


196 ❯ Answers

349. (B) Choice (B) is the best answer. With many stores and unique products, the
clothing market can be described as monopolistic competition, so clothing stores are
likely to spend much of their revenue on ads.
350. (C) Choice (C) is the best answer. By granting the water company a natural
monopoly, water can be delivered efficiently without incurring excess infrastructure
costs. Regulators often control pricing policy for firms with natural monopolies.
351. (C) The correct choice is (C). Because both monopolies and oligopolies reduce
supply and, in the case of oligopolies, work together to keep the price higher, there
are deadweight costs for consumers, making them examples of ineffective market
structures.
352. (D) The correct choice is (D). Heavy advertisement, a significant number of com-
petitors, and similar (but not identical) products are all traits of a monopolistic market.
353. (E) The correct choice is (E). By definition, companies are price setters in both
market structures. Because oligopolies tend to collude to set a higher price, one
company’s lowering the price can trigger a price war. On the other hand, this is not
possible in a monopolistic market, as there is only one firm in the industry.
354. (B) The correct choice is (B). Because there are numerous companies in the indus-
try, buyers are very responsive to a change in the price.

Chapter 5
355. (C) Choice (C) is correct. The equilibrium price is $80 and the average cost is
$60, so the firm earns $20 in economic profit for each golf club. If it sells 100 golf
clubs, it earns $2,000.
356. (A) Choice (A) is the best answer. Lower consumer interest would reduce the
equilibrium price for golf clubs but wouldn’t increase production costs. An equipment
breakdown would result in higher production costs but wouldn’t reduce the equilibrium
price. Increased competition over time explains both the increase in manufacturing costs
(raw materials would actually become more expensive because more golf club manufac-
turers would demand them) and the fall in equilibrium price (price competition among
golf club companies would result in a lower price for golf clubs).
357. (C) Choice (C) is correct. Cross-​licensing the patents gives both software compa-
nies a way to prevent other firms from entering the market.
358. (B) Choice (B) is correct. Regulators grant a drug company a monopoly through
a patent, but the patent will eventually expire. A natural monopoly held by a utility is
often granted by regulators but doesn’t end after a fixed period of time.
359. (A) Choice (A) is correct. If the pharmaceutical firm invents a new drug, it will
get a monopoly on the new drug and can keep earning an economic profit. Once the
patent on the existing drug expires, other firms (generic drug manufacturers) will enter

AP Microeconomics_p001-224.indd 196 22/12/20 5:00 PM


Answers ❮ 197

the market, so the pharmaceutical firm will lose its opportunity to make an economic
profit on that drug.
360. (A) Choice (A) is correct. Diseconomies of scale result in lower production
efficiency.
361. (D) Choice (D) is the best answer.
362. (A) Choice (A) is the best answer. The other choices would be caused by an
increase in demand for popcorn.
363. (D) Choice (D) is the best answer. Manufacturers will minimize costs by hiring
workers where the MFC of labor is the lowest.
364. (B) Choice (B) is the best answer. Job training increases worker productivity,
which increases the MRP for labor.
365. (E) Choice (E) is correct. The rancher pays $9 for the eleventh bag of feed, plus
an additional $1 for each of the first 10 bags, so MFC is $19.
366. (D) Choice (D) is the best answer. Consumers don’t demand raw materials,
a manufacturer demands raw materials so it can make finished goods for consumers.
367. (A) Choice (A) is correct. A shift from D2 to D3 for bread demand is an increase
in demand, increasing demand for wheat because it is an input for bread.
368. (A) Choice (A) is correct. Demand for wheat is a derived demand, and demand
for wheat is a direct demand because wheat is an input for bread.
369. (D) Choice (D) is correct. The supply of labor would increase and the curve
would shift to the right.
370. (C) Choice (C) is correct. Remember the determinants of supply. If there is a new
development in technology, it would potentially decrease a firm’s cost of production
and shift the curve to the right.
371. (A) Choice (A) is the best answer. MRP is $600, $400, and $200 for the first,
second, and third worker, respectively. If the MFC for labor was $400, the factory
would have two workers. If the MFC for labor was $200, the factory would hire a
third worker.
372. (B) Choice (B) is correct. Revenue for four workers is $4 × $100, or $400.
Revenue for five workers is $3 × 150, or $450. So MRP for the fifth worker is $50
and MRP = MFC at $50.
373. (C) Choice (C) is correct. Revenue would increase from $2 × 100, or $200, to
$1.50 × 200, or $300. Thus, MRP is $100.
374. (C) Choice (C) is the best answer. Because the college canceled its nursing classes,
fewer trained nurses will be available for the hospitals to hire.

AP Microeconomics_p001-224.indd 197 22/12/20 5:00 PM


198 ❯ Answers

375. (C) Choice (C) is correct. A higher onion cost would increase the cost of produc-
tion and the food processor’s labor demand curve would shift to the left.
376. (B) Choice (B) is the best answer. If the motorcycle factory can substitute machin-
ery for labor, then it needs fewer workers if machinery becomes more efficient.
377. (D) Choice (D) is the best answer. If machinery and labor are complementary
resources, more efficient machines will increase the MRP of labor so the airplane
factory will hire more workers.
378. (A) Choice (A) is correct. When demand for a product is elastic, the firm’s demand
for inputs used to produce the product will be elastic as well.
379. (A) Choice (A) is correct. If substitute inputs are available in the market, the firm’s
demand for inputs will be more elastic.
380. (A) Choice (A) is the best answer. When a production input is less important,
the firm’s demand for the input will be more elastic.
381. (B) Choice (B) is the best answer. Company B’s MP of labor declines more rapidly
because it doesn’t have unused trucks available, so company B has more elastic labor
demand.
382. (B) Choice (B) is correct. The price elasticity of labor demand is –15 percent/
45 percent, which is –0.33.
383. (A) Choice (A) is correct. The price elasticity of labor supply is 20 percent/
40 percent, which is 0.5.
384. (A) Choice (A) is correct. If the MFC of labor decreases, employment increases
because firms will demand more labor.
385. (A) Choice (A) is correct. Demand for production inputs is a derived demand
because consumers don’t purchase production inputs themselves.
386. (B) Choice (B) is correct. Consumers demand hamburgers so this is an example
of direct demand.
387. (A) Choice (A) is correct. The truck manufacturer hires production workers
because consumers demand trucks.
388. (A) Choice (A) is the best answer. The factory will cut back on sugar purchases
until MFC = MRP.
389. (B) The correct choice is (B). Adding the ninth employee allows the suit manu-
facturer to produce (24 – 20) = 4 more suits, which are worth $100 each, so marginal
revenue product is $400.
390. (C) Choice (C) is correct. Adding the sixth employee increases output by two
refrigerators, which are worth $500 each, so marginal revenue product is $1,000.

AP Microeconomics_p001-224.indd 198 22/12/20 5:00 PM


Answers ❮ 199

391. (D) Choice (D) is correct. Marginal product is –5, so marginal revenue product
is –5 × $10, or –$50.
392. (B) Choice (B) is the best answer. The firm must compare marginal revenue
product with marginal factor cost. Even if marginal revenue product increases, if MFC
is greater than MRP the firm will earn less profit if it increases output. MFC is less than
MRP in this example so the golf club factory should increase production.
393. (A) Choice (A) is correct. MRP is $125 and MR is $2.50, so MP must be 50.
394. (C) Choice (C) is the best answer. Because of derived demand, demand for wood
will increase and the price of wood will rise.
395. (A) Choice (A) is correct. Human capital refers to the skills a worker applies on
the job. Acquiring human capital increases a worker’s value in the marketplace. Human
capital is a major factor of production, and any increase in human capital through
educational training would increase the productivity of labor.
396. (B) The correct choice is (B). Marginal revenue product of labor is calculated by
multiplying marginal revenue and marginal cost. This is shown in the table and is 7 × 8,
which equals $56.
397. (E) The correct choice is (E). If marginal revenue is three times higher than marginal
cost, then it is 5 × 3, which equals $15. The marginal revenue product of labor can then
be calculated by multiplying marginal product, which is 6, with 15, and this equals $90.
398. (C) Choice (C) is correct. Choice (B) is marginal revenue product.
399. (B) Choice (B) is the best answer because of derived demand.
400. (C) Choice (C) is the best answer because tomatoes are an input for ketchup.
401. (D) Choice (D) is correct. MRP now equals MFC at $150, instead of $130, so
the hot dog truck will lay off its third and fourth workers.
402. (B) Choice (B) is correct. Labor costs would increase by ($5,000 × 7) + $30,000,
or $65,000.
403. (A) Choice (A) is correct. The price of a factor of production increased, so the
price of another unit of output increases as well.
404. (A) Choice (A) is the best answer, because MFC < MRP and the MFC for another
employee is less than the MFC for machinery.
405. (C) Choice (C) is the best answer because wheat is a factor of production for
producing bagels. If the price of a factor of production decreases, Seth can increase
production and his demand for labor will increase.
406. (B) Choice (B) is the correct answer because derived demand refers to the demand
for resources from the demand for the goods produced by the resources. Choice (A)

AP Microeconomics_p001-224.indd 199 22/12/20 5:00 PM


200 ❯ Answers

is incorrect because it refers to the demand for labor. Choice (C) is incorrect because
it illustrates the least-​cost rule. Choice (D) is incorrect because that is the profit-​
maximizing point for resource employment. Choice (E) is incorrect because it refers
to a monopsony.
407. (C) Choice (C) is correct. Wages are part of MFC so manufacturers will hire
workers where MFC is lower and lay off workers where MFC is higher.
408. (B) Choice (B) is the best answer. MRP for labor will increase in New York
because of the free education, while it will fall in Nebraska because fewer students can
afford university education.
409. (A) Derived demand refers to the demand for resources that come from the demand
for the goods produced by the resource. People have a demand not for the worker but for
the product the worker produces. Choice (A) is the best answer because the demand
for tractors and workers is derived from people’s demand for food.
410. (C) Choice (C) is the best answer because many people go to the doctor when
they catch a cold. If a cure is found for the common cold, people will have fewer doctor
visits and would decrease the demand for doctors in the long run. Every other choice
would increase the demand for doctors.
411. (A) Choice (A) is correct. The output effect states that there will be a rise in output
if the price of a resource used in the production of a product decreases, and firms will
increase production and the demand for labor will increase.
412. (B) Choice (B) is correct. Remember that the MRP curve and the labor demand
curve are similar in that they are both downward sloping. If MRP increases, so will
labor demand.
413. (B) Choice (B) is correct. Choice (A) is incorrect because it refers to the marginal
revenue cost. Choice (C) is incorrect because it refers to the least-​cost rule. Choice (D) is
incorrect because it refers to profit maximizing. Choice (E) is incorrect because it refers
to a monopsony. The marginal revenue product of labor (MRP) refers to the change in
a firm’s total revenue from hiring one additional unit of labor.
414. (D) Choice (D) is correct. The firm should lay off workers until MRP of labor
equals the wage of the last worker hired.
415. (C) Choice (C) is correct. Immigration will affect a labor supply curve because if
it decreases, it will shift that country’s labor supply curve to the left. The labor supply
curve will shift to the right for the country the labor supply enters.
416. (C) Choice (C) is correct. If the price of raw materials increases, MFC will increase.
The firm will have to lay off workers until MRP equals the MFC of labor again.
417. (A) Choice (A) is correct. Remember that complements are products that are used
with another product. For example, you cannot play tennis without a racket and tennis
balls. If the price of a good increases, so will the price of its complements. This will
result in a decrease in labor because firms will lay off workers until MRP = W.

AP Microeconomics_p001-224.indd 200 22/12/20 5:00 PM


Answers ❮ 201

418. (C) Choice (C) is the best answer because the least-​cost rule seeks to find the best
combination between resources and capital that would be the cheapest for the firm.
Choice (A) is incorrect because it refers to a monopsony and the answer does not fully
answer the question. Choice (B) is incorrect because it refers to profit maximizing.
Choice (D) is incorrect because it refers to marginal revenue cost. Choice (E) is incorrect
because it refers to the marginal revenue product.
419. (B) Choice (B) is correct. Wages are a major cost to firms. If the price of labor
increases, then firms will lay off workers to the point where MRP = MFC.
420. (B) Choice (B) is the best answer. Because diners want to eat food at the restaurant
(demand), the restaurant will demand cooking ingredients (inputs used to make food).
421. (D) The correct choice is (D). With one more unit of input (the sixth worker)
production increases by five auto parts, so MP = 5. The auto parts are worth $3, so MR
is $3. MR × MP is $3 × 5, or $15.
422. (D) Choice (D) is correct. For the tenth employee, production increases by three
ice cream cones (27 – 24 = 3) so MP = 3. The ice cream cones sell for $2, so MR is $2.
MR × MP is $2 × 3, or $6.
423. (D) Choice (D) is correct. If the fishing boat hires a seventh crew member, it
brings back five fewer fish. As a result, MP = –5 for the seventh crew member. MR is $4,
so MR × MP is $4 × –5, or –$20. The boat has limited space available, so if it hires
too many crew members they will get in each other’s way and productivity will drop.
424. (D) Choice (D) is the best answer. If the hot dog stand hired a fifth worker,
marginal factor cost would increase by $120, while marginal revenue product would
increase by 20 × $5 = $100. Thus, MFC is greater than MRP and the hot dog stand
shouldn’t hire the fifth worker.
425. (A) Choice (A) is the best answer. If graphite is used to make golf clubs and
demand for golf clubs increases, golf club manufacturers will demand more graphite
and the demand curve for graphite will shift to the right. As a result, the price of
graphite will increase.
426. (D) Choice (D) is correct. MRP is $150 and MR is $3, so MP must be 50.
427. (B) Choice (B) is correct. The surfboard manufacturer’s decision to hire a seventh
worker increases total revenue by $100, so MRP is $100. MP increased by 2, so MR is
$100 divided by 2, or $50.
428. (C) Choice (C) is the best answer. Again, if perfect competition exists, labor and
raw materials costs will remain constant as output increases because firms aren’t large
enough for their production decisions to affect supply. If the market is a monopsony
or an oligopsony, a firm will incur higher labor and raw materials costs if it increases
output.
429. (C) Choice (C) is the best answer. All of the other choices would result in higher
demand for cars.

AP Microeconomics_p001-224.indd 201 22/12/20 5:00 PM


202 ❯ Answers

430. (A) Choice (A) is correct. The demand for corn increased because corn is an input
for corn tortillas and demand for corn tortillas increased.
431. (C) Choice (C) is correct. MFC = $120 and the hot dog stand should hire workers
until MRP = MFC.
432. (B) Choice (B) is correct. MFC increased so the equilibrium point where
MRP = MFC has moved from $85 to $90.
433. (C) Choice (C) is correct. The skateboard manufacturer is a monopsony. If the
firm hired another worker, it would have to pay $50,000 plus $10,000 × 10 in addi-
tional labor costs. Thus, MFC is $150,000 and MRP is $60,000, so hiring another
employee would reduce profit.
434. (D) Choice (D) is correct. Adding another employee increases labor costs by
$40,000 + ($10,000 × 6), or $100,000, so MFC is $100,000.
435. (E) Choice (E) is correct. The firm pays $250 plus (10 × $50) to buy another ton
of sugar, so MFC is $750.
436. (A) Choice (A) is correct. If the price of a factor of production decreases, the mar-
ginal cost of another unit of output decreases as well.
437. (A) Choice (A) is correct. An oligopsony will have lower wages because firms will
not hire as many workers as they would with perfect competition.
438. (C) Choice (C) is correct. Consumers don’t demand aluminum; they demand
bicycles, so the bicycle manufacturer buys aluminum to satisfy bicycle demand.
439. (A) Choice (A) is correct. Frosting demand will shift to the right because of
derived demand, increasing the frosting price and quantity.

Chapter 6
440. (D) The correct choice is (D). The company could pay a tax of 1.5 percent, as this
is equal to $15,000. This would be the most precise amount to equalize the harm the
company does to society through carbon dioxide emissions, compared with the benefits
that the local community could do with $15,000 in taxes.
441. (E) The correct choice is (E). Antitrust laws are used when the government is
afraid that a potential merger or acquisition could lead to a company having a too
significant share of the market.
442. (E) Choice (E) is correct. If a good is produced that not all people enjoy the
benefit of, it is known as a private good. A private good is a good that can rival
other goods and is excludable to other consumers. For example, if a person does not
have the money to pay for a taxi, then he or she is forced to find another mode of
transportation.

AP Microeconomics_p001-224.indd 202 22/12/20 5:00 PM


Answers ❮ 203

443. (A) Choice (A) is correct. If a positive externality is produced, a person who does
not consume or work on the product benefits from it nonetheless. This is known as a
marginal external benefit (MEB). The demand curve would not reflect MEB, unless
the government intervened and internalized it.
444. (C) Choice (C) is correct. Taxes such as a progressive tax help with the redistribu-
tion of wealth.
445. (B) A negative externality refers to costs in the production of goods and services that
are put upon a third party. For example, pollution is an example of a negative externality.
Therefore, any answer that describes a benefit to society should be eliminated. Choice (B)
is the best answer. Choice (A) is incorrect because it describes egalitarianism.
446. (C) Choice (C) is correct. A free rider is a person who benefits from goods and
services without incurring any of the costs. The goal of a private firm is to earn a profit;
if free riders are enjoying the benefits of a good or service, then firms do not produce.
447. (C) Choice (C) is correct. A positive externality refers to the benefits experienced
by a third party outside the production of a good or service. Therefore, any choice
that describes a cost to society should be eliminated. Choice (A) is incorrect because it
describes egalitarianism.
448. (A) Choice (A) is correct. If society would benefit when the good has a higher
price and a lower quantity produced, then a negative externality exists.
449. (B) The correct choice is (B). This is the only example of a negative externality of
production. Both (A) and (C) are positive externalities of production.
450. (B) The correct choice is (B). Samantha is paying more taxes as her earnings go up,
so it is clear that this is an example of a progressive tax system, (in which taxes increase
with earnings). When Samantha earned only $25,000, her tax rate was 10 percent, but
it was 50 percent when she earned $72,000.
451. (B) The correct choice is (B). By providing a company with the subsidy, it is more
likely that the company will produce at the level that will meet the demand for the
vaccine.
452. (C) The correct choice is (C). By taking a look at Tim’s previous taxes, it is clear
that his country has a flat tax rat which is equal to 10 percent. Ten percent of Tim’s
earnings in 2020 is equal to $13,500.
453. (B) Choice (B) is correct because since the government intervened and shifted
the supply curve to the left, production will decrease and the negative externality of
overproduction will be corrected.
454. (D) Choice (D) is correct. A free rider is a person who benefits from goods and
services without incurring any of the costs. The goal of a private firm is to earn a profit;
if free riders are enjoying the benefits of a good or service, then firms do not produce.

AP Microeconomics_p001-224.indd 203 22/12/20 5:00 PM


204 ❯ Answers

455. (D) The correct choice is (D). The Gini coefficient can only be between 0 and 1.
The closer the value is to 0, the lower the level of inequality. In the case of this table,
Slovenia has a Gini coefficient of roughly 0.2, which would indicate that there are no
significant issues concerning inequality.
456. (A) Choice (A) is correct. A tax is regressive if the proportion of income paid in
taxes decreases as income increases. A sales tax is an example of this because people with
low incomes pay a higher portion of their money on sales tax compared with people
with high incomes.
457. (A) Choice (A) is correct. Compensating differential refers to the measurement
between the unpleasantness of a job and wage. For example, a firefighter is doing a
more dangerous job than a telemarketer. It does not place the value of one job over the
other; it only means that these jobs are different. Although college professors may be
more esteemed in the workforce because of their level of education, many plumbers
earn more money. The key difference is the nonmonetary differences of a job.
458. (D) Choice (D) is the best answer because the fighter jet is purchased by the gov-
ernment, not an individual consumer, and all domestic consumers receive its national
defense benefits.
459. (C) Choice (C) is correct. A proportional tax is a tax that is paid from someone’s
income regardless of income level and is also known as a flat tax.
460. (D) The correct choice is (D). Regardless of income, Bill has always paid a 20 per-
cent tax rate. On the other hand, Caroline paid a lower tax rate as her income increased,
meaning that this is an example of a regressive tax rate.
461. (C) Choice (C) is correct. A proportional tax is a tax where the same proportion
of income is paid in taxes regardless of income level. Anna received a raise in salary, but
she still paid 20 percent of her income in taxes.
462. (D) A public good is a good that is nonexcludable. Choice (D) is the best choice
because firefighters must respond to help, regardless if the person paid his or her taxes.
463. (A) A Gini ratio is a measure of income inequality. If the Gini ratio is closer to
zero, income distribution is more equal. If the Gini ratio is closer to 1, income dis-
tribution is more unequal. Choice (A) is the best answer because this illustrates the
definition of a Gini ratio.
464. (E) Choice (E) is correct. The diagram is called a Lorenz curve: a graph that illus-
trates how a nation’s income is distributed throughout a nation’s households.
465. (A) Choice (A) is the best answer because a Gini ratio of 0.9 is very close to 1;
therefore, it shows a highly unequal distribution of income.
466. (E) Choice (E) is the best answer because this scenario represents a regressive
tax. A regressive tax is a tax where the proportion of income paid in taxes decreases as
income rises. There is a big difference in Anna and Sarah’s salaries, yet Sarah pays only
$2,000 more in taxes than Anna does.

AP Microeconomics_p001-224.indd 204 22/12/20 5:00 PM


Answers ❮ 205

467. (D) The correct choice is (D). Public goods, such as official government statistics,
are non-excludable. This means that everyone has access to them.
468. (A) Choice (A) is correct. A regressive tax is a tax where the proportion of income
paid in taxes decreases as income rises. Choice (B) is incorrect because a proportional
tax is one that is paid from someone’s income regardless of income level. Choice (C)
is incorrect because a flat tax is the same as a proportional tax. Choice (D) is incorrect
because a progressive tax is one where the proportion of income paid in taxes increases
as income increases. Choice (E) is incorrect because a tax bracket is based on a person’s
income level: as your income increases, you might fall into a tax bracket that pays
higher taxes.
469. (A) Choice (A) is correct. A progressive tax is one in which the proportion of
income paid in taxes increases as income increases.
470. (B) Choice (B) is correct. A private good is a good that can rival other goods and is
excludable to other consumers. If a person does not have the money to pay for a taxicab
ride, then he or she is forced to find another mode of transportation.
471. (D) Choice (D) is correct. Since the world benefited from this satellite transmis-
sion, the satellite is a public good. A public good is a good that is nonexcludable.
472. (C) Choice (C) is the best answer because taxes (such as a progressive tax) help
with the redistribution of wealth. Choice (A) is incorrect because other economic sys-
tems help in the redistribution of wealth, such as a mixed economic system. Choice (D)
is incorrect because it does not differentiate income levels on the percentage of tax paid.
473. (A) The correct choice is (A). These taxes are used when there is a need to tax a
market that has some type of negative externality. An example would be taxing an oil
manufacturing company for excessive carbon dioxide emissions.
474. (E) The correct choice is (E). By doing so, the government would effectively be
encouraging companies to use more solar panels rather than nonrenewable sources of
energy.
475. (B) The correct choice is (B). As the fact that the company has produced this
product will provide benefit to other elements of society, this is an example of a positive
externality of production.
476. (D) The correct choice is (D). Efficiency-enhancing taxes are used to prevent a
specific negative externality, and the example in (D) is a tariff caused by regular market
conditions.
477. (D) Choice (D) is correct. The shopkeeper’s income rose and the total tax
remained the same, so the pension tax is a regressive tax.
478. (B) Choice (B) is the best answer. The marginal cost of air pollution to society is
$10 per barrel, so charging gas stations a tax of $10 per barrel would make the marginal
social cost of fuel equal the marginal social benefit of fuel.

AP Microeconomics_p001-224.indd 205 22/12/20 5:00 PM


206 ❯ Answers

479. (A) Choice (A) is the best answer. A Pigovian subsidy paid to bus companies will
increase the supply of buses, shifting the bus supply curve to the right.
480. (A) Choice (A) is the best answer. A Pigovian subsidy paid to consumers will shift
demand for the subsidized good to the right.
481. (B) Choice (B) is the best answer. A police car provides law enforcement for all
citizens without charging additional fees, and the service is non-​rival because police
normally have enough cars to enforce the law for all citizens.
482. (A) Choice (A) is the best answer. The Gini ratio fell, so the income distribution
in country X became more equal.
483. (B) Choice (B) is the best answer. The Gini ratio increased, so the income distri-
bution in country Y became less equal.
484. (D) Choice (D) is correct. Country B has a lower Gini ratio than country C so it
has a more equal income distribution.
485. (B) Choice (B) is correct. The progressive tax will result in a more equal income
distribution, and a lower Gini ratio, for country E.
486. (B) Choice (B) is correct. The regressive tax will result in a less equal income
distribution, and a higher Gini ratio, for city C.
487. (B) Choice (B) is the best answer. While the birds are a nonexcludable resource,
there is a limited supply of birds, so they are a rival good as well. Thus, they are a com-
mon resource and not a public good.
488. (C) Choice (C) is the best answer. If the bridge has sufficient capacity, it is a non-
rival good. The bridge does have a toll, so it is an excludable resource. Thus, the bridge
is a quasi-​public good.
489. (D) Choice (D) is correct. When there is little traffic, the freeway is a public good
because it is nonexcludable and non-​rival. During rush hour, the freeway becomes a
rival good (while remaining nonexcludable) so it becomes a common resource. High
utilization can also change a quasi-​public good into a private good temporarily.
490. (A) Choice (A) is the best answer. Consumers can reach an agreement on negative
externalities without government intervention when negotiating costs are low. Reaching
an agreement requires MC = MB.
491. (A) Choice (A) is the best answer. The pollution affects many people in the city,
which complicates the negotiation process and makes the factory’s liabilities unclear.
It would be more effective for government regulators to set a price on the pollution
caused by the factory.
492. (B) Choice (B) is correct. If society would benefit if production increased and the
price fell, a positive externality exists.

AP Microeconomics_p001-224.indd 206 22/12/20 5:00 PM


Answers ❮ 207

493. (E) Choice (E) is the best answer. Subsidized student loans are a consumer subsidy.
If grants are available to all students, university education is nonexcludable (since anyone
can get a grant) and rival (since universities can admit a limited number of students)
so it is also a common resource.
494. (D) Choice (D) is the best answer. A property tax exemption would benefit uni-
versities directly, subsidizing the producer. If many students still can’t afford tuition,
university education is excludable, as well as rival because universities can admit
a limited number of students.
495. (B) Choice (B) is the best answer. The government used resources to educate its
citizens, and businesses received the spillover benefit of a more productive workforce.
Choice (E) is not correct, because an alternative to publicly funded education is firms
providing education and charging tuition fees.
496. (D) Choice (D) is the best answer. Demand for public goods is considered a phan-
tom demand. An individual does not decide to purchase one unit of a public good by
considering the marginal benefits and costs of buying one more unit.
497. (B) Choice (B) is the best answer. The demand curve for a public good is the
vertical sum of each consumer’s demand curve.
498. (D) Choice (D) is correct. This graph shows demand curves for a public good.
499. (D) Choice (D) is correct. The vertical sum of MB1, MB2, and MB3 (MB SUM) =
MC at the equilibrium price of the public good.
500. (B) Choice (B) is correct. For a public good, the sum of consumers’ demand
curves will decline more sharply than the demand curve for an individual consumer.

AP Microeconomics_p001-224.indd 207 22/12/20 5:00 PM


NOTES

AP Microeconomics_p001-224.indd 208 22/12/20 5:00 PM


NOTES

AP Microeconomics_p001-224.indd 209 22/12/20 5:00 PM


NOTES

AP Microeconomics_p001-224.indd 210 22/12/20 5:00 PM


NOTES

AP Microeconomics_p001-224.indd 211 22/12/20 5:00 PM


NOTES

AP Microeconomics_p001-224.indd 212 22/12/20 5:00 PM


NOTES

AP Microeconomics_p001-224.indd 213 22/12/20 5:00 PM


NOTES

AP Microeconomics_p001-224.indd 214 22/12/20 5:00 PM


NOTES

AP Microeconomics_p001-224.indd 215 22/12/20 5:00 PM


NOTES

AP Microeconomics_p001-224.indd 216 22/12/20 5:00 PM

You might also like

pFad - Phonifier reborn

Pfad - The Proxy pFad of © 2024 Garber Painting. All rights reserved.

Note: This service is not intended for secure transactions such as banking, social media, email, or purchasing. Use at your own risk. We assume no liability whatsoever for broken pages.


Alternative Proxies:

Alternative Proxy

pFad Proxy

pFad v3 Proxy

pFad v4 Proxy